UW Qbank Step-3 MW

April 2, 2017 | Author: Sukhdeep Singh | Category: N/A
Share Embed Donate


Short Description

Download UW Qbank Step-3 MW...

Description

1. Even though it is not common, a new pleural effusion in a patient already on anti-tuberculosis treatment may occur and must be studied, because this can progress despite the clinical improvement of the patient. There is no need to change therapy, unless there is proof of a new infection or drug resistance. Some authors advocate the use of steroids in this setting, but this is not a universally accepted practice. There have been reported cases of new pleural effusions or lung infiltrates during the course of treatment of tuberculosis in patients who are recovering from the disease. The new effusions are usually exudates with lymphocytic predominance. Granulomatous lesions can be found on biopsies, but AFBs are not usually present. Thoracentesis is usually sufficient for diagnostic and therapeutic purposes. It is extremely rare to find new evidence of tuberculosis, as this new effusion is usually an enhanced immunologic response. It is not an indicator of failure of therapy or drug toxicity. If the patient is having systemic symptoms (such as ongoing fever, weight loss, etc.), a bacterial infection or empyema could be suspected, prompting the need for broad-spectrum antibiotic coverage, and maybe even chest tube placement 2. The consumption of undercooked meat during pregnancy may be associated with congenital toxoplasmosis, which can manifest as microcephaly or other abnormalities, such as chorioretinitis, MR, deafness, & seizures Domestic cats are definite hosts for T. gondii Humans can acquire the infection by: (1) the consumption of raw or undercooked meat of infected animals (including lamb, beef, or game), or (2) contact with cat feces Other congenital infections of the TORCH group can also result in microcephaly Asymmetric growth retardation (normal weight, decreased head circumference) is not characteristic for poor infant feeding Maternal smoking during pregnancy is associated with low-birth-weight infants, not microcephaly Excessive maternal caffeine consumption during pregnancy seems to be associated with an increased risk of spontaneous abortion and stillbirth Vast experience with the use of methyldopa & magnesium sulfate (for the treatment of PIH and preeclampsia) indicates that these drugs are unlikely to be associated with long-term adverse outcomes 3. Rupture of chordae tendineae should be suspected in healthy individuals who develop flash pulmonary edema (heart failure) associated with an acute mitral regurgitation. The patient presents with signs and symptoms of acute heart failure (sudden onset of shortness of breath, diaphoresis, Pallor). His EKG findings reveal occasional PVC; there are no signs of ischemia or ventricular hypertrophy. His pathologic murmur (systolic murmur that is heard in the apex, radiates to the axilla, increases with the grip maneuver, and decreases with Valsalva) is characteristic of mitral regurgitation, which may lead to acute heart failure. Acute MR is usually characterized by a soft, decrescendo systolic murmur (can be early, midsystolic or holosystolic), a decreased first heart sound, and the presence of a fourth heart sound. The 4 common causes of acute heart failure are papillary muscle rupture secondary to ischemia, infective endocarditis, rupture of chordae tendineae, and chest wall trauma secondary to mitral valve rupture. The most common cause of isolated, severe acute mitral regurgitation in adults is rupture of chordae tendineae with or without associated myxomatous disease. The diagnosis is confirmed by echocardiography. Myocardial infarction can be complicated by acute mitral regurgitation when there is rupture of the papillary muscle; however, this patient does not have evidence of ischemia in the EKG. Patients with pulmonary embolism can develop acute cor pulmonale; however, in such a setting, the EKG will show right axis deviation, right bundle branch block or both. The above patient clearly has acute pulmonary edema. Spontaneous papillary muscle rupture usually presents in elderly people who have acute chest pain or as a complication of MI. None of these are present here. The patient has features compatible with Ehlers-Danlos syndrome. This condition can cause a myxomatous degeneration of the mitral valve, leading to acute rupture of chordae tendineae. Pes planus and scoliosis are frequent, though not specific findings. The skin can be velvety or thin, and is usually covered with multiple characteristic "cigarette-paper" scars, due to its frailty and easy

bruisability. A past history of hernias and mitral valve prolapse is not uncommon. Joint hypermobility and skin hyperextensibility ("rubber man syndrome") can be dramatic in severe cases. Thyrotoxicosis can lead to acute, high-output, heart failure; however, this will not explain the MR. Skin changes of thyrotoxicosis can be confounded with those found in Ehlers-Danlos, but other characteristic signs & symptoms are lacking. Rheumatic fever is a common cause of mitral regurgitation in young individuals; however, the course of the disease is chronic, and the patients usually have evidence of left ventricular and atrial hypertrophy in the EKG. Marfan syndrome can affect the valvular apparatus, causing degeneration of the mitral & aortic valves. The presentation is usually chronic, progressive MR, and rarely, acute regurgitation due to ruptured chordae tendineae. This patient does not have arachnodactyly, loose joints or increased arm span, which are characteristic of Marfan syndrome. Educational Objective: Marfan or Ehlers-Danlos syndrome must be suspected in patients with connective tissue abnormalities and an acute MR secondary to chordae tendineae rupture, although a primary, pre-existing mitral valve prolapse (MVP) is the most common cause. Some cases may be idiopathic (individuals who experience rupture of the chordae tendineae without previous MVP or connective tissue disease), but an etiology can be found in most occasions. 4. Organ transplantation from cadaveric donors has become increasingly common in the past few years. Most of the organs are obtained from brain dead donors. The successful recovery of viable organs for transplantation depends on appropriate identification and medical care of brain dead patients. It also depends on the speed of recovery of the organs from the potential donors, with a shorter time interval between brain death and organ recovery leading to better outcomes. Any delay in organ procurement leads to an increase in the number and severity of complications; therefore, this patient should be immediately transferred to a specialized transplant center. Brain dead organ donors should ideally be managed in an intensive care setting. The goal of intensive medical care is to achieve hemodynamic stability and maintain physiologic homeostasis to improve the viability of the organs. Particular attention should be paid to maintain a normotensive and euvolemic state. One of the goals is to provide optimal ventilator support to prevent hypoxia and hypercapnia. Completely discontinuing or changing the ventilator setting is not appropriate when the patient is hemodynamically stable. Hypotension is very common in brain dead patients. It is related to the loss of sympathetic tone, systemic infections, volume depletion secondary to losses and diabetes insipidus. Adequate volume resuscitation is therefore an important step in ensuring donor organ viability. IV hydration should be continued to maintain euvolemia and an adequate urine output. Waiting for a cardiac arrest before procuring the organs is not the standard of care, and is not recommended. Educational Objective: Coordination of care between the different teams involved in organ procurement is a crucial step in the management of brain dead organ donors. 5. This patient is deeply demented, completely dependent on others for her care, and is most likely developing pneumonia. Her current condition has made her incapable of making any decisions regarding her health. She needs a surrogate decision-maker to speak on her behalf, and to preserve her right of autonomy. Her son seems to be the most appropriate person for this position, as he is the only direct relative who is constantly visiting her. The physician must always consider the patient’s clinical picture and prognosis, and respect the opinion of the surrogate decision-maker. Although it is very unusual, dementia can be an indicator of a terminal condition, and aspiration pneumonia commonly presents towards the last episodes of the disease. If the patient or surrogate feels that no further therapy should be given since the disease is non-reversible, and because the patient’s quality of life is poor, the physician has no right to give further treatment. In this case, the physician has to respect the son’s request that his mother receive only oxygen therapy Studies have shown that it is safe to treat elderly patients with pneumonia in a nursing home if they are not critically ill. However, this patient should be transferred to the hospital if her son wants her to be treated with antibiotics and full supportive care. Pain treatment may eventually be needed if the shortness of breath does not improve with oxygen therapy; however, it is not indicated at this point. It is the physician’s duty to always act in the patient’s best interest. In this case, he may at least alleviate the patient’s pain and give comfort, despite the seriousness of the disease. It is unethical to

do nothing even when the patient’s prognosis or quality of life is poor. Giving oxygen therapy is the most appropriate next step in management because aside from this step being requested by the surrogate, doing so may control the patient’s symptoms. Giving antibiotics to the patient may indicate the physician’s inattention and lack of regard for the family (or surrogate’s) decision Educational Objective: Severely demented patients in nursing homes have a poor quality of life and can be viewed as individuals who are in the terminal phase of a prolonged and debilitating illness. The physician has no right to give or withhold potential life-saving measures based on only his own values, personal evaluation, or opinion of what the acceptable quality of life is. Decisions regarding the treatment of acute and life-threatening conditions have to be made with the surrogate, who acts to preserve the patient’s right to autonomy. On the other hand, the physician cannot be forced to provide unnecessary therapy only because the surrogate of relatives request so. 6. Studies have shown that adolescents in a private practice setting are very concerned about the prospect of gaining weight secondary to OCP use. However, available data has demonstrated that OCPs do not cause an increase in body weight or percent body fat. Occasionally adolescents voice other concerns about OCP use, including fears of developing blood clots, birth defects, & infertility, but these concerns are more common in the lower socioeconomic groups. Confidentiality concerns, cost, desire to have children, and partner’s opposition are rarely cited by adolescents as reasons not to use OCPs 7. Physicians are often faced with difficult scenarios involving the withdrawal of life-sustaining treatment. The situation may arise when a patient is rendered incompetent or is unable to participate in decision-making, and does not have any advance directives. In such situations, the physician must still recognize and respect the patient’s autonomy and right to make healthcare decisions. It is the physician’s responsibility to act in the patient’s best interest by identifying a surrogate who must make healthcare decisions for the patient based on substituted judgment. The surrogate speaks on the patient's behalf, and must have the most knowledge on what the patient would have done or wanted if he were able to make his own healthcare decisions. The patient’s spouse or next of kin usually acts as the surrogate decision-maker in the absence of a formally or legally designated surrogate. The patient’s son appears to have some insight into his father’s wishes when he claimed that "he would have never wanted to live like this." The patient may have previously expressed his wishes regarding life-sustaining treatment to his son. The son should therefore be asked to provide more information and reasoning behind the decision to withdraw mechanical ventilation at this point. Telling the son that mechanical ventilation cannot be discontinued without an advance directive is incorrect. The son can assume the role of surrogate decision-maker as long as the physician believes that he is acting in the patient’s best interest. The physician should discuss the situation with the son in greater detail before involving the family. Understanding the reason and thoughts behind the son's opinion should be attempted to ascertain if he can act as the patient's surrogate decision maker. Once a surrogate is identified, the decision to withdraw or maintain ventilatory support can then be made. Important: Before making any decisions regarding the withdrawal of life support measures, it is important for a physician to act in the patient's best interest by identifying a surrogate, with whom he must effectively communicate and discuss all issues and concerns. Quite frequently, a physician is faced with a situation wherein multiple first-degree relatives cannot agree on the approach to medical care, despite appropriate and adequate counseling. In such cases of conflict, the hospital’s ethics committee should be involved to act as a mediator between the different family members. In extreme cases, the case may need to be taken to court, where a guardian is appointed to assist in the medical decision-making. All the family members should be involved in the decision-making process regarding the withdrawal of mechanical ventilation. Important: The hospital ethics committee should be involved if a conflict exists between multiple family members regarding the appropriate approach to the patient's medical care despite adequate mediation by the physician.

8. Analgesic nephropathy is seen in patients with heavy, long-term use of aspirin, phenacetin, acetaminophen or other nonsteroidal antiinflammatory drugs. Cumulative amounts of analgesics play a very significant role in the pathophysiology of analgesic nephropathy. Chronic necrosis of the renal papilla with calcification is typically seen. Acute papillary necrosis can also occur and lead to acute renal colic. Urine examination reveals hematuria, proteinuria, and pyuria. The urine culture is usually sterile. The associated anemia is out of proportion to the renal failure, & this is possibly due to GI blood loss. Discontinuation of the analgesic causes stabilization or even improvement in renal function Patients with pyelonephritis have renal colic, but the presentation is not sudden. Fever & dysuria are also commonly observed with this disease A unilateral ureteric calculus will not lead to an increase in the BUN & Creatinine levels Prerenal azotemia is associated with a decrease in renal perfusion and an increase in the BUN level > creatinine; the urine specific gravity is high. Allergic interstitial nephritis characteristically produces azotemia, skin rashes, and eosinophilia. Urine eosinophils can be detected by using Hansel’s stain 9. Annual screening for prostate cancer should be performed between the ages of 50 and 70 using digital rectal examination and measurement of serum PSA levels; however, those with a higher risk for prostate cancer (i.e., African-Americans, those with a significant family history) should begin annual screening at 45 years of age

10. This patient presents with polyarticular joint pain of acute onset (< 6 weeks). The two important differential diagnoses that should be considered in patients with such a scenario are viral arthritis and early systemic rheumatic disease. Since the duration of the patient's symptoms is relatively short, and there exists a temporal relationship with an upper respiratory infection, viral arthritis should be the primary consideration. Rheumatoid arthritis (RA)-like symptoms with small joint involvement and a weakly positive RF test are especially characteristic for parvovirus infection. In this case, appropriate diagnostic tests should be ordered and the patient should be carefully followed-up RA is unlikely if there is no evidence of synovitis and the symptoms are of < 6 weeks duration (The hallmarks of synovitis include soft tissue swelling, warmth over a joint and joint effusion) SLE may be a possibility, but a more widespread symptomatology is usually more typical of this diagnosis. Furthermore, this patient does not fulfill the diagnostic criteria for SLE. Rheumatic fever occurs after streptococcal pharyngitis and may initially manifest with inflammatory joint disease, but involvement of the small joints is not common and joint involvement is migratory in nature. Sarcoidosis is a systemic disease that may have joint manifestations, but accompanying symptoms (e.g., pulmonary symptoms, erythema nodosum, neurological symptoms) are usually present. Viral arthritis is typically self-limited and of short duration. Therapy is generally directed at the relief of symptoms and maintenance of function; therefore, patients are treated with simple analgesic and anti-inflammatory drugs. Physical and occupational therapy may be employed if required to maintain or improve function Joint deformity is a very uncommon complication in these patients. Rheumatoid arthritis can cause significant joint deformities without appropriate long-term treatment. Joint deformities are less characteristic for other systemic diseases such as SLE and rheumatic fever, but involvement of other organs may be significant in these diseases 11. Assessment of immune status is important in patients with HIV infection in terms of susceptibility to various opportunistic agents and available options for treatment and prophylaxis. Currently, two indicators of disease progression in HIV patients are emphasized as important tools for the assessment of disease progression: viral load and CD4 count. CD4 lymphocyte count is an indicator of the current level of immunosuppression, and is referred to by some authors as, ‘the immunologic damage that has already occurred.' Viral load is a good marker of disease activity, or the potential for future damage to the immune system. It is referred to as ‘the damage that is about to occur.’

Therefore, plasma viral load has prognostic significance at any level of CD4 count in patients with HIV infection. Other than absolute CD4 lymphocyte count, CD4 percentage and CD4/CD8 ratio are sometimes used to assess immune status in patients with HIV infection, but these are less useful indicators. p24 and p41 antibodies are used as serological markers to diagnose HIV infection, but are not employed to assess the severity of the disease. 12. The patient in the vignette has a clinical presentation consistent with a partial small bowel obstruction. The presence of air in the distal colon makes the diagnosis of complete obstruction less likely. Partial small bowel obstruction should be initially managed with observation and supportive treatment (e.g., intravenous hydration, nasogastric suctioning and correction of electrolyte abnormalities). If the patient fails to improve in the next 12 to 24 hours, early surgical intervention is recommended. Patients with partial small bowel obstruction should be managed by conservative therapy initially before proceeding to invasive interventions; however, patients with signs of impending strangulation (incarcerated hernias) or mesenteric ischemia should undergo urgent surgical intervention to prevent further deterioration of the clinical status. Colonoscopy has no role in the management of patients with small bowel obstruction. A rectal tube is not indicated in patients with small bowel obstruction. 13. Lesbian women often conceal their sexual orientation from their primary care physicians. Recent developments and legislations regarding same sex marriages have encouraged more homosexual couples to "come forward" or reveal their sexual orientation. It is important for primary care physicians to be sensitive and knowledgeable to their special needs and concerns. Human papilloma virus (HPV) infection has been strongly linked with the development of cervical intraepithelial neoplasia (CIN) and cervical cancer. Sexual intercourse, especially with multiple new partners, is the main risk factor for the acquisition of HPV infection. The risk of acquiring HPV infection (and, hence, CIN/cervical cancer) is much lower in lesbian women if they do not engage in sexual intercourse with men. Transmission of HIV infection, though rare, can occur via exposure to cervical and vaginal secretions of an HIV-infected patient. Lesbian women have a lower risk of contracting syphilis and Chlamydia infections than heterosexual (women who have sex with men) or bisexual women (women who have sex with both men and women). Lesbian women should be given Hepatitis B vaccination. Although they are at much lower risk of acquiring hepatitis B infection than gay men, they can still acquire the infection via vaginal and cervical secretions, especially if they have multiple or new partners. 14. Central venous catheters are very commonly used in hospitalized patients and in outpatient settings. Catheter-related infections are a major complication of their prolonged use. Common clinical features include sudden onset of fever with chills, hypotension, altered mental status, and nonspecific GI symptoms. Local infection at the insertion site, exit site, or subcutaneous tunnel manifests as erythema, induration, and tenderness at the site, with pus coming out of the exit site. The general approach in treatment depends on a variety of factors - local vs. systemic infection, type of device used (tunneled vs. non-tunneled), the infecting organism, and the status of the host (immunocompromised or neutropenic). In the above vignette, suspicion of a systemic infection (fever, chills, with altered mental status) in an immunocompromised patient with a tunneled catheter (Hickman, Broviac, Groshong, or Quinton) as a suspected source warrants the removal of the catheter. The patient should also be started on empiric therapy with vancomycin (for gram positive organisms) and gentamicin (for gram negative bacilli). This should be continued until final microbiologic identification of the causative organism is obtained. The outcomes with central catheter-related infections are significantly affected by the speed of initiation of anti-microbial therapy. Waiting for laboratory data and culture results can lead to an inappropriate delay in treatment and can adversely affect clinical outcomes. The catheter tip should be sent for cultures after removal of the catheter; however, initiating empiric antibiotics remains as the priority. Rifampin with a fluoroquinolone has been used in some preliminary studies for Staphylococcus aureus infections, and there have been good results; however, these drugs should not be used as an empiric therapy for suspected infections.

Metastatic infections and complications are common in patients with Staphylococcus aureus infections. Examples include septic thrombophlebitis, infective endocarditis, osteomyelitis, and rarely, retinitis. The diagnosis is suspected by persistent bacteremia or unchanged clinical status despite adequate treatment. This patient appears to have developed vertebral osteomyelitis or diskitis (infection of the intervertebral disk space), which is a well-recognized complication of catheter-related systemic infections. It usually presents with an insidious onset of low back pain, local tenderness to spinal percussion, reduced back mobility, and spasm of nearby muscles. MRI is the investigative procedure of choice for vertebral osteomyelitis (highly sensitive in detecting vertebral osteomyelitis and/or diskitis). Typical MRI findings in vertebral osteomyelitis include decreased signal intensity in the disk and adjacent vertebral bodies, loss of endplate definition, contrast enhancement of the disk, adjacent vertebral bodies and involved paraspinal and paravertebral soft tissues. Finally, a CT-guided needle biopsy is generally necessary to confirm the clinical and/or radiographic suspicion of the presence of vertebral osteomyelitis or diskitis. Typical radiographic changes of vertebral osteomyelitis consist of destruction of the vertebral bodies with collapse of the intervening disk space. Plain radiographs are often normal in the early phases of infection. The appropriate therapy for vertebral osteomyelitis is to continue vancomycin alone for at least six weeks. Prolonged therapy (12 weeks) is usually required for patients with extensive bone destruction, adjacent soft tissue or paravertebral infection. Switching to linezolid or adding gentamicin is not recommended as long as the bacteria are susceptible to the drug. 15. An upright chest x-ray is the initial test of choice to confirm the diagnosis of pneumothorax. The accumulation of air occurs primarily in the apical and lateral regions when the patient is upright, and is usually seen as a convex white visceral pleural line on x-ray. As little as 50 ml of pleural gas can be visible on upright x-ray. It has been stated that an expiratory x-ray may significantly improve the rate of detection of pneumothorax; however, it has been shown that this statement is incorrect. As an example, one study of 85 patients with pneumothoraces and 93 controls found that inspiratory and expiratory upright chest radiographs have equal sensitivity for pneumothorax detection. Furthermore, considering the limitations of expiratory radiographs, only inspiratory films are recommended as the initial examination of choice for pneumothorax detection. A lateral decubitus chest x-ray may be used to look for a small pneumothorax, but it is not routinely employed as the initial test of choice. CT scan is not recommended for routine use, but may be helpful in selected cases (e.g., to distinguish between a large bulla and a pneumothorax) 16. Ampicillin-associated maculopapular rash is a well-known phenomenon in patients with infectious mononucleosis. The reported incidence of this reaction is as high as 80%. It is believed that this vasculitic rash is immune-mediated, and is caused by circulating IgG and IgM antibodies toward penicillin derivatives. Such antibodies have actually been demonstrated in patients with EBVassociated infectious mononucleosis. The rash does not represent immediate or delayed hypersensitivity to ampicillin, which can be used safely when the infection subsides. Supportive treatment and observation are the mainstays of treatment for individuals with infectious mononucleosis. Supportive treatment includes acetaminophen and NSAIDs for fever, throat pain and malaise, as well as adequate nutrition, fluids, and rest. The antibiotic should be discontinued. Corticosteroids are reserved for patients with severe complications such as impeding airway obstruction, liver failure, or aplastic anemia. Although acyclovir is effective in inhibiting EBV replication, it has not been shown to have significant clinical benefits in patients with infectious mononucleosis. 17. Most often occurring in middle-aged women, Sjogren’s syndrome (SS) is a chronic and progressive autoimmune disorder characterized by lymphocytic infiltration of the exocrine glands. Primary symptoms include dry eyes and dry mouth secondary to lacrimal and salivary gland involvement. The syndrome is commonly associated with other autoimmune conditions, such as SLE or scleroderma. To establish that the dryness experienced by the patient is secondary to SS, laboratory evaluation for characteristic autoantibodies (i.e., anti-Ro/SSA or anti-La/SSB) should be undertaken. Quantifying breakup time after fluorescein staining of the cornea is a means of measuring tear film instability. Since this test requires slit lamp examination, it is usually performed by an ophthalmologist. Although

fluorescein staining is a useful means of objectifying patient complaints about dry eyes, it is not a definitive method of diagnosing Sjogren’s syndrome. The instillation of Rose Bengal into the eye will stain areas of devitalized tissue. This staining allows for the objective measurement of damage to conjunctival and corneal epithelial cells. Since the Rose Bengal test requires slit lamp examination, it is usually performed by an ophthalmologist. Although Rose Bengal staining is a useful means of objectifying patient complaints about dry eyes, it is not a definitive method of diagnosing Sjogren’s syndrome. Parotid gland biopsy is not commonly performed when evaluating patients for Sjogren’s syndrome. The Schirmer test is used to measure tear production. The test involves inserting a small piece of filter paper into the lower eyelid to quantify the extent of wetting within a certain time frame. Although the Schirmer test is a useful means of objectifying patient complaints about dry eyes, it is not a definitive method of diagnosing Sjogren’s syndrome. Patients with suspected Sjogren’s syndrome (SS) should have the diagnosis confirmed with a labial salivary gland biopsy, which is considered the "gold standard." The biopsy sample is obtained from a normal-appearing portion of the lower lip, and the classic histologic finding in SS is focal collections of lymphocytes. Anti-centromere antibodies are suggestive of CREST syndrome, not Sjogren’s syndrome. Although an elevated erythrocyte sedimentation rate is found in up to 70% of patients with SS, this finding is not specific and cannot be used to confirm the diagnosis. Malignant lymphoproliferative disorders are more common in patients with Sjogren’s syndrome (SS). The lymphocytic infiltration of exocrine glands seen in SS is typically accompanied by polyclonal B-cell activation, as evidenced by the presence of autoantibodies anti-Ro/SSA and anti-La/SSB. The resulting chronic, excessive B-cell stimulation contributes to the increased incidence of non-Hodgkin's lymphoma (extranodal marginal zone B-cell lymphoma) in this patient population. Studies indicate that the time between the onset of SS and the diagnosis of non-Hodgkin’s lymphoma varies from 4 to 12 years. Fortunately, this increased incidence of lymphoma is not associated with an elevated risk of death in the SS patient population. T-cell lymphoma is associated with infection with human Tlymphotrophic virus, type 1 (HTLV-1). There is no known association between this condition and Sjogren’s syndrome. Acute pancreatitis is associated with mumps infection, which can cause parotid gland inflammation. Pancreatitis has not been associated with Sjogren’s syndrome. The risk factors for development of carcinoma of the salivary glands are radiation exposure, Epstein-Barr virus infection, genetic factors (e.g., tumor gene inactivation), environmental factors (e.g., exposure to silica dust or kerosene), and dietary factors (e.g., reduced fruit and vegetable intake). 18. Most antibiotics are not neurotoxic, but some are associated with an increased risk of seizures. Conditions that would predispose an individual to antibiotic-induced seizure include renal insufficiency, older age, pre-existing CNS disease, and concomitant use of proconvulsant drugs. Of all antibiotics, beta-lactams are the most commonly associated with adverse CNS events. Specifically, penicillins, cephalosporins, monobactams, carbapenems (including imipenem), and fluoroquinolones are the antibiotics most likely to trigger seizures 19. The patient's history, physical examination findings, thyroid function test results, and thyroid scan results (focal uptake) are very characteristic of a toxic nodule. Graves' disease is the most common cause of hyperthyroidism; however, focal uptake in the thyroid scan rules out this disease. Subacute and painless thyroiditis are both uncommon in elderly patients. These diseases demonstrate a diffuse reduction in radioiodine uptake.

This patient most likely has Marfan syndrome (MFS), a disease characterized by arachnodactyly, increased arm span relative to height, and valvular (mitral or aortic) insufficiency. Dural ectasia is the most common finding (present in more than 90% of patients), and usually requires an MRI of the lumbar spine for the confirmation of the diagnosis. Ectopia lentis can be seen in 50-80% of the patients with MFS, and is characterized by an upward displacement of the lens. Aortic dilatation can

be seen in 50% of the children with MFS and 70-80% of the adults. Some of these patients will eventually develop aortic insufficiency. 20. Diffuse esophageal spasm manifests with chest pain and dysphagia. The etiology is unclear, although in many patients it is associated with emotional factors and functional gastrointestinal disorders. Manometric studies demonstrate high amplitude peristaltic contractions. In contrast to achalasia, the lower esophageal sphincter usually has a normal relaxation response. Manometric findings may be intermittent, thus making the diagnosis difficult. The esophagogram is frequently normal, although the classic corkscrew esophagus is seen occasionally. Treatment is with antispasmodics, dietary modulation, and psychiatric counseling. Surgery is very rarely required for this disorder. Zenker’s diverticulum is a disorder of the proximal esophagus generally seen in females. The diverticulum may vary in size and is generally asymptomatic in presentation. The occasional patient may present with complaints of food sticking in the throat, halitosis, and regurgitation. There is no pain associated with the diverticulum. Treatment is surgery. In achalasia, the lower esophageal sphincter does not relax (high tone). Histopathology reveals hypertrophied, inner circular muscle with the absence or degeneration of ganglia in Auerbach’s plexus. Manometry will show the absence of peristalsis. The cause is not known, but a similar condition in South America is caused by the parasite, Trypanosoma cruzi. An esophagogram typically reveals a dilated esophagus with a bird’s beak narrowing of the distal esophagus. Therapy is balloon dilation of the narrowed esophagus or surgery. Scleroderma is a collagen vascular disorder which can present with loss of distal peristalsis of the esophagus. There is complete atrophy of the esophageal smooth muscle and fibrosis. The lower esophageal sphincter becomes incompetent (low tone) with time, leading to reflux esophagitis and a stricture. The condition is progressive and difficult to treat. Esophagitis can be due to several causes, the most common being Herpes, Candida, or a Cytomegalovirus. Esophagitis generally occurs in immunocompromised individuals (e.g., those with AIDS, malignancy, diabetes) and may present with dysphagia, oral thrush, or odynophagia. Endoscopy with washings, culture, and biopsy may reveal the cause. **Extremely high yield question for USMLE. Understand the pathophysiology, the presence of and the absence of peristalsis, and LES tone in all of the above conditions. 21. Patients with borderline personality disorder suffer from considerable instability in self-image, moods, impulse control, and relationships. Relatively minor events or disagreements are often interpreted as threatening a relationship, causing many borderline patients to respond with dramatic displays of anger or self-harm. Marked changes in mood can occur throughout the day. When primitive idealization occurs, the patient views another individual as perfect and without flaw, and is unable to tolerate any evidence to the contrary. This often happens when the borderline patient interacts with a "savior," someone (such as the physician, in this case) who has cared for her in a time of crisis. Compensation results when an individual overemphasizes achievements in one sphere because of failure in another. One example would be an unattractive individual focusing on scholastic success. Projection occurs when an individual attributes his thoughts or desires ‘especially those that are socially unacceptable’ to another person. One example would be a woman who resents a coworker’s success and says, "He doesn’t like me." Reaction formation is the overcompensation for uncomfortable impulses. One example is a man who is attracted to other men but behaves in a homophobic manner. The hallmark of borderline personality disorder is splitting, a phenomenon in which all external objects are classified as wholly good or wholly bad (a categorization that may abruptly change based on one positive or negative encounter). Patients with borderline personality disorder tend to use a psychological defense mechanism known as splitting, in which all external objects are classified as wholly good or wholly bad. Primitive idealization is one aspect of splitting in which another individual is viewed as perfect. *Extremely important question for USMLE step-3 Dialectical behavior therapy is one of the most successful means of treating borderline personality disorder. It is an intensive process and may take more than a year of therapy before improvement is seen. Unlike traditional psychotherapy (which analyzes unconscious motives), dialectical behavior

therapy focuses on behavior modification and the building of skills. Important issues to address during therapy sessions include the establishment of appropriate boundaries, validation of the patient’s experience, assumption of responsibility for one's own actions, management of feelings on both sides, promotion of reflecting before acting rather than being impulsive, reduction of tendency to engage in splitting, and the setting of limits on self-destructive behaviors. Selective serotonin reuptake inhibitors can help reduce mood lability and temper outbursts in borderline patients; however, medications are not first-line in the treatment of this disorder, and should only be used as a supplement to psychotherapy. Tricyclic antidepressants are contraindicated because of the high risk of suicide attempts. 22. A 32-year-old white female with no significant past medical history comes to the office because she has noticed a small lump in her left breast. She is married, has no children, and works as an attorney in a very busy local bureau. She does not use tobacco, alcohol, or drugs. There is no history of breast cancer in the family. Her last menstrual period was five days ago. On physical examination, you find a 1.5 cm round, smooth, soft, mobile, mildly tender mass in the left breast. No axillary nodes, skin lesions, or nipple discharge are found. Which of the following is the most appropriate next step in her management? A. Fine needle aspiration of the lesion B. Breast ultrasound C. Mammogram D. Reevaluation three weeks from now E. Order serum BRCA1 and 2 Explanation: Female patients younger than 35 years have a decreased risk of breast cancer, especially if there is no positive family history. The patient is a busy woman who needs a fast answer to her problem. At the same time, the characteristics of her breast mass - smooth, soft, mobile, and round - usually correspond to a breast cyst, which may easily be assessed through fine needle aspiration (FNA). (Choice C) A mammogram is not usually recommended in patients less than 35 years of age because the breast tissue at this age is too dense to allow good imaging. (Choice B) If the mass has no cystic characteristics, or if the patient refuses FNA, a breast ultrasound is indicated. (Choice D) Benign lesions are supposed to decrease in size 3 to 10 days after menstruation, not three weeks later. Since the patient came five days after her menstrual period, reevaluation of the lesion will not be useful. (Choice E) BRCA tests are not advisable. Educational Objective: Even though breast cancer is rare in women younger than 35 years, every breast lesion must be taken seriously. Patients can be reevaluated 3 to 10 days after the menstrual period to look for regression. If the lesion appears cystic, FNA should be done unless the patient declines. If the mass looks solid, is too small, or cannot be felt, ultrasound is the next step to determine if biopsy is needed. 40% of people answered this question correctly. Case 26 The following vignette applies to the next 2 items

A 56-year-old African American male presents with several months history of a swallowing difficulty. He says, ‘It started with meat sticking in my throat and then got worse.’ He now has to drink a lot of fluids with his meals. He denies choking episodes, shortness of breath, voice change and heartburns, but notes that sometimes he coughs during meals. He has lost several pounds during the last two months. His past medical history is insignificant. He smokes 2 packs of cigarettes daily and consumes 6-8 bottles of beer on weekends. He is not sexually active. His blood pressure is 120/70 mmHg and heart rate is 80/min. His lungs are clear on auscultation. Neck palpation reveals no lymph node enlargement. Item 1 of 2 What is the best initial step in the management of this patient? A. Chest x-ray B. CT scan of the chest C. Barium swallow D. Upper GI Endoscopy E. Bronchoscopy Explanation: Dysphagia is an alarm symptom that warrants immediate evaluation. A thorough history and physical examination can provide important clues to the correct diagnosis and appropriate tests to be run. In this case, the progressive dysphagia to solids indicates a high probability of mechanical obstruction vs motility disorder. Considering this patient's risk factors (e.g., age, smoking, alcohol, weight loss), esophageal cancer is likely. In patients with suspected upper esophageal lesions, it is always safer to proceed first with barium swallow than with endoscopy. (Choice D) Although many physicians consider endoscopy as the initial test of choice, barium swallow can be run initially in certain patients. Intubation of the proximal esophagus during endoscopy is done relatively blindly, thereby risking perforation in patients with an upper esophageal pathology. (Choice B) CT scan of the chest can be a part of the work-up after the diagnosis is established. It is useful in assessing the extent of the disease and helps to choose an appropriate treatment strategy. (Choice E) Bronchoscopy is sometimes employed to detect tracheal and bronchial involvement in patients with upper esophageal lesions. (Choice A) A chest x-ray will convey little diagnostic information in this situation. Educational Objective: If an upper esophageal lesion is being suspected, it is always safer to proceed with barium swallow first before doing an endoscopy. 71% of people answered this question correctly. Item 2 of 2 Endoscopy reveals an upper esophageal mass with irregular contours that partially obstruct the lumen of the esophagus. What is the most likely histological type of the tumor? A. Mucoepidermoid B. Adenoid cystic C. Squamous cell D. Small cell E. Adenocarcinoma

Explanation: The two most common histological types of esophageal cancer are squamous cell carcinoma and adenocarcinoma. Several decades ago, squamous cell carcinoma was clearly predominant, accounting for up to 90% of esophageal cancer. Nowadays, the incidences of these two types are almost equal. The two differ in their pathophysiology and patient risk profile. Squamous cell carcinoma is more common in African Americans and shows a significant association with smoking, alcohol consumption, and some dietary factors. It usually affects the upper and middle parts of the esophagus. (Choice E) Adenocarcinoma is more common in Caucasians and usually arises from Barrett’s esophagus, a well-known metaplastic complication of GERD. It is associated with obesity, but shows little association with alcohol consumption. It typically affects the distal esophagus, gastroesophageal junction and gastric cardia region. (Choices A and B) Other histological types of esophageal cancer, including mucoepidermoid carcinoma and adenoid cystic carcinoma, are rare. Educational objective: Squamous cell carcinoma usually affects the upper and middle parts of the esophagus. It is more common in African Americans and shows a significant association with smoking, alcohol consumption, and some dietary factors. Adenocarcinoma is more common in Caucasians and usually arises from Barrett’s esophagus. 88% of people answered this question correctly.

Case 27 The following vignette applies to the next 2 items A 65-year-old male presents to the emergency department with an ulcer on his left foot. The lesion started as an erythematous patch on the medial aspect of his left forefoot three weeks ago. Two weeks ago, a small ulcer formed, enlarged, and eventually developed some purulent discharge. The amount of purulent discharge increased during the past week. He denies pain in his left foot. He complains of mild fever and chills. He has had type-2 diabetes mellitus for the past fourteen years. He is currently on a mixed split regimen of insulin, with suboptimal glycemic control. He has hypertension, for which he is on captopril and hydrochlorothiazide. His hypercholesterolemia is being managed with pravastatin. He has a 20 pack-year history of smoking. On physical examination, he is 6’0" (180 cm) tall, and weighs 152 lb. (71 kg). His blood pressure is 165/94 mm Hg, heart rate is 96/min regular, and temperature is 100 F (37.7 C). Neck examination reveals a left carotid bruit. Examination of the left foot revealed a 3x2-cm ulcer with a necrotic base on the medial aspect of the first metatarsophalangeal joint. There is some purulent foul-smelling discharge oozing from this ulcer. The surrounding skin is erythematous and warm. There is no significant tenderness in the left foot. Probing of the ulcer is suspicious for entering into the first metatarsal head. Peripheral pulsations are not felt in both feet. There is a significant sensory impairment up to the knees in both lower extremities. Ankle jerks are absent. Other systems are unremarkable. Item 1 of 2 Which of the following non-invasive modality has the highest accuracy for diagnosing osteomyelitis in his foot? A. MRI scan B. Technetium bone scan C. WBC scan D. Plain radiograph E. CT scan Explanation: The patient’s clinical features are highly suggestive of osteomyelitis. Probing into the bone is highly suggestive of osteomyelitis, which is generally confirmed using a non-invasive test. Of all the noninvasive tests, MRI is most accurate, with a sensitivity of 99% and specificity of 88%. The gold standard for diagnosing osteomyelitis is bone biopsy. It is important to diagnose osteomyelitis because the treatment may vary once the diagnosis is established. Majority of patients require long-term parental antibiotic treatment and resection of the affected bone. In this case, because there is a suspicion of significant ischemia, it might be prudent to consult a vascular surgeon, particularly if the response to antibiotic(s) and surgical debridement is poor. (Choices B, C, D and E) The specificity and sensitivity of other non-invasive tests is much lower. Educational objective: MRI is the most accurate test in diagnosing osteomyelitis in the vertebrae and in diabetic foot. 48% of people answered this question correctly. Item 2 of 2 Which of the following is the most useful to obtain a microbiologic diagnosis in this patient? A. Blood culture B. Culture swab from ulcer base C. Aspiration from surrounding erythematous skin D. Culturing pus coming out from ulcer E. Culture of deep tissue obtained by curettage

Explanation: Establishing a correct microbiological diagnosis will help in choosing the proper antibiotic(s) treatment. The majority of foot infections in diabetics are caused by mixed aerobic and anaerobic organisms (80%). The most commonly involved organisms are S. aureus, group B Streptococci, Proteus, Pseudomonas, E. coli, Candida, Bacteroides, Peptococcus and Clostridium. Organisms isolated from cultures of specimens from deep curettage correlate closely with cultures obtained from surgical resection of the deep tissue. Antimicrobial therapy for diabetic foot infections should be individualized. Most superficial skin infections require outpatient oral antimicrobial therapy against S. aureus, while extensive, deep infections require parental antibiotic therapy with a broad range for microbes, including anaerobes. Major surgical debridement may be necessary. (Choiced B, C, and D) A superficial swab from the ulcer, culture of the purulent discharge, and needle aspiration do not always reflect the correct etiologic organism. (Choice A) Blood cultures are positive in about 40% of patients with foot infection. Educational Objective: Organisms isolated from cultures of specimens from deep curettage correlate closely with cultures obtained from surgical resection of the deep tissue in patients with infected foot ulcers. 51% of people answered this question correctly. Case 28 A 75-year-old female is brought to the emergency room after she fell in her bathroom. She is unable to walk or bear weight on her right leg. Her past medical history is significant for hypertension, coronary artery disease, myocardial infarction, congestive heart failure and osteoporosis. On examination, you note that her right leg is externally rotated, and all the movements involving the right hip joint are very painful. The x-ray of the right hip shows a fracture in the neck of the femur. She is scheduled for right hip replacement. Which of the following is the most appropriate course of action to prevent deep venous thrombosis in this patient? A. Aspirin alone B. Warfarin alone C. Pressure stockings D. Low molecular weight heparin E. Inferior vena cava filter Explanation: Approximately 250,000 hip fractures occur every year in the United States. Development of venous thromboembolism in these patients is one of the major causes of postoperative morbidity and mortality. 4-7% of DVT patients suffer from fatal pulmonary embolism. Because of this, thromboembolic prophylaxis has been recommended for these patients. Studies comparing the prophylactic effects of aspirin, coumadin and unfractionated or low molecular weight heparin (LMWH) have shown that the LMWH group has superior results when compared with the other two groups. LMWH is therefore considered to be the therapy of choice, and should be given unless contraindications for its use are present. (Choice B) Coumadin is considered a second line prophylactic agent. It can be used as first line if any contraindication to the use of LMWH exists. Administration of coumadin should be started on the day of admission, and the target INR of 2 to 3 is achieved for adequate prophylaxis. (Choice C) Pressure stockings and intermittent pneumatic compression are used as adjuvants to other therapies. They are not sufficient to prevent DVT as sole agents because they are only effective in preventing DVT formation in calf veins, and have no prophylactic effect on pelvic vein thrombosis.

(Choice A) Aspirin is used as a third line agent after heparin and coumadin. It is recommended as a sole prophylactic agent against thromboembolism only if both heparin and coumadin (warfarin) are contraindicated. (Choice E) Inferior vena cava filter placement is indicated in patients who have contraindications to anti-coagulation. It is also used in patients who develop DVT despite ongoing anticoagulation treatment. Educational Objective: LMWH is considered to be the prophylactic therapy of choice for preventing deep vein thrombosis in patients at high risk. Case 29 The following vignette applies to the next 3 items It is estimated that the prevalence of smoking in a population is 50%. A cohort study conducted using a random sample from this population showed that the five-year risk of ischemic stroke is 1:1,000 in smokers and 0.5:1,000 in non-smokers. Item 1 of 3 What is the relative risk (RR) of ischemic stroke for smokers compared to non-smokers? A. RR = 1.0 B. RR = 1.5 C. RR = 2.0 D. RR = 2.5 E. RR = 3.0 Explanation: Relative risk represents a measure of outcome in follow-up studies. It is the risk ratio which compares the risk among the exposed to the risk among the unexposed. In this case, the five-year risk of the exposed (smokers) is 0.1% (1:1000), and the risk of the unexposed is 0.05 (0.5:1000); therefore, the relative risk is 0.1/0.05 = 2.0. The following interpretation is valid in this case: A five-year risk of stroke in smokers is twice that of non-smokers. Educational Objective: Relative risk is a measure of outcome in follow-up studies. It is the risk of the exposed, divided by the risk of the unexposed. 78% of people answered this question correctly. Item 2 of 3 What percentage of the strokes observed in smokers is attributed to their smoking status? A. 10% B. 25% C. 33% D. 50% E. 75% Explanation: In this scenario, you should calculate the attributable risk percent (ARP), which is also called etiologic fraction. ARP is a measure of excess risk. It estimates the proportion of the disease in exposed subjects that is attributed to exposure status. Two approaches can be used to calculate ARP. The first approach uses the following formula: ARP = (Risk in exposed – Risk in unexposed)/Risk in exposed = (0.1 – 0.05)/0.1 = 0.5 (50%). The other approach uses relative risk (RR) to calculate ARP:

ARP = (RR – 1)/RR = (2 – 1)/2 = 0.5 (50%). The following interpretation is valid in this case: 50% of the ischemic strokes observed in smokers can be attributed to their smoking status, and could therefore have been eliminated if they had not smoked. Educational Objective: ARP is a measure of excess risk. It estimates the proportion of the disease in exposed subjects that is attributed to exposure status. Item 3 of 3 What percentage of the strokes observed in the population is attributed to smoking? A. 10% B. 25% C. 33% D. 50% E. 75% Explanation: In this scenario, you should calculate the population attributable risk percent (PARP). PARP estimates the proportion of the disease in the population that is attributed to the exposure. Unlike attributable risk percent, PARP is the measure of excess risk in the total population, not only in exposed subjects. PARP can be calculated using the following approach: PARP = (Risk in the total population – Risk in unexposed)/Risk in the total population Knowing the risk of stroke in exposed (0.1%), the risk in unexposed (0.05%), and the prevalence of exposure in the population (0.5, or 50%), it is possible to calculate the risk in the total population: Risk in the total population = 0.1 x 0.5 + 0.05 x 0.5 = 0.075 The risk in the total population is 0.075%. Now we can calculate PARP: PARP = (0.075 – 0.05)/0.075 = 0.33 (33%) The following interpretation is valid in this case: 33% of the ischemic strokes observed in the population can be attributed to smoking. Educational Objective: PARP estimates the proportion of the disease in the population that is attributed to the exposure. Unlike attributable risk percent, PARP is the measure of excess risk in the total population, not only in the exposed subjects. Case 30 The following vignette applies to the next 3 items A 27-year-old woman with no medical history presents to your office complaining of recent onset chest pain. Her description of the chest pain is vague and nonspecific even when pressed for details, although she is quite dramatic in her presentation and at one point becomes teary-eyed. She is a very attractive woman dressed in revealing clothing and mentions that she is one of your neighbors. Physical examination, including vital signs, is unremarkable. Laboratory evaluation and an electrocardiogram are normal and suggest no etiology for her chest pain. You reassure her that the more serious causes of chest pain are highly unlikely, but she insists upon scheduling a follow-up appointment for next week. On the day of the second appointment, she arrives after your office is closed and intercepts you in the parking lot. She asks that you re-examine her now. Item 1 of 3 What is the most appropriate next step?

A. Refuse care and refer her to another physician B. Briefly examine her in the parking lot C. Perform full examination in the office D. Inform her that an examination is not possible now and suggest she return to the office tomorrow during business hours E. Send her to the emergency room by ambulance Explanation: Frequently encountered in the primary care setting, difficult patients tend to elicit strong reactions from their physicians. The task of properly diagnosing and managing the medical and psychiatric disorders of these patients is therefore quite challenging and invariably requires patience and insight on the behalf of the physician. When a patient behaves seductively or makes sexual overtures, the best response is to calmly establish definitive boundaries that allow for appropriate medical treatment while also maintaining a professional doctor-patient relationship. If a patient in stable condition insists upon being examined outside of the office, it is best to be pleasant but firm in reiterating that the patient must schedule an appointment to be seen during normal business hours (Choice D). Transferring this patient now (Choice A) certainly avoids the difficulties inherent in dealing with her, but is likely premature at this stage. Generally, termination of the doctor-patient relationship is done as a last resort in extreme cases (eg, with a verbally abusive or physically threatening patient) after all attempts to establish a successful relationship have failed and fair warning has been given. It is important to document the termination in writing and to ensure that the patient is not abandoned before she establishes care with a new physician. Performing a brief examination in the parking lot (Choice B) is highly inadvisable. It is important to maintain professional environs and boundaries with all patients, especially those who are behaving in an inappropriate or seductive manner. Performing a full examination in the office now (Choice C) despite the late hour is not ideal because it suggests a willingness to accommodate even unreasonable requests made by the patient. Since she is stable and failed to arrive at the appointed time, she should be advised it is necessary to return the next day. Sending her to the emergency department by ambulance (Choice E) is inappropriate since she is stable and her condition is not urgent. Educational Objective: When a patient behaves seductively or makes sexual overtures, the best response is to calmly establish definitive boundaries that allow for appropriate medical treatment while also maintaining a professional doctor-patient relationship. 91% of people answered this question correctly. Item 2 of 3 Appropriate action is taken. The patient responds by insisting that you comment on her appearance, as she is dressed in a sexually provocative manner. When you refuse, she becomes extremely upset. Which of the following conditions is she most likely suffering from? A. Antisocial personality disorder B. Borderline personality disorder C. Dependent personality disorder D. Histrionic personality disorder E. Narcissistic personality disorder Explanation:

According to the DSM-IV, patients with histrionic personality disorder (Choice D) display a persistent pattern of gratuitous emotionality and attention seeking as evidenced by five or more of the following criteria: 1) discomfort when not the center of attention; 2) behaves in a sexually seductive or provocative manner; 3) rapid shifting and shallowness of emotion; 4) usage of physical appearance to draw attention; 5) speech is vague and impressionistic; 6) dramatic, theatrical, and exaggerated behavior and emotion; 7) easily influenced by others; 8) exaggerates intimacy of relationships. Patients with antisocial personality disorder (Choice A) frequently violate the rights of others and the rules of society. Physical aggressiveness, lack of remorse, and consistent irresponsibility are also hallmark features. Patients with borderline personality disorder (Choice B) have unstable and intense interpersonal relationships, moods, and self-image. Impulse control is poor and suicidal behaviors are common. "Splitting" may be observed. Patients with dependent personality disorder (Choice C) crave being cared for by others and are often excessively clingy or submissive. Patients with narcissistic personality disorder (Choice E) are grandiose and desire adulation and admiration from other people. They may envy and exploit others, and are sometimes known to exaggerate their own accomplishments. Educational Objective: Histrionic personality disorder is characterized by gratuitous emotionality and attention seeking. Behaving in a dramatic, sexually provocative manner is common in patients with this personality disorder. 75% of people answered this question correctly. Item 3 of 3 As your neighbor, this patient is aware that you are single. She invites you to have dinner with her at a nearby restaurant later this week. You find her attractive. What is the most appropriate response? A. Accept her offer and document all personal interactions in her chart B. Accept her offer but do not document any personal interactions in her chart C. Refuse her offer and report the incident to her boyfriend, who is also your patient D. Refuse her offer and admit her to the acute-care psychiatric ward for stabilization E. Refuse her offer and inform her that you are prohibited from dating patients Explanation: It is the American Medical Association’s formal position that any sexual interaction occurring between a doctor and patient constitutes professional misconduct. Because of the inherent power structure of the doctor-patient relationship and the commonality of transference and counter-transference, sexual contact would allow for the potential exploitation of patients or the clouding of a physician’s judgment. It is expected that at the least, the physician will terminate the doctor-patient relationship prior to entering a sexual or romantic relationship with a patient. Many ethicists, however, argue that it is never appropriate to date a patient ‘even years’ after the doctor-patient relationship ended (Choice E). Whether or not any personal interactions are documented, it is always considered inappropriate to date a current patient (Choices A and B). Reporting the incident to the patient’s boyfriend (Choice C) would be a violation of the confidential nature of the doctor-patient relationship and is therefore inappropriate.

Admission to the psychiatric ward for stabilization (Choice D) is appropriate when the patient is at imminent risk of harming herself or others. Although she appears to be dysfunctional and to have some serious psychiatric issues, there is no evidence of imminent danger. Educational Objective: At the least, a physician should terminate the doctor-patient relationship prior to entering a sexual or romantic relationship with a patient. Many ethicists, however, argue that it is always inappropriate to date a patient “even years” after the doctor-patient relationship ended. 95% of people answered this question correctly. Case 31 An 84-year-old Caucasian woman is brought to the emergency department because of severe pain in her left hip and leg. She fell from the bed while sleeping. She has been bedridden for the past two years due to morbid obesity and advanced Alzheimer’s dementia. Her other medical problems include hypertension, hypercholesterolemia, diabetes mellitus type 2, and coronary artery disease. She lives at home with her granddaughter. Her medications include aspirin, hydrochlorothiazide, enalapril, donepezil, glimepiride and atorvastatin. Physical examination shows an obese, alert, mildly dehydrated elderly woman. Her temperature is 36.1 C (97 F), blood pressure is 170/85 mmHg, pulse is 100 /min and respirations are 18/min. The heart sounds are normal, and the lungs are clear. The abdomen is soft, non-tender and non-distended. Bowel sounds are normal. There is no rebound tenderness or rigidity. An ecchymotic area is present on the lateral aspect of the left thigh. Her left leg is rotated outwards, and looks shorter than the right one. Passive movement of the left lower extremity is extremely painful. An x-ray of the hip reveals a trochanteric fracture of the left femur. Her granddaughter is concerned about the fracture, and says that she is willing to sign consent forms so that her grandmother can undergo hip surgery to alleviate her pain. Which of the following is the most appropriate statement about the patient’s condition? A. If surgery is not done, the pain will persist B. Surgery will not offer benefits in her case and can be dangerous C. If surgery is not performed, she may become more disabled and need nursing home placement D. Because of her advanced age & dementia, surgery will be an unnecessary risk E. Instead of surgery, external traction will be preferred Explanation: The patient is not a good candidate for surgery. The type of surgery, the patient’s age, multiple comorbidities, and severe disability places her at an extremely high risk for perioperative complications. The ideal candidates who may benefit from a surgical intervention are elderly patients who were ambulatory prior to their hip fracture. Since the patient is already bedridden, performing the surgical procedure will make no difference. The possibility of a successful rehabilitation and recovery is difficult to predict or evaluate before surgery because this heavily relies on the surgical outcome and the patient’s response to the procedure. (Choice C) Although the decision to send the patient to a nursing home may be affected by her disability or need for surgical intervention, one of the main considerations is her current available social support. Since this patient’s granddaughter seems to provide ample social support, there is no need to send her to a nursing home. (Choice D) Advanced age and dementia should not preclude surgical intervention if the procedure offers the opportunity to improve or at least partially restore function and the quality of life of the patient. (Choices A and E) Several studies have evaluated and discussed the role of conservative therapy in patients who are unable to undergo hip surgery. These have shown that hip pain due to a fracture is not an indication for surgery because conservative measures can effectively control pain symptoms.

On the other hand, systematic reviews have concluded that other forms of conservative therapy such as external traction may cause more pain. Educational Objective: Even in cases where the risk is high, only the absence of benefits may prompt the physician to refuse a procedure; therefore, a physician must always look at the entire clinical picture and prognosis before suggesting or refusing a surgical intervention. The patient’s age, presence of comorbidities, functional status, quality of life, and personal values must all be considered to determine if the patient will benefit from hip surgery. If the patient is already bedridden, the benefit of surgery is minimal. If the patient has other serious medical problems, the risks may outweigh the benefits. Pain alone is not an indication for surgery, as conservative therapy can also effectively control this symptom. Case 32 A 7-year-old Caucasian boy is brought to the emergency room by his mother because of abdominal pain of acute onset. He had a mild, upper respiratory tract infection several days ago, but, otherwise, his past medical history is insignificant. His temperature is 36.7 C (98 F), blood pressure is 110/65 mmHg, pulse is 105/min., and respirations are 20/min. Physical examination reveals a symmetric, erythematous, macular rash on his lower extremities that progresses to papules after several hours. The abdomen is mildly tender on physical examination. Which of the following is the most likely diagnosis in this patient? A. Hemolytic uremic syndrome B. Thrombotic thrombocytopenic purpura C. Henoch-Schonlein purpura D. Acute intermittent porphyria E. Chickenpox Explanation: The clinical scenario described is highly suggestive of Henoch-Scholein purpura. An antecedent upper respiratory infection is present in 50% of patients. Abdominal pain is a presenting symptom in 1015% of patients. The skin lesions are symmetric, involve dependent parts of the body, and classically progress from an erythematous, macular rash to papular purpura. The joints and kidneys are also commonly involved. (Choice A) Hemolytic uremic syndrome typically develops in patients younger than two years old, and presents as abdominal pain and diarrhea (usually bloody) that progresses to acute renal failure. (Choice B) Thrombotic thrombocytopenic purpura (TTP) is a serious disorder characterized by the following classical pentad: 1. Severe thrombocytopenia 2. Microangiopathic hemolytic anemia (RBC fragments) 3. Fluctuating neurological signs 4. Renal failure 5. Fever Patients with TTP generally present with fever, pallor, petechiae, and confusion. The peripheral smear shows RBC fragments. PT and PTT are usually normal. The LDH is elevated due to hemolysis. Hemolytic uremic syndrome (HUS) and TTP comes under a spectrum of diseases. If a patient has more neurologic symptoms and less renal failure, the disease is considered TTP. On the other hand, if a patient has significant renal failure and less neurologic symptoms, the disease is considered HUS. Both TTP and HUS are very serious conditions, and require emergent plasmapheresis. (Choice D) Acute abdominal pain is common in patients with acute intermittent porphyria; however, it is unusual before 18 years of age, and skin lesions are absent.

(Choice E) Chickenpox may present as erythematous lesions early in the course of the disease; however, the rash appears on the scalp, face, trunk, and proximal limbs, and then progresses sequentially to form vesicles. Educational Objective: Classical clinical manifestations of Henoch-Scholein purpura include abdominal pain, arthralgias, skin lesions, and renal involvement. 85% of people answered this question correctly. Case 33 The following vignette applies to the next 2 items A 19-year-old man comes to the student health center, complaining of dysuria and a watery urethral discharge. He has had these symptoms for five days. He has no other medical problems. The patient remembers having sexual intercourse with a prostitute almost two weeks ago. He started smoking when he was 14, and has been smoking one pack of cigarettes daily ever since. He uses marijuana and "crack" almost daily, and drinks alcohol on weekends. He is unemployed and does not have any medications. Examination shows no abnormalities, except for a clear watery urethral discharge. A urethral swab is done and sent to the laboratory. Item 1 of 2 Which of the following is the most appropriate pharmacotherapy? A. Doxycycline B. Ceftriaxone C. Azithromycin D. Ofloxacin E. Metronidazole Explanation: The patient’s history and physical examination findings are highly suggestive of non-gonococcal urethritis (NGU). The incubation period is usually 5 to 10 days post-exposure, compared to 2 to 7 days for gonococcal infection. (According to the patient’s information, exposure may have occurred 9 to 10 days before the initiation of symptoms.) The urethral discharge is typically mucoid or watery in NGU, while in gonococcal urethritis, the secretion is purulent and abundant. Therapy is the same as for cervicitis in women: azithromycin or doxycycline. Azithromycin (1 g) is given as a single oral dose, and is preferred in this case to assure patient adherence. (Choice A) Doxycycline is given for 7 days, and the 100 mg tablets should be taken twice daily. The patient might have difficulties adhering to this type of therapy, which is therefore less preferred. (Choice B) Ceftriaxone is indicated for gonococcal urethritis. (Choice D) Ofloxacin has not been shown to be effective against Chlamydia trachomatis, Ureaplasma urealyticum or Mycoplasma genitalium, which are responsible for the majority of cases of NGU. Ofloxacin is effective against gonococcal infections. Educational Objective: NGU usually presents 5 to 10 days post-exposure, and is characterized by dysuria and a mucoid or watery urethral discharge. Therapy consists of a single dose of azithromycin or a 7-day course of doxycycline. Rates of success with either regimen are around 90%. Item 2 of 2 The patient returns ten days later with his elder brother. He states that he continues to have dysuria and some watery urethral discharge. His brother confirms that the patient has not been re-exposed to

any sexual partner, and that he took the medication as indicated. The results of the urethral swab are negative for Chlamydia and Neisseria gonorrhoeae. Which will be the most appropriate next step in management? A. Refer the patient to the urologist. B. Start metronidazole. C. Start doxycycline. D. Start ofloxacin. E. Start azithromycin. Explanation: The patient has non-gonococcal urethritis (NGU), which, in 90% of the cases, responds to therapy with azithromycin or doxycycline. If re-exposure and non-adherence are ruled out, the Center for Disease Control (CDC) guidelines do not recommend repeating the treatment with doxycycline or azithromycin (Choices C and E). Treatment is instead aimed at covering Trichomonas and resistant NGU pathogens. Metronidazole is given as a single dose (2 grams), followed by 7 days of erythromycin (500 mg every six hours). (Choice A) There are no current indications to refer the patient to the urologist. (Choice D) Ofloxacin will be effective for enteric bacteria and gonococcus, and not for Trichomonas, Chlamydia or Ureaplasma. Educational Objective: NGU that is refractory to antibiotic therapy must be treated with metronidazole and erythromycin, according to CDC recommendations. The objective is to treat Trichomonas and resistant NGU agents. Another alternative is erythromycin monotherapy, which requires high doses of the drug (800 mg six times a day). 36% of people answered this question correctly. Case34 The following vignette applies to the next 2 items A 26-year-old Caucasian female comes to see you in the office with complaints of fatigue, headache, generalized body aches, and low-grade fever for the past four days. She has just returned from a hiking and camping trip in Long Island, New York, two days ago. She saw a local physician there on the first day of her symptoms. He sent her for serologic testing for Rocky Mountain spotted fever, the results of which were inconclusive at that time. She goes to the same place in Long Island with her friends during spring break every year, and has never had any problems in the past. She is otherwise in good health, and does not take any medications regularly. On examination, her temperature is 37.8 C (100F), heart rate is 92/min, blood pressure is 122/74 mmHg, and respiratory rate is 16/min. There is evidence of a fine petechial rash over both wrists and the left ankle. The rest of her physical examination is unremarkable. Item 1 of 2 Which of the following is the most appropriate next step in her management? A. Repeat the serologic testing for Rocky Mountain spotted fever in one week. B. Perform Weil-Felix test now. C. Obtain a skin biopsy of the petechial lesions. D. Follow her platelet counts serially. E. Start the patient on treatment for Rocky Mountain spotted fever. Explanation: Rocky Mountain spotted fever is a tick-borne rickettsial infection caused by an intracellular gramnegative organism, Rickettsia rickettsii. It is seen throughout the United States, and has a major

prevalence in the southeastern and central states. It is usually seen in spring and early summer, when outdoor activities are at their peak. Most of the patients become symptomatic five to seven days after the tick bite. Early symptoms of the infection are nonspecific and can be misleading. Such symptoms include: low-grade fever, lethargy, myalgias and headaches. The rash of Rocky Mountain spotted fever is typically seen on the third to fifth day of illness. It is a petechial rash which usually begins on the ankles and wrists, and spreads to the palms, soles, and to the central body. In severe, fulminant cases, the patient may develop changes in mental status (i.e. confusion), focal neurological signs, seizures, and multiorgan dysfunction, leading to death. There is no single, reliable, diagnostic test during the early phase of the illness. The diagnosis and the decision to treat patients should be based clinically - on the patient’s presentation in the right setting (endemic area in spring or early summer). Patients should be started on treatment early, without waiting for confirmatory tests, since the delay in treatment is associated with a higher mortality rate. (Choice A) Although serologic testing with indirect fluorescent-antibody testing, enzyme immunoassay, or complement fixation test can be used to confirm the diagnosis of Rocky Mountain spotted fever, it is not useful during the first five days of illness because the antibodies are typically seen 7-10 days after the onset of illness. Repeating the serology in this patient after one week would delay the initiation of treatment, and worsen her prognosis. (Choice B) Weil-Felix test was used in the past to detect cross-reacting antibodies. It is a nonspecific test, and is not recommended for the diagnosis of Rocky Mountain spotted fever. (Choice C) Biopsy of the skin lesions for Rickettsia can confirm the diagnosis of Rocky Mountain spotted fever; however, skin lesions do not typically appear before the third to fifth day of illness. The results of biopsies also take some time; therefore, this can cause a potentially fatal delay in the management of the patient. (Choice D) Thrombocytopenia is one of the many fatal complications of this disease. The patient should be started on treatment for Rocky Mountain spotted fever once the diagnosis is suspected, based on her symptoms and epidemiology. Simply monitoring the platelet counts serially is not recommended. Educational Objective: Patients with suspected Rocky Mountain spotted fever should be treated empirically (without waiting for confirmation of the diagnosis). Item 2 of 2 Which of the following is the most appropriate treatment for patients diagnosed with Rocky Mountain spotted fever? A. Doxycycline B. Erythromycin C. Cephalexin D. Chloramphenicol E. Levofloxacin Explanation: Doxycycline is the treatment of choice in both children and adult patients with proven or suspected Rocky Mountain spotted fever. It is typically continued for at least three days after defervescence. Early initiation of therapy is indicated for all patients suspected of having Rocky Mountain spotted fever, and is based on the clinical picture and epidemiologic setting. Any delays in treatment can lead to potentially fatal complications and higher mortality rates. (Choice D) Chloramphenicol is an alternative option for the treatment of patients with suspected Rocky Mountain spotted fever. It has a higher incidence of side effects, and is only reserved for pregnant females and for patients who are unable to tolerate tetracycline.

(Choices B, C, and E) Erythromycin, cephalexin and levofloxacin are not recommended for the treatment of Rocky Mountain spotted fever. Educational Objective: Doxycycline is the treatment of choice for patients with Rocky Mountain spotted fever. Case 35 The following vignette applies to the next 2 items You are called to see a 32-year-old African American man with no past medical history who sustained a mid-shaft fracture of his left tibia during a motor vehicle accident. Open reduction and internal fixation were performed immediately after the patient arrived in the emergency department. The leg was stabilized in a plaster cast. Continuous morphine was provided for analgesic effect. At present, the patient complains of significant pain in his left calf. He appears very uncomfortable and is sweating and shifting restlessly in bed. He pleads for better analgesia. Pulses are present bilaterally in his lower extremities. Compartment syndrome seems likely. Item 1 of 2 Which of the following is an early sign of compartment syndrome in a limb? A. Erythema B. Homans’ sign C. Loss of deep tendon reflexes D. Pain with passive muscle motion E. Tissue ulceration and necrosis Explanation: The events most commonly responsible for compartment syndrome include crush injuries, fractures, prolonged external compression, burns, and snake bites. Such events cause either a decrease in compartment size or an increase in compartment pressure. As the intracompartmental pressure rises, the capillary blood perfusion is reduced until it can no longer maintain tissue viability. Early physical signs of acute compartment syndrome include tightness, weakness, and pain with passive muscle motion (Choice D). One of the most suggestive signs of compartment syndrome is pain out of proportion to the injury. Hypesthesia and paresthesia may be documented as well. Erythema (Choice A) is typically seen with venous congestion. Acute compartment syndrome is more commonly associated with cyanosis, pallor, or mottling of the extremity. Pain or increased resistance with dorsiflexion of the foot is known as Homans sign (Choice B). It is an unreliable indication of deep venous thrombosis. The loss of deep tendon reflexes (Choice C) can be seen in association with acute compartment syndrome, but is typically a later finding. Tissue ulceration and necrosis (Choice E) would be a very late finding of compartment syndrome. Educational Objective: Early physical signs of acute compartment syndrome include tightness, weakness, and pain with passive muscle motion. One of the most suggestive signs of compartment syndrome is pain out of proportion to the injury. Item 2 of 2 Which of the following is considered the most ominous sign of compartment syndrome? A. Paresthesia B. Loss of arterial pulse C. Mottling of the extremity

D. Pallor of the extremity E. Excruciating pain that persists after cast removal Explanation: The most ominous sign of compartment syndrome is the loss of the arterial pulse, as that signifies a cessation of blood flow to the extremity (Choice B). Muscle tissue has impairment of function after 2-4 hours of ischemia and irreversible loss of function after 4-12 hours, while nerve tissue has impairment of function after 30 minutes of ischemia and irreversible loss of function after 12-24 hours. Treatment includes splitting of the cast and underlying padding, which can decrease the compartment pressure by 50-85%. Paresthesia (Choice A) is an early finding and can be associated with arterial pulses that are diminished and not absent. It is therefore not considered the most ominous finding of compartment syndrome. Mottling (Choice C) and pallor (Choice D) of the extremity are physical findings occasionally associated with acute compartment syndrome. However, they can be seen in conjunction with arterial pulses that are diminished and not absent, and are therefore not considered the most ominous findings of compartment syndrome. Excruciating pain does sometimes persist after cast removal (Choice E) and is not always cause for increased concern. However, if any symptoms do not resolve within an hour after cast removal and the pressure measurement remains elevated, fasciotomy is indicated. Normal function will be regained in approximately two-thirds of patients if fasciotomy is performed within 12 hours of the onset of compartment syndrome. Educational Objective: The most ominous sign of compartment syndrome is the loss of the arterial pulse, as that signifies a cessation of blood flow to the extremity. Treatment includes splitting of the cast and underlying padding, which can decrease the compartment pressure by 50-85%. Case 36 A 36-year-old Caucasian woman comes to the emergency department and complains of severe left calf pain. While she was running on a racetrack, she heard a loud snap, after which she felt an "excruciating pain" in her left calf area. She was a marathon runner in the past, but she stopped running two years ago, after she had her first baby. She just started retraining a week ago for an upcoming race. The physical findings are highly suggestive of a complete Achilles tendon rupture. Which of the following is the best clinical sign that will support this diagnosis? A. Increased plantar flexion of the ankle on calf muscle compression B. Decreased plantar flexion of the ankle on calf muscle compression C. Increased dorsiflexion of the ankle on calf muscle compression D. Decreased dorsiflexion of the ankle on calf muscle compression E. No foot movement on calf muscle compression. Explanation: Rupture of the Achilles tendon may occur after abrupt calf muscle contraction. This typically occurs in men over the age of 40 who do not perform a regular leg-conditioning program. The most common symptoms are severe pain in the calf and the inability to stand up on the toes. The patient may note an audible snap at the time of injury. A positive Thompson test is further evidence of an Achilles tendon rupture. This test is performed with the patient kneeling on a chair, or lying prone on an examination table with his feet hanging over the edge. When the examiner squeezes the calf muscle on the normal side, the foot responds with plantar flexion. On the affected side, there is no foot response. Treatment consists of immediate immobilization of the lower leg and surgical repair of the tendon as soon as possible.

Educational Objective: A complete Achilles tendon rupture leads to severe pain in the calf and the inability to stand up on the toes. A positive Thompson test further supports the diagnosis.

Case 37 A female infant born at term to a 28-year-old Caucasian woman has dorsal feet and hands edema, short webbed neck, and a cardiac murmur. Buccal smear reveals no Barr body. The mother is concerned about the recurrence risk of such an anomaly in subsequent pregnancies. Which of the following is the best response? A. The recurrence risk is approximately 50% B. The risk increases with maternal age C. The recurrence risk is close to 25% D. The recurrence risk is close to 10% E. The recurrence risk is close to that of the general population Explanation: The clinical scenario described (dorsal feet and hands edema, short webbed neck, and a cardiac murmur) is typical for Turner syndrome. It is characterized by monosomy of the X chromosome (45, X); this explains why there is no Barr body on the buccal smear. Less common chromosomal abnormalities that can be present in patients with Turner syndrome include X chromosome mosaicism and Xp deletion. No increased recurrence risk is present after having an infant with Turner syndrome. (Choice B) Interestingly, the risk of having an infant with monosomy for the X chromosome does not increase with advance maternal age, unlike Down’s syndrome and Klinefelter’s syndrome (47, XXY). Educational Objective: No increased recurrence risk is present after having an infant with Turner syndrome. The risk of 45, X does not increase with increased maternal age. Case 38 A 20-year-old Caucasian woman comes to see you in the office for a regular follow-up visit. She has had three heterosexual partners in the past four years. Her last Pap smear two years ago was negative. She informs you that she is a lesbian now, and does not have any heterosexual partners. She wants to discontinue getting regular Pap smears now that she is not having sexual intercourse with men. Which of the following is the most appropriate response? A. I agree with you, there is no need for further Pap smears. B. You should still get a Pap smear every now and then. C. You should get a regular Pap smear if your other lesbian partner is HPV (human papilloma virus) positive. D. You should still get regular Pap smears every year. E. You should have a Pap smear done every 3 - 5 years. Explanation: Lesbian women often conceal their sexual orientation from their primary care physicians. With more recent developments and legislations regarding same sex marriages, more homosexual couples are now coming forward with their sexual orientation. It is important for a primary care physician to be knowledgeable and sensitive to their special needs and concerns. Human papilloma virus (HPV) infection has been strongly linked with the development of cervical intraepithelial neoplasia (CIN) and invasive cervical cancer. Sexual intercourse, especially with multiple partners, is the main risk factor for the acquisition of HPV infection. Lesbian women still need

to undergo routine screening for cervical cancer. Annual screening by Papanicolaou smear is recommended for all women approximately three years after the onset of vaginal intercourse, or at the age of 21, whichever is earlier. The risk of acquiring HPV infection (and CIN/cervical cancer) is lower in lesbian women if they do not engage in sexual intercourse with men. The risk of cervical neoplasia is highest in lesbian women who have had sex with more than one male sexual partner, have an early age at first coitus with men, have been infected with HPV, and have been treated for an abnormal cervical cytology test in the past. The risk is also increased with cigarette smoking. (Choices A, B and E) Increasing the screening interval or not screening lesbian women at all can lead to the delayed diagnosis and treatment of high-grade cervical intraepithelial neoplasia and cervical cancer. (Choice C) The patient should have an annual Pap smear even if her partner is HPV negative. Her partner’s HPV status does not change her risk of cervical cancer due to her previous sexual history. Educational Objective: Annual screening by Papanicolaou smear is recommended for all women (including lesbian women) approximately three years after the onset of vaginal intercourse, or at the age of 21, whichever is earlier. 64% of people answered this question correctly. Case 39 The following vignette applies to the next 3 items A previously healthy 62-year-old African-American woman is hospitalized because of shortness of breath and chest pain. She has had these symptoms for the past two days. She has no other medical problems, and takes no medications. She has smoked one pack of cigarettes daily for the last 20 years. Her temperature is 36.7 C (98 F), blood pressure is 130/80 mmHg, pulse is 98/min., and respirations are 22/min. The patient's pulse oximetry showed 94% on 4-liters of oxygen. V/Q scan of the chest reveals a high probability for pulmonary embolism. Her baseline labs reveal the following: CBC Hb 13 g/dL Hct 38% Platelet count 240,000/cmm Leukocyte count 8,000/cmm Coags Prothrombin time 14 sec INR 1.06 You start her on anticoagulation with unfractionated heparin and warfarin. Her symptoms gradually resolve over the next five days. On day six of her hospitalization, she complains of pain and pallor in her left arm. Physical examination reveals a pale and tender distal left arm with diminished pulses. The patient’s labs reveal: CBC Hb 12.6 g/dL Hct 37.2% Platelet count 40,000/cmm Leukocyte count 10,000/cmm Coagulation Tests Prothrombin time 19 sec INR 1.78 PTT 60 sec

Item 1 of 3 Which of the following is the most likely cause of her condition? A. Subtherapeutic anticoagulation B. Heparin-induced thrombocytopenia C. Warfarin induced skin necrosis D. Heparin-induced skin necrosis E. Warfarin-induced thrombocytopenia Explanation: This patient had an arterial thrombosis of the left arm following the administration of unfractionated heparin. She has developed heparin-induced thrombocytopenia (HIT), which is a well-known complication of heparin therapy. Two forms of HIT have been recognized, depending upon the onset, clinical course, and severity of the disease. Type I HIT is seen within two days of initiation of heparin therapy, and is usually associated with a lesser degree of fall in the platelet count (nadir platelet count of 100,000/microliter). The platelet count usually returns to normal with discontinuation of heparin, and there are no clinical consequences. Type II HIT is a more serious immune-mediated disorder characterized by the formation of antibodies against heparin-platelet factor 4 complex. The heparin-platelet factor 4 antibody complex then binds to the platelet surface, causing platelet activation and aggregation, leading to thrombocytopenia and platelet-rich clots. It typically develops 4 to 10 days after the initiation of heparin therapy. Patients have the platelet count in the range of 30,000 to 60,000/microliter. Spontaneous bleeding is unusual. Immune-mediated HIT is associated with both venous and arterial thrombosis. The major manifestations of venous thrombosis are deep venous thrombosis, pulmonary embolism, venous limb gangrene, and cerebral sinus thrombosis. Arterial thrombosis can lead to strokes, myocardial infarction, and limb and organ (kidneys, mesenteric) ischemia. (Choice C) Warfarin-induced skin necrosis has been reported in patients within the first few days of taking high doses of warfarin. High-dose warfarin induces a transient hypercoagulable state by causing a rapid reduction in protein C levels on the first day of therapy. The lesions are seen over the extremities, trunk, and breasts. It is not associated with thrombocytopenia. (Choice D) Heparin-induced skin necrosis is also a well-known complication of unfractionated heparin use. It usually involves areas rich in fat, such as the abdomen, although distal extremities can also be involved. It presents as an area of erythema, which quickly progresses to purpura, hemorrhage, and necrosis. Most of these patients do not develop thrombocytopenia. (Choice A) Even though his PT/INR is subtherapeutic, his PTT is therapeutic. This patient is well anticoagulated.

means that the

Educational Objective: HIT should be suspected in patients receiving heparin anticoagulation if they present with thrombocytopenia, thrombosis with thrombocytopenia, or a > 50% fall in the platelet count, 4- 10 days after the initiation of treatment. 74% of people answered this question correctly. Item 2 of 3 Which of the following is the most appropriate next step in the management of this patient? A. Discontinue warfarin

B. Discontinue unfractionated heparin C. Discontinue warfarin and unfractionated heparin D. Discontinue unfractionated heparin and warfarin; initiate low molecular weight heparin E. Discontinue heparin and warfarin; initiate argatroban Explanation: The first and the most important intervention in a patient with suspected or documented HIT is the immediate cessation of exposure to all heparin products. The patients with HIT still remain at risk for thrombosis, even after the discontinuation of heparin. All such patients should be anticoagulated with direct thrombin inhibitors such as lepirudin or argatroban. They should be used for the prophylaxis and treatment of patients with HIT with or without thrombosis. The choice of agent depends on the coexisting medical conditions. (Lepirudin should be used with caution in patients with renal insufficiency, while the dose of argatroban should be adjusted in patients with hepatic dysfunction.) (Choice B) Warfarin therapy alone may increase the risk of venous gangrene in patients with deep vein thrombosis. Its use should be avoided in the absence of other anticoagulants until the platelet count rises above 100,000/microliter. (Choice D) Low molecular weight heparin should not be substituted for unfractionated heparin because it can also cross react with heparin-induced antibodies, and induce more antibody formation. Educational Objective: Exposure to both heparin and warfarin should be discontinued in patients with HIT. They should be anticoagulated with direct thrombin inhibitors for the prevention or treatment of thromboembolic complications. Item 3 of 3 Which of the following would have been the most useful strategy in preventing the above condition? A. Monitoring prothrombin time frequently B. Monitoring platelet counts frequently C. Substitution of unfractionated heparin with low molecular weight heparin D. Initiating warfarin after 3 – 5 days of heparin therapy E. Using higher doses of heparin early in the course of treatment Explanation: The best way to prevent HIT is to use low molecular weight heparin (LMWH) instead of unfractionated heparin whenever possible. LMWH and heparinoids, such as danaparoid, are associated with a much lower incidence of HIT compared with unfractionated heparin. Another option is to limit heparin use to less than five days in order to prevent an antibody response, as well as the development of HIT. (Choice A) Prothrombin time is used to monitor the effects and adjust the dose of warfarin, not heparin. It does not affect the development of HIT. (Choice B) Monitoring the platelet counts will lead to early detection and treatment of HIT. It will not prevent the formation of heparin-platelet factor 4 antibodies and their clinical consequences. (Choice D) Initiating warfarin early can minimize the duration of heparin use, and prevent antibody formation. It should be started at the same time as (or within 24 hours of) heparin administration. (Choices E and F) Development of HIT is independent of the dose used, and has been reported to occur even with heparin flushes and the use of heparin-coated catheters. Educational Objective:

The best way to prevent heparin-induced thrombocytopenia is by substituting unfractionated heparin with low molecular weight heparin. Case 40 The following vignette applies to the next 3 items A 29-year-old male comes to the emergency department (ED) because of palpitations and a severe headache. By the time you see him, he is more or less asymptomatic. He has had at least two similar episodes for the past one and a half months. He is requesting for "a presciption for some pills to prevent the recurrence of the symptoms." He currently has no medications. Physical examination reveals a thin young male who appears anxious and diaphoretic. His blood pressure is 126/84 mmHg, pulse is 86/min, temperature is 36.7C (98F) and respirations are 16/min. During his prior visit to the ED for an identical episode, his blood pressure was 149/98 mmHg. The thyroid is normal to palpation and without any obvious nodules. Item 1 of 3 Which of the following is the most appropriate next step? A. Measure blood pressure after two weeks in the office B. Start diuretics C. A 24-hour urinary metanephrine and free catecholamines D. MRI of the abdomen E. Alpha blockade Explanation: The patient has clinical features suggestive of pheochromocytoma. Biochemical confirmation of the diagnosis is required before imaging is performed to localize the tumor. Measurement of urinary metanephrine and catecholamine levels is a good screening test that is used for biochemical diagnosis. Both tests are ordered simultaneously. A 24-hour urine collection is superior to spot urinary collection; multiple 24-hour urine collections may be required to make a diagnosis of pheochromocytoma. Urinary vanillylmandelic acid (VMA) can also be used with other tests as a screening test. VMA has low sensitivity but higher specificity. Urine levels can be altered by a number of drugs and foods. For this reason, some centers use measurement of plasma free metanephrine levels as the initial screening test. (Choice A) This patient has features that are highly suggestive of pheochromocytoma. Investigations should be started without further delay. (Choice B) Diuretics are not good initial therapeutic agents for controlling the hypertension of patients with pheochromocytoma. Many of these patients have a lower intravascular volume, which can be further aggravated by giving diuretics. (Choice D) MRI/CT is performed to localize the tumor only after biochemical confirmation of pheochromocytoma has been achieved. (Choice E) Treatment with alpha-blockers is not started before the biochemical diagnosis of pheochromocytoma is confirmed because these increase serum catecholamine and metanephrine levels. If these drugs are immediately started, it would be extremely difficult to interpret some of the biochemical test results. Educational Objective: A biochemical diagnosis of pheochromocytoma is typically made by measurement of plasma free metanephrine levels or a 24-hour urine collection for measurement of catecholamine, metanephrine, and vanillylmandelic acid levels. Confirmation of the diagnosis with these biochemical tests is required

before imaging is performed for tumor localization. A number of drugs (e.g., alpha-blockers) can interfere with the biochemical test results. *Extremely important question for USMLE step-3 Item 2 of 3 The diagnosis is confirmed. What is the next best step in the management of this patient? A. CT/MRI of adrenals B. Alpha-blocking agent C. Beta-blocking agent D. Removal of tumor E. Metaiodobenzylguanidine (MIBG) scan Explanation: Alpha blockade is started after biochemical confirmation of the diagnosis of the pheochromocytoma is made. Alpha blockade, along with liberal salt and fluid intake, is required preoperatively to restore the intravascular volume. While the patient is started on a preoperative surgical regimen, investigations to localize his tumor can be performed. The most common agent used for preoperative preparation is the long-acting noncompetitive alpha-blocker phenoxybenzamine. Majority of patients require 40-80 mg of phenoxybenzamine daily. Patients should be treated for 10-14 days preoperatively before surgical resection can be performed. Adequate alpha blockade preoperatively reduces intraoperative complications. (Choice A) CT/MRI is typically done while the patient is on alpha blockade. CT and MRI have equal sensitivities for tumor localization, although MRI can be more sensitive in localizing extraadrenal pheochromocytoma. In addition, MRI can sometimes differentiate benign from malignant pheochromocytoma. (Choice C) Beta-blocking agents started before alpha blockade can lead to paradoxical increase in the blood pressure. Beta blockade is only preformed after alpha blockade is complete.

(Choice D) Removal of the tumor (after localization) is performed after adequate alpha blockade. The chances of intraoperative complications are much higher with inadequate preoperative alpha blockade. Approximately 10% of patients have bilateral adrenal pheochromocytomas, and 10% are extraadrenal in location. (Choice E) MIBG resembles norepinephrine and is taken up by pheochromocytomas. The sensitivity of MIBG scan is 70%. MIBG scan is performed in patients who have borderline biochemical values, but with the CT showing an adrenal mass. MIBG can also be performed in patients with biochemically confirmed pheochromocytoma, but the CT or MRI are unable to localize the tumor. Educational Objective: Alpha blockade is done for 10-14 days preoperatively to control hypertension and restore intravascular volume. Beta-blockers are only given to patients who are adequately alpha blocked. *Extremely important question for USMLE step-3 Item 3 of 3 The appropriate steps are taken, and the patient is taken for surgical removal of the tumor. During the procedure, he rapidly becomes hypotensive. His blood pressure falls from 110/89 mmHg to 80/50 mmHg. Which of the following is the most appropriate therapy for this patient’s hypotension?

A. Bolus of normal saline followed by continuous normal saline infusion B. Intravenous colloid bolus C. Intravenous phentolamine bolus D. Dopamine infusion E. Dobutamine infusion Explanation: Even with adequate preoperative alpha blockade, some patients with pheochromocytoma have intraoperative complications. One of the important intraoperative complications is hypotension. This generally occurs after the removal of the tumor, which is followed by a decrease in circulatory catecholamine levels and alpha blockade, thereby leading to a marked decrease in the vascular tone. The resulting intraoperative hypotension responds nicely to a normal saline bolus followed by infusion. (Choice B) Intravenous colloid may have theoretical benefits compared to crystalloid normal saline. Possible benefits of colloid are rapid restoration of intravascular volume and decreased chances of pulmonary edema. Studies have not shown that colloids are superior to the use of crystalloid solution in the management of acute hypotension. Finally, colloids are more expensive than normal saline. (Choice C) Intravenous phentolamine is used for acute hypertension intraoperatively. (Choices D & E) Dopamine and dobutamine are less effective in patients with pheochromocytoma and intraoperative hypotension because majority of these patients are on preoperative long-acting alpha blockade; therefore, the vascular response to vasoconstrictors is reduced. Educational Objective: Intraoperative hypotension in patients with pheochromocytoma responds to an intravenous bolus of normal saline. The use of colloid is not superior to the use of normal saline bolus for the management of intraoperative hypotension. Vasopressors are less effective than a bolus of normal saline. Intravenous bolus of phentolamine is used for acute severe hypertension (not hypotension) during surgery for pheochromocytoma. *Extremely important question for USMLE step-3 Case 41 Several tests have been developed to measure the serologic markers of breast cancer. These tests have different specificities and sensitivities for the early stage of breast cancer. If positive, which of the following tests will have the highest predictive value for the disease? A. Sensitivity B. Sensitivity C. Sensitivity D. Sensitivity E. Sensitivity

80%, specificity 65%, specificity 70%, specificity 75%, specificity 85%, specificity

90% 97% 94% 92% 90%

Explanation: A high specificity increases the positive predictive value (PPV) of the test: PPV = True Positives / (True positives + False positives) Confirmatory tests must have a high specificity. In this case, the test with the highest specificity is the best choice, since this high specificity decreases the number of false-positive results and helps ‘RULE OUT’ the disease. (Choice E) Increasing the sensitivity of the test will increase the negative predictive value. Educational Objective:

Confirmatory tests must have a high specificity. This high specificity helps ‘RULE OUT’ the disease by decreasing the number of false-positive results, and by increasing the positive predictive value. 35% of people answered this question correctly. Case 42 A 70-year-old widowed African-American woman is admitted late one night to the hospital for an acute exacerbation of her congestive heart failure. During morning rounds, the medical team responsible for this woman’s care meets with her for the first time. The senior resident begins to ask the patient some questions about her medical history and addresses her by first name. What is the appropriate response on the part of the attending physician? A. Address the patient by first name as well B. Address the patient with the salutation ‘Madam’ followed by her surname C. Address the patient with the salutation ‘Mrs.’ followed by her surname D. Address the patient with the salutation ‘Ms.’ followed by her surname E. Reprimand the resident and send him from the room Explanation: New patients should always be addressed as "Ms. Smith" or "Mr. Smith," as that properly demonstrates the respect due the patient (Choice D). Keep in mind as well that older patients or patients with a different cultural heritage (such as the woman in this question) are often very sensitive to being spoken to in a manner that they perceive as disrespectful. The doctor-patient relationship may needlessly suffer if the doctor speaks informally without direct permission to do so. There is some debate regarding whether a patient should ever be called by first name, even if they so request it. Specifically, some ethicists insist that the level of formality in address must be applied mutually; therefore, if a patient is called by her first name, then the physician too must be called by his first name. Ultimately, it is best to err on the side of formality rather than informality. Addressing the patient by the first name (Choice A) is presumptive and risks alienating the patient. The salutation of "Madam" (Choice B) is typically used for individuals of high standing (e.g., "Madam President" or "Madam Ambassador"). Using this form of address with all patients may be interpreted as sarcasm. The salutation of "Mrs." (Choice C) should be used if the patient has indicated it to be her preference. Keep in mind that some women do not want reference made to their marital status, which is why the salutation "Ms." is more often used. Openly reprimanding the resident (Choice E) is excessively harsh if the resident otherwise appeared to have good intentions in speaking with the patient. It also creates an awkward situation for the patient, who may feel obligated to defend the resident. If an attending wants to discuss the matter of proper salutation with the resident, it is best to do so outside of the patient’s presence. Educational Objective: New patients should be addressed as "Ms. Smith" or "Mr. Smith" and not by first name. 45% of people answered this question correctly. Case 43 The following vignette applies to the next 2 items A 55-year-old Caucasian woman is admitted to the hospital with symptoms of left leg pain and swelling of two weeks duration. She denies any chest pain or difficulty in breathing. She has a past medical history of hypothyroidism and hypertension. Her daily medications include L-thyroxine and hydrochlorothiazide. In addition, she has been on hormone replacement therapy (HRT) with estrogenprogestin combination for the last two years for intractable postmenopausal symptoms. She smokes a

pack of cigarettes a day. On physical examination, she has pitting edema and warmth over the left lower leg. Lower extremities Doppler reveal the presence of left proximal femoral deep vein thrombosis. Item 1 of 2 Which of the following is the most appropriate next step in the management of this patient? A. Discontinue hormone replacement therapy immediately B. Start her on warfarin 5 mg a day C. Start her on warfarin 10 mg a day D. Start her on weight-based unfractionated heparin E. Repeat the ultrasound in a week Explanation: The patient has developed a deep vein thrombosis (DVT) of the left femoral vein. Anticoagulation is indicated for all patients with symptomatic proximal DVT. The primary goal of therapy is to prevent clot extension and prevent acute pulmonary embolism. Patients with DVT or pulmonary embolism should be treated with unfractionated intravenous heparin or low molecular weight (LMW) heparin. Therapy with heparin should be continued for at least five days. Oral anticoagulation with warfarin can be started simultaneously, and should be overlapped for at least four to five days. Treatment with intravenous unfractionated or LMW heparin should be continued until the INR (International normalized ratio) has been therapeutic (2.0 - 3.0) for at least two days. LMW heparin is generally recommended over unfractionated heparin. (Choice A) HRT has been associated with an increased risk of venous thromboembolism (VTE), especially in the first year of use. Observational studies, the HERS trial, and the Women's health initiative have suggested a two-fold increase in the risk of VTE with HRT use. HRT should therefore be discontinued in patients with an increased risk of DVT or a documented DVT; however, HRT should be tapered gradually because an abrupt discontinuation can precipitate a flare of postmenopausal symptoms. (Choices B and C) Warfarin causes a rapid reduction in the protein C levels on the first day of therapy. This induces a transient hypercoagulable state, which can rarely lead to warfarin-induced skin necrosis. Furthermore, it takes time to have a therapeutic effect. For these reasons, warfarin should always be started after the patient has been under the cover of unfractionated or LMW heparin. (Choice E) DVT documented by an initial ultrasound should be treated with full anticoagulation. There is no need to repeat the ultrasound. (Choice F) The patient has no clinical features of an acute pulmonary embolization Educational Objective: Anticoagulation with heparin followed by warfarin is indicated in all patients with symptomatic deep vein thrombosis. Item 2 of 2 You manage the patient appropriately, and she is ready for discharge on the fifth hospital day. You counsel her regarding the side effects of continuing hormone replacement therapy and the duration of anticoagulation. She wishes to continue taking HRT as it makes her ‘look and feel good.’ Which of the following is the most appropriate response? A. She can continue HRT without any additional risks, as long as she promises to quit smoking B. She can continue HRT, if she takes warfarin for at least six months C. She should stop HRT and continue warfarin for at least three months D. She can continue HRT, if she agrees to daily subcutaneous, low-molecular eight heparin therapy E. She should stop HRT and switch to tamoxifen for hot flashes

Explanation: It is important to consider the presence or absence of risk factors before deciding on the duration of anticoagulation therapy. There are guidelines based on the evidence-based recommendations of the sixth ACCP Consensus Conference on Antithrombotic therapy. According to these guidelines, patients with a first thromboembolic event in the presence of a reversible risk factor should be treated with anticoagulation for at least three months. In contrast, patients with an idiopathic first event, continuing risk factor, or a recurrent thromboembolic event should be treated for a prolonged period of time. (Choices A, B, D, and F) All patients on HRT with a documented DVT should be strongly urged to stop hormone replacement therapy. It is not recommended to increase the duration of anticoagulation or to switch to LMW heparin in order to facilitate the continuing use of HRT. Smoking cessation alone will not prevent the future episodes of thromboembolism. Estrogen-only preparations carry the same risk of causing venous thromboembolism. In contrast, these may increase the risk of endometrial hyperplasia and cancer due to the loss of the protective effects of progesterone. (Choice E) Tamoxifen has no role in the prevention of hot flashes in postmenopausal women, and should not be recommended. Educational Objective: A first episode of thromboembolism in the presence of reversible or time-limited risk factors should be treated for at least three months to prevent recurrence. Case 44 A 38-year-old Caucasian woman is brought to the emergency department by her coworkers, who noticed that she had been "breathing heavily" and had been restless for the last couple of hours, when she started to become drowsy. She complains of nausea, vomiting, dizziness and tinnitus; these have been present for the past few hours. She denies shortness of breath, cough or chest pain. Her other medical problems include rheumatoid arthritis and depression. She has smoked one pack of cigarettes daily for the past twenty years. She uses marijuana occasionally and drinks one glass of beer daily. Her medications include aspirin, fluoxetine and low-dose prednisone. She has no known drug allergies. Her temperature is 38 C (100.4 F), blood pressure is 120/80 mm Hg, pulse is 100/min, respirations are 26/min, and oxygen saturation is 99% at room air. Examination reveals an awake, but mildly drowsy young woman with dry mucus membranes. Heart sounds are normal. Lungs are clear. The abdomen is soft, non-tender, and non-distended; bowel sounds are present. There is no rebound tenderness or rigidity. No motor or sensory focal deficit is present. Meningeal signs are absent. A 12lead electrocardiogram (EKG) shows sinus tachycardia. A portable chest x-ray reveals no abnormalities: the lung fields are clear; there is no cardiomegaly or effusions. The patient is given intravenous fluids and 1-liter oxygen per nasal cannula. She is placed on continuous EKG monitoring. Laboratory tests are ordered. Item 1 of 2 Which of the following is the most likely cause of her condition? A. Anxiety B. Diastolic heart failure C. Pulmonary embolism D. Drug overdose E. Acute bacterial pneumonia Explanation: The patient’s symptoms, along with her history of illicit drug intake and aspirin use, highly suggest an overdose of aspirin or another drug that causes metabolic acidosis. The initial manifestations of salicylate intoxication are tinnitus, restlessness, nausea, vomiting, and mild gastrointestinal discomfort. After a few hours, a depressed level of consciousness will ensue. Fever, metabolic acidosis, and hyperventilation eventually develop. Hyperventilation without dyspnea is suggestive of

respiratory alkalosis, which may be a compensatory response to the metabolic acidosis. Aside from tachycardia and tachypnea, there are no other indications of cardiac or pulmonary compromise. (Choice B) Although diastolic dysfunction can be suspected in a patient with signs and symptoms of heart failure with a normal ejection fraction, there is no clinical evidence of heart failure in this case. (Choice C) Pulmonary embolism can present with sinus tachycardia, normal chest x-ray and mild fever; however, this diagnosis cannot fully explain the patient’s other presenting characteristics (i.e., oxygen saturation is 99%, drowsy state without being hypoxic, nausea, vomiting and tinnitus). (Choice A) The presence of fever, drowsiness, tachycardia and tachypnea, as well as the patient’s denial of dyspnea, points to an organic cause of her disease. (Choice E) The patient does not have a history of cough, dyspnea or sputum production. The chest xray is clear, and her oxygen desaturation is 99%. For these reasons, the possibility of pneumonia is very unlikely. Educational Objective: Dyspnea is the subjective sensation of labored or difficult breathing. Athletes and well-trained individuals develop dyspnea only when doing hard work or exercise, while sedentary subjects may experience it even after minor efforts. Hyperventilation associated with metabolic acidosis is hardly ever accompanied by dyspnea. Metabolic acidosis must be suspected as the cause of hyperventilation (compensatory respiratory alkalosis) without dyspnea if there is no evidence of cardiac or pulmonary disease, the patient is a potential drug abuser, and there are signs and symptoms compatible with compromise of other organs. Item 2 of 2 The patient continues to be drowsy, but she is now easily arousable. Her temperature is 37.8 C (100 F), blood pressure is 120/70 mm Hg, pulse is 104/min, respirations are 28/min, and oxygen saturation is 100% with one-liter nasal cannula. Her laboratory tests reveal the following: CBC Hb 13.8 g/dL Ht 41% MCV 91fl Platelet count 320,000/cmm Leukocyte count 6,000/cmm Segmented neutrophils 70% Bands 2% Eosinophils 2% Lymphocytes 18% Monocytes 8% Serum chemistry Serum Na 145 mEq/L Chloride 108 mEq/L BUN 30 mg/dL Calcium 9.9 mg/dL

Serum K 4.3 mEq/L Bicarbonate 17 mEq/L Serum creatinine 1.6 mg/dL Blood glucose 75 mg/dL

LFT Total bilirubin 1.3 mg/dL Alkaline phosphatase 200 U/L Alanine aminotransferase 110 U/L Gases, arterial blood pH 7.25 pO2 110 mm Hg

Direct bilirubin 0.8 mg/dL Aspartate aminotransferase 150 U/L

pCO2 20 mm Hg

Which of the following is the most appropriate next step in management?

A. Intravenous antibiotics B. Oral lactulose and metronidazole C. Intravenous furosemide D. Non invasive mechanical ventilation E. Gastric lavage and alkalinization of the urine Explanation: The patient has moderate metabolic acidosis, with a pH of 7.25, a bicarbonate level of 17 mEq/L and an anion gap of 20 (obtained by subtracting the values of bicarbonate and chloride from the sodium). Her anion gap is high (normal anion gap: 12, ranging from 8-16). Her slightly elevated BUN and creatinine levels indicate mild renal failure, which can be due to dehydration or acute interstitial inflammation. Other abnormal findings are the moderately elevated aminotransferases, while the white blood cell count is normal. The clinical vignette is consistent with aspirin overdose, which is associated with the development of acute renal failure and transient hepatotoxicity, which can lead to coagulopathy, severe encephalopathy and non-cardiogenic pulmonary edema. Therapy consists of gastric lavage, administration of activated charcoal, and alkalinization of the urine to enhance secretion. (Choice C) The patient is dehydrated. Giving furosemide will aggravate her dehydrated status and renal failure. (Choice B) The encephalopathy in this case is due to intoxication and metabolic acidosis, not hepatic disease. Lactulose and metronidazole will not be useful here. (Choice D) Non-invasive mechanical ventilation is not needed as the patient does not have pCO2 retention and is not in ventilatory failure. (Choice A) Even though the patient is febrile, antibiotics are not needed because the cause of her hyperthermia is drug overdose, and there is no evidence of infection. Aspirin causes uncoupling of the oxidative phosphorylation, resulting in hyperthermia. Educational Objective: Salicylate overdose presents with high anion gap metabolic acidosis, compensatory respiratory alkalosis, transient hepatotoxicity, acute renal failure and depressed mental status. Therapy is aimed at eliminating the offending agent through gastric lavage and administration of activated charcoal. If the mental status is optimal, vomiting can be induced. It is important to determine salicylate serum levels. Values greater than 35 mg/dL indicate significant acidosis, and alkalinization of the urine is indicated to enhance aspirin excretion. Case 45 A 36-year-old Caucasian nulligravida presents to your office for a routine check-up. Her past medical history is insignificant. She is concerned about the possibility of cervical cancer, because a friend of hers was recently diagnosed with invasive cervical cancer. You perform a Pap smear, and the results indicate that the sample is satisfactory, and a high-grade squamous intraepithelial lesion (HGSIL) is present. Which of the following is the next best step in the management of this patient? A. Repeat cytology in 6 months B. Proceed with colposcopy C. Do HPV testing D. Do excisional biopsy E. Reassure and repeat Pap smear in 12 months Explanation:

HGSIL revealed on Pap smear indicates a 1-2% probability of already having invasive cervical cancer and a 20% probability of acquiring invasive cervical cancer if left untreated. Immediate referral for colposcopy and endocervical curettage is indicated. (Choice D) If colposcopy suggests HGSIL, a diagnostic excisional procedure should be performed. (Choice C) HPV testing may be indicated if cytologic examination reveals atypical squamous cells of undetermined significance (ASCUS). The results of HPV testing may influence the decision to proceed with a colposcopy in such cases. (Choices A and E) Repeating the cytology in 6 or 12 months is not correct because invasive cervical cancer can be missed. Educational Objective: If a Pap smear reveals a high-grade squamous intraepithelial lesion (HGSIL), immediate referral for a colposcopy is indicated. 79% of people answered this question correctly. Case 46 An 18-month-old girl is brought to the office by her mother because of bilateral breast enlargement for the last three months. She has no known medical problems. There is no history of headaches, visual problems, growth of axillary or pubic hair, change in genitalia, rapid increase in height, or change in body habitus. On physical examination, she appears to be normal and playful. Her height and weight are at the 75th percentile on the growth curve for normal females (her weight and weight has remained at the 75th percentile on this growth chart for the last year). She has 3 cm of breast tissue on both sides. She has stage 1 pubic hair and no axillary hair or odor. Her genitalia are normal. The rest of the physical examination is within normal limits. Her LH, FSH, and estradiol levels are within normal range for her age group. TSH and prolactin levels are within normal limits as well. What is the most likely cause of this patient's breast enlargement? A. Benign premature thelarche B. Hypothalamic hamartoma C. Adrenal tumor D. Ovarian tumor E. McCune-Albright syndrome Explanation: The clinical features in this patient are consistent with benign premature thelarche, which is characterized by bilateral breast enlargement not accompanied by other signs of isosexual precocious puberty. These other signs of precocious puberty include rapid increase in height, increase in bone maturity, appearance of axillary and pubic hair, and menstrual bleeding. The treatment for benign premature thelarche is expectant because majority of the patients remain stable or have reversal of the breast enlargement in a few months. Patients with benign premature thelarche have a normal hormone profile. Their final height is generally not compromised. (Choice B) Hypothalamic hamartomas secrete GnRH and cause central isosexual precocious puberty in both males and females. Central precocious puberty is characterized by rapid acceleration of height, increase in bone age, thelarche, adrenarche, pubarche, and menarche. Lab investigations in patients with hypothalamic hamartoma reveal gonadotropin levels in the pubertal range and elevated estrogen levels. Majority of these patients will require treatment with a GnRH analog. (Choice C) Adrenal tumors which lead to the excessive production of estrogen are exceedingly uncommon. Adrenal tumors can produce cortisol and androgen. Excessive androgen production can

cause virilization in females (heterosexual precocious puberty) and peripheral isosexual precocious puberty in males. Excessive cortisol secretion leads to Cushing’s syndrome.

(Choice D) Estrogen production from an ovarian tumor can lead to peripheral precocious puberty. This syndrome has similar clinical features as hypothalamic hamartoma, and is characterized by accelerated height and bone age, and menstrual bleeding. Hormonal profile reveals elevated estradiol in the presence suppressed LH and FSH. Treatment is usually surgical. (Choice E) McCune-Albright syndrome consists of "café-au-lait" spots, fibrous dysplasia of the bone, and precocious puberty. This patient does not have any features to suggest McCune-Albright syndrome. The cause of precocious puberty in McCune-Albright syndrome is excessive production of estrogen from ovarian cysts. Educational Objective: Benign premature thelarche occurs at 18 to 24 months of age and is characterized by breast enlargement without other features of precocious puberty. The treatment of benign premature thelarche is expectant because majority of patients remain stable or improve in a few months. 58% of people answered this question correctly. Case 47 A 46-year-old male with type-2 diabetes mellitus is scheduled to undergo cardiac catheterization in two days for an undiagnosed episodic chest pain. The patient has been having retrosternal chest pain radiating to his arm for the past 15 days. The pain is gradually becoming more frequent. He went to the emergency department about 7 days ago following an episode of chest pain. He was admitted overnight, and he left against medical advice when three sets of cardiac enzymes were negative. His EKG revealed nonspecific ST-T changes. The patient has had diabetes for the past six years, and has been on metformin (1,000 mg twice daily) for the past several years. He does not perform selfmonitoring of his blood sugar. His chemistry profile and CBC from his last emergency department visit were normal. He also has a history of hypertension that is controlled with atenolol (50 mg once daily) and hydrochlorothiazide (12.5 mg once daily). Baby aspirin (81 mg once daily) was started on his last emergency room visit. He smokes one pack a day, and drinks alcohol socially. He has a strong family history of premature coronary artery disease. On examination, he is 5?6" (165 cm) tall, and weighs 180 lbs. (82 kg). His blood pressure is 136/70 mmHg and pulse rate is 66/min. The rest of the clinical examination is normal. What is the next best step in this patient?s care? A. Stop metformin B. Stop hydrochlorothiazide C. Increase atenolol D. Discontinue aspirin E. No change in therapy until cardiac catheterization Explanation: Metformin was approved in the United States for the treatment of type-2 diabetes after its use in the rest of the world showed that the chances of lactic acidosis are minimal when used in patients without any contraindications. Major contraindications include renal insufficiency (creatinine more than 1.5 mg/dl in males, 1.4 mg/dl in females or creatinine clearance less than 60 ml/min), hepatic dysfunction, alcoholics, sepsis, and congestive heart failure. Because of its potential to cause renal dysfunction when a large amount of contrast is being infused, metformin is typically held before or at the time of procedure. It should not be restarted unless normal renal function is documented following the procedure.

(Choice B) Hydrochlorothiazide is usually not catheterization, especially when used in low doses.

associated

with

any

problems with

cardiac

(Choice C) There is no need to increase atenolol dosage, because the patient appears to be adequately beta-blocked. (Choice D) Aspirin can be continued safely during cardiac catheterization. In procedures where an increased risk for bleeding is expected, aspirin should be discontinued at least seven days before the procedure, because it may cause platelet dysfunction that can last for more than a week. (Choice E) Metformin needs to be discontinued before cardiac catheterization. Educational Objective: Metformin use is contraindicated in patients with renal failure, sepsis, hepatic dysfunction, and severe heart failure. It should also be stopped in patients who are at risk to develop renal failure, such as those who will undergo angiography, a procedure that involves infusion of a high load of contrast agents. 37% of people answered this question correctly. Case 48 The following vignette applies to the next 3 items A 46-year-old Hispanic man is brought by the police to the emergency department (ED). The police found him unresponsive on a bench in a nearby park. His clothes are dirty and torn, and there is evidence of vomitus on his lips and clothes. His breath smells of alcohol. His temperature is 36 C (97 F), blood pressure is 96/75 mmHg, heart rate is 110/min, and respiratory rate is 12/min. He is extremely drowsy, disoriented and unresponsive to any of your questions. He has a disheveled appearance, and has cool extremities. Spider angiomas and palmar erythema can be identified on closer inspection. He has decreased breath sounds at the lower base of the right lung. The cardiovascular examination is within normal limits. His abdomen is soft; there is no tenderness or guarding. There is evidence of mild hepatomegaly. While in the ED, he has a large emesis with gross bright, red colored blood. Item 1 of 3 What is the next best step in the management of this patient? A. Perform the endotracheal intubation B. Initiate aggressive fluid resuscitation C. Start the patient on IV octreotide D. Send him for stat endoscopy E. Give the patient intravenous propranolol Explanation: Based on the above presentation, the patient is most likely suffering from an acute variceal hemorrhage secondary to chronic alcoholic liver disease. Acute variceal hemorrhage is a major cause of death in patients with chronic liver disease or cirrhosis. Apart from the acute blood loss, the complications of bleeding and its treatment are responsible for a significant amount of morbidity and mortality in these patients. The primary goal of management of patients with variceal hemorrhage is to maintain hemodynamic stability, prevent the complications of bleeding and its treatment, and control active bleeding. Adequate control and protection of the airway should always be a priority in the initial management of all patients with active or recent variceal bleeding. This can be initially done by keeping the patient in a left decubitus position or by gastric decompression via the insertion of a nasogastric tube. In patients who are disoriented and unresponsive, the airway can be secured via endotracheal intubation to prevent the risk of aspiration and its complications during active bleeding or its treatment. If the

patient is hemodynamically unstable at the initial presentation, aggressive fluid resuscitation and airway management should be performed simultaneously. The patient in the above vignette has marked disorientation and can deteriorate rapidly with further recurrent episodes of bleeding; therefore, it is important to secure his airway with endotracheal intubation at this point before instituting any specific therapy. (Choice B) Although the patient has had an acute bleeding episode, he appears to be hemodynamically stable. Airway protection should be the priority at this time. If there is any evidence of hemodynamic compromise, aggressive resuscitation and airway protection should be done simultaneously. (Choices C and D) The patient should not be sent for any specific treatment before securing his airway. (Choice E) Intravenous propranolol has no role in the management of active variceal bleeding. Educational Objective: Airway protection, followed closely or simultaneously by hemodynamic resuscitation, should be a priority in all patients with active variceal hemorrhage. Item 2 of 3 The initial appropriate steps were taken for the patient. His current temperature is 36.2 C (97.4 F), blood pressure is 112/66 mmHg, heart rate is 96/min, and respiratory rate is 13/min. The upper GI endoscopy shows large esophageal varices with evidence of recent bleeding, which is treated with injection sclerotherapy. He is admitted to the hospital for further observation and management. On the second day of hospitalization, he becomes extremely disoriented and belligerent. You suspect that he is having acute alcohol withdrawal. He is started on treatment for alcohol withdrawal, and his condition improves over the next day. On his fourth day of hospitalization, he has another episode of large bloody emesis. What is the most appropriate next step in the management of this patient? A. Repeat endoscopy and band ligation B. Perform a Transjugular intrahepatic portosystemic shunt C. High-dose intravenous octreotide D. Sengstaken-Blakemore tube insertion E. Give the patient intravenous propranolol Explanation: Urgent upper GI endoscopy with either injection sclerotherapy or band ligation is the definite treatment of choice for patients with active variceal hemorrhage. It is generally successful in controlling acute bleeding in 80-90% of the patients. All patients with a recent variceal hemorrhage are at a high risk of recurrent hemorrhage or early re-bleeding. This risk of re-bleeding is greatest within the first 48-72 hours, and remains high for the first six weeks following the cessation of active bleeding. Some of the known risk factors for early re-bleeding include an age greater than 60 years, severe initial bleeding, large varices, and the presence of renal failure. In all patients who have another episode of variceal bleeding more than 48 hours after the initial endoscopic treatment, as in the above vignette, a repeat endoscopy with band ligation should be attempted initially to achieve hemostasis. If the patient continues to bleed despite endoscopic treatment, more definitive treatment with either balloon tamponade or portosystemic shunt should be used to control the active bleeding. (Choices B and D) Endoscopic treatment with band ligation or sclerotherapy is the treatment of choice for active variceal bleeding. Sengstaken-Blakemore tube insertion for balloon tamponade or TIPS procedure should be employed to achieve hemostasis only if the endoscopic treatment fails to control the bleeding.

(Choice C) Somatostatin or intravenous octreotide can initially be used in patients with suspected acute variceal hemorrhage. It should not replace endoscopic treatment as a more definite procedure of choice for acute variceal hemorrhage. Educational Objective: Endoscopic therapy with either band ligation or sclerotherapy is the definite procedure of choice for early re-bleeding in patients with recent variceal hemorrhage. Item 3 of 3 The appropriate step is taken. One week later, the patient is ready for discharge. In addition to providing counseling to completely stop alcohol consumption, which of the following is the treatment of choice to reduce recurrent bleeding? A. Proton pump inhibitors B. Propranolol C. Cimetidine D. Prostaglandin analogs E. Follow-up endoscopy every six months Explanation: All patients with a history of variceal hemorrhage are at high risk for re-bleeding or recurrent variceal hemorrhage. This risk is greatest in the first few days, and returns to baseline by six weeks; however, if left untreated, approximately 70% of the patients suffer from recurrent variceal bleeding within the first year from the initial bleeding episode. Recurrent bleeding is one of the important causes of death in such patients. Therefore, it is important to institute specific measures to prevent recurrent variceal hemorrhage. Of all the above available options, propranolol (non-selective beta-blockers) has been shown to reduce the risk of recurrent variceal hemorrhage. This is likely due to reduction in the portal pressure or hepatic vein pressure gradient seen with the use of non-selective beta-blockers. (Choices A, C, and D) H2 blockers (Cimetidine), proton pump inhibitors or prostaglandin analogs have not been shown to reduce the risk of recurrent variceal hemorrhage. (Choice E) Regular endoscopic treatment with band ligation is the treatment of choice for long-term management of variceal hemorrhage. Follow-up endoscopy alone has no role in the prevention of recurrent bleeding. Educational Objective: Non-selective beta-blockers (propranolol or nadolol) have been shown to reduce the risk of re-bleeding in patients with a history of variceal hemorrhage. 74% of people answered this question correctly. Case 49 A 12-year-old Caucasian female develops an episode of severe diarrhea that requires hospitalization. Her past medical history is insignificant, and she has no history of recent travel. Her stool sample is positive for Giardia lamblia trophozoites. After hearing about her condition, the parents of her classmates insisted on having their children screened for giardiasis. Twenty percent of the class turned out to be positive for Giardia cysts on stool ova and parasite test. Which of the following is the best strategy to manage giardiasis in this case? A. Isolate all carriers B. Treat all carriers, but not isolate them C. Treat all carriers and their family members D. Treat only symptomatic carriers E. Treat all carriers and people at risk

Explanation: Giardiasis has several routes of transmission: person-to-person, food-borne and waterborne. Personto-person transmission occurs in two settings: (1) in institutions where there is fecal incontinence and poor hygiene (e.g., some daycare centers), and (2) in male homosexuals. Symptomatic patients with giardiasis should receive appropriate treatment. (Choice A) Asymptomatic infection occurs in approximately 60 percent of people exposed to Giardia. Asymptomatic cyst carriage can last over six months. There is no need to isolate asymptomatic carriers once appropriate hygienic measures are undertaken. (Choices B, C and E) Asymptomatic carriers are not usually treated, except in specific instances such as outbreak control and for prevention of household transmission by toddlers to pregnant women and patients with hypogammaglobulinemia or cystic fibrosis. Educational Objective: Asymptomatic carriers of Giardia lamblia are not usually treated, except in specific instances such as in outbreak control and for prevention of household transmission by toddlers to pregnant women and patients with hypogammaglobulinemia or cystic fibrosis. 38% of people answered this question correctly.

Case 50 A 6-year-old boy is brought to the pediatrician after a sudden onset of fever and difficulty in breathing. According to his mother, he was complaining of a sore throat and some trouble swallowing yesterday, but these symptoms resolved after he received some over-the-counter analgesics. On physical examination, he appears toxic. His voice is muffled. He is leaning forward and moving around restlessly. His mouth is open wide and his tongue is protruding. What is the most appropriate course of action? A. Obtain lateral neck radiograph B. Perform laryngoscopy in office C. Refer to local pediatric hospital for laryngoscopy D. Administer corticosteroids and racemic epinephrine E. Call ambulance and send to emergency department Explanation: With the introduction of the Haemophilus influenzae type b vaccine in 1985, epiglottitis has become much less common in recent years. When it does occur, it causes an inflammatory edema of the epiglottis that impinges upon the airway. Respiratory arrest is easily provoked at this stage, and treatment should be focused on relieving any airway obstruction and treating the infection. The threshold for performing intubation should be very low. Since the outpatient setting is inadequate for such procedures, it is imperative that an ambulance be called so the child’s condition can be properly addressed by emergency personnel. (Choice A) Frequently, epiglottitis is diagnosed solely by the clinical presentation. However, if a lateral neck radiograph is deemed necessary, it will reveal a swollen epiglottis (the "thumbprint sign") in classic epiglottitis. In the child in this case, an endotracheal intubation should be performed before the radiograph is obtained, because he is at great risk for respiratory arrest. (Choices B and C) Laryngoscopy can be used to exclude other causes of airway obstruction in a cooperative child; however, it is not recommended for cases of suspected epiglottitis as it may worsen the patient's condition.

(Choice D) Corticosteroids and racemic epinephrine have not been shown to be helpful in the treatment of epiglottitis. Educational Objective: Epiglottitis is a serious, life-threatening infection. Treatment should be focused on relieving any airway obstruction and treating the infection. The threshold for performing intubation should be very low. Case 51 The following vignette applies to the next 3 items A 9-month-old Caucasian male is brought to the emergency department on one winter night with oneday history of poor appetite, nasal discharge and wheezing. His past medical history is insignificant. He has no known allergies, and all his vaccinations are up-to-date. He has been meeting all developmental milestones appropriately. His temperature is 38.3 C (101 F), blood pressure is 90/60 mm Hg, pulse is 150/min, and respirations are 40/min. Oxygen saturation on room air by pulse oximetry is 91%. Lung auscultation reveals expiratory wheezing and prolonged expiration. The liver edge is palpable. Chest-x ray shows hyperinflation. The patient is admitted to the hospital. Item 1 of 3 Which of the following is the best management for this patient? A. Respiratory isolation and trial of bronchodilators B. Short course of corticosteroids C. Respiratory isolation and ribavirin therapy D. Administer decongestants and expectorants E. Respiratory isolation and erythromycin for 7-10 days Explanation: This patient presents with a clinical picture that is most consistent with bronchiolitis: young age (less than 2 years), mild upper respiratory symptoms, mild-to-moderate fever and wheezing are typical. The liver and spleen are palpable because of hyperinflation of the lungs, but these are not enlarged. In healthy infants and young children, bronchiolitis is usually a self-limited disease; however, patients who are hypoxic or cannot feed because of distress should be hospitalized. These children should be kept in respiratory isolation. Therapy in most cases consists of supportive measures. Humidified oxygen and tube or intravenous feedings are indicated. Although there is no strong evidence that inhaled bronchodilators are effective in patients with bronchiolitis, it is a routine practice to administer these (nebulized albuterol or epinephrine) and observe the patient for any effect. If no prompt clinical response is seen, most clinicians discontinue these drugs. (Choice B) Corticosteroids are not routinely recommended, but may be beneficial in patients with an underlying chronic lung disease and in those with prior wheezing episodes. (Choice C) Although ribavirin is a nucleoside analogue with good in vitro activity against RSV, studies examining its effect in children have been conflicting, and the cost for a course of therapy is substantial. It is usually reserved for patients with severe disease. (Choices D and E) Antibiotics, decongestants, and expectorants are of no value in the treatment of patients with bronchiolitis. Educational Objective: Although there is no strong evidence that inhaled bronchodilators are effective in patients with bronchiolitis, it is a routine practice to administer these and observe the patient's clinical response. 55% of people answered this question correctly.

Item 2 of 3 Which of the following is the most appropriate next step in the diagnosis of his condition? A. Rapid tissue culture B. Rapid detection of antigen in nasal secretions C. Serology for antibody detection D. Sputum induction and Gram stain E. Urinary antigen assays Explanation: Rapid detection of RSV antigen in nasal or pulmonary secretions utilizing antigen capture technology is now available and can be performed in less than 30 minutes. The sensitivity and specificity of such tests exceed 90 percent. (Choice B) Even though the definitive diagnosis is made by tissue culture, it takes 4 days to 2 weeks for the results to be reported. (Choice C) Diagnostic serology is not helpful in infants because this also detects maternal antibodies. Educational Objective: Diagnosis of RSV is quickly made by detection of RSV antigen in nasal or pulmonary secretions by ELISA. 63% of people answered this question correctly. Item 3 of 3 While you are giving the appropriate management to this patient, his mother asks about what she should expect in the future. Which of the following diseases is the child at risk of developing? A. Pneumonia B. Lung abscess C. Cystic fibrosis D. Asthma E. Aspergillosis Explanation: Although mild RSV infection does not produce apparent long-term pulmonary sequelae in most individuals, up to 30% of patients hospitalized with severe infection will subsequently develop reactive airway disease later in childhood (Choice A) A secondary bacterial infection of the middle ear is the most common complication (1020%) of RSV bronchiolitis, but bacterial pneumonia is very rare (0.5-1%). (Choice B) Lung abscess is not seen with respiratory syncytial virus. (Choice C) These children are not prone to cystic fibrosis (CF). CF is a congenital condition and is not acquired from any infection. (Choice E) Aspergillosis infections generally occur in immunocompromised patients. The disease may present as an allergic reaction involving the lung, or may present with a cavitary lesion in the lung. Disseminated Aspergillus infections are only seen in immunocompromised individuals. Aspergillus infections are not related to prior respiratory syncytial infections. Educational Objective:

A secondary bacterial infection of the middle ear and future risk of bronchial hyperreactivity are the most common complications of RSV bronchiolitis. 68% of people answered this question correctly. Case 52 The following vignette applies to the next 2 items A 50-year-old obese man comes to the emergency department with a painful left great toe. He was sleeping peacefully at night, when he woke up with sudden severe pain in his left great toe. He never had this kind of problem in the past. He denies trauma to the affected part. He smokes 1 pack of cigarettes daily and drinks alcohol regularly. Physical examination shows normal vital signs. His left great toe is inflamed and swollen. The patient does not allow further examination due to intense pain. There is no skin rash. Item 1 of 2 What is the next step in making the diagnosis? A. Allopurinol B. Arthrocentesis C. X-ray of the great toe D. MRI E. Aspergillosis Explanation: The patient has features suggestive of acute monoarticular gout. The diagnosis should be confirmed by needle aspiration of the joint fluid. The joint aspirate in gout shows characteristic negatively birefringent crystals of monosodium urate. An infection can coexist with acute gout; therefore, the joint fluid should be cultured if there is any suspicion for an infectious process. (Choice A) Serum uric acid is not used for making a diagnosis of acute gout because the uric acid levels can be normal or even low during an acute attack. (Choices C & D) Radiologic imaging plays no role in the diagnosis of acute gout. Educational Objective: A diagnosis of acute gout is confirmed by arthrocentesis by the demonstration of negatively birefringent crystals. Serum uric acid can be normal during acute gout and is not useful in making the diagnosis. 82% of people answered this question correctly. Item 2 of 2 The diagnosis of acute gout is made. Which of the following is the treatment of choice at this time? A. Allopurinol B. Colchicine C. Indomethacin D. Probenecid E. Methotrexate Explanation: NSAIDS are used as the first line agents in the treatment of acute gout in most patients. NSAIDS are effective in more than 90% of patients with acute gout. Agents with a short half-life (e.g.,

indomethacin and ibuprofen) are most effective. Majority of the patients become asymptomatic in 5-7 days. Other anti-inflammatory agents such as colchicine or glucocorticoids can be used. Intra-articular glucocorticoids may be used in elderly patients with renal failure. (Choices A and D) Hypouricemic therapy with allopurinol or probenecid should not be started during acute gout. When hypouricemic therapy is started during an acute episode, there are high chances of flare-up of the disease. (Choice B) Colchicine can be used in the treatment of acute gout. Typically, 0.6 mg is given every hour until there is relief of symptoms or GI toxicity occurs. A total of 2 to 3 mg can be given. High doses of colchicine are associated with serious toxicity. Lower doses are advocated in elderly patients. Due to its potential serious toxicity, colchicine is not the first line agent in most patients with acute gout. (Choice E) Methotrexate is not indicated in the treatment of acute or chronic gout. Educational Objective: Acute gouty arthritis is treated with NSAIDs as first line therapy. Colchicine and glucocorticoids can also be used. Hypouricemic therapy is never used during an acute episode of gout. 79% of people answered this question correctly. Case 53 A large-scale clinical trial was conducted to assess the effect of carvedilol (a mixed alpha- and betablocker on the clinical course of chronic heart failure), NYHA class III-IV. The study showed that serum sodium level correlated with serum noradrenaline level, a marker of the degree of neurohumoral activation: correlation coefficient, r = - 0.39 and p = 0.005. Which of the following statements correctly describes the association between serum sodium level and serum noradrenaline level observed in this study? A. Strong correlation was observed B. As serum sodium level increases serum noradrenaline level also increases C. Serum sodium changes cause serum noradrenaline changes D. The association does not reach statistical significance E. Negative correlation is observed Explanation: A negative correlation is present between serum sodium level and serum noradrenaline level, which is indicated by the negative sign of the correlation coefficient. The ‘null’ value for the correlation coefficient is 0 (no association), and the range of plausible values is from -1 to 1. The sign of the correlation coefficient indicates positive or negative association. The closer the value is to its margins (-1 or 1), the stronger the association. (Choice B) Negative correlation means that as serum sodium level decreases, serum noradrenaline level increases. (Choices A and D) In this scenario, the association is weak because the value of the correlation coefficient is close to 0, although it is statistically significant: p = 0.005. (Choice C) It is very important to know that the correlation coefficient shows the strength of association, but does NOT necessarily imply causality. Educational Objective: The correlation coefficient shows the strength and the direction (positive, negative) of linear association between two variables. It does not necessarily imply causality.

48% of people answered this question correctly. Case 54 A 49-year-old blue-eyed, blonde-haired Caucasian woman presents to clinic requesting treatment for "this unattractive redness of my facial skin." She reports that in the past three months, she has developed new blemishes on her nose, cheeks, and chin. She has also noticed that she now flushes intensely in response to extreme temperatures, emotion, and the consumption of spicy foods or alcohol. Based on recommendations from her friends, she modified her diet and tried various herbal preparations and skin creams, but nothing has alleviated her symptoms. She has no history of dermatological problems and did not have acne as a teenager. She has already undergone menopause. Physical examination reveals a reddening of the central face, erythematous papules on the chin, and multiple telangiectasias on the nose and cheeks. Her skin is very fair otherwise. Item 1 of 2 Which of the following is the most appropriate treatment of this condition? A. Oral griseofulvin B. Topical clindamycin C. Topical isotretinoin D. Topical metronidazole E. Topical permethrin Explanation: Rosacea is a chronic acneiform condition characterized by vascular dilation in the central face. It is most common in adults ages 30 to 60 years and occurs more often in individuals with light skin, hair, and eye color. Periods of exacerbation and remission are expected. Symptoms generally include facial erythema (both transient and nontransient), telangiectasias, and papules and pustules. The inflammatory lesions seen generally appear identical to those found with acne, although comedones are not present. Therapy is usually successful in reducing the number and severity of the inflammatory lesions, but facial erythema may persist despite treatment. It is important that patients with rosacea use mild cleansers and sunscreens regularly while also avoiding irritants. One of the most frequently used initial treatments for this condition is topical metronidazole (Choice D), with or without a course of oral antibiotics. The metronidazole may need to be applied daily on a long-term basis to maintain remission. Oral griseofulvin (Choice A) is not prescribed for the treatment of rosacea. The oral antibiotics commonly used include tetracycline, doxycycline, minocycline, and erythromycin. Topical clindamycin (Choice B) can be used in the form of a solution, gel, or lotion, but is generally considered less effective than topical metronidazole. Topical isotretinoin (Choice C) is appropriate for those patients with papular or pustular lesions unresponsive to initial therapies. It may also be used in conjunction with topical antibiotics, especially in those patients with refractory symptoms. Some clinical data suggests that topical permethrin (Choice E) improves symptoms of rosacea, a not unexpected finding as Demodex mites are frequently found in increased numbers in the skin scrapings of patients with rosacea. However, further evaluation is indicated before this treatment can be recommended as appropriate first-line therapy. Educational Objective: Rosacea is a chronic acneiform condition characterized by vascular dilation in the central face. An excellent initial treatment for this condition is topical metronidazole, with or without a course of oral antibiotics. Item 2 of 2 The appropriate action was taken. Which of the following complications is most frequently associated with this condition?

A. Chalazion B. Glaucoma C. Endophthalmitis D. Sinusitis E. Squamous cell carcinoma Explanation: Patients with rosacea often report experiencing ocular symptoms. Complaints frequently include burning or foreign body sensations, blepharitis, keratitis, conjunctivitis, episcleritis, and chalazion (Choice A). The chalazion is characterized by a granulomatous inflammation of a meibomian gland and presents as a painless, pea-sized nodule within the eyelid. Acute angle-closure glaucoma (Choice B) is manifested as a painful, red eye. Accompanying symptoms may include nausea, vomiting, and headache. Glaucoma is not typically associated with rosacea. Endophthalmitis (Choice C) is classically associated with corneal ulcers and ocular infections with organisms such as Pseudomonas, Yersinia, and Mycobacterium. Rosacea, in contrast, usually causes a more focal and superficial ocular irritation. Sinus infections (Choice D) can arise when mucosal edema obstructs the ostia, often subsequent to viral upper respiratory tract infections. Sinusitis is not typically associated with rosacea. Classically, squamous cell carcinoma (Choice E) presents as an ulcerated nodule or superficial erosion of sun-damaged skin or lip. Although it is more common in the fair-skinned, is not typically associated with rosacea. Educational Objective: Patients with rosacea may also experience ocular symptoms. Complaints frequently include burning, foreign body sensation, blepharitis, keratitis, conjunctivitis, episcleritis, and chalazion (dysfunction of the meibomian gland). Case 55 A 66-year-old postmenopausal female comes to the emergency department and complains of fever and left flank pain for the past four days. She also complains of chills, increased frequency of micturition, and dysuria. The pain in her left flank is described as agonizing, non-radiating, and aggravated by motion. She denies any hematuria, cough, dyspnea, nausea, vomitings, and skin rash. Her past medical history is significant for type-2 diabetes for the past ten years, hypertension, dyslipidemia and osteoporosis. Her fasting blood glucose levels have been ranging between 100-140 mg/dl. Her HBA1c, performed 2 weeks ago, was 8.2 % (normal 4-6%). Physical examination reveals a middle-aged woman in distress, with a temperature of 102 F(39 C), blood pressure of 100/70 mm Hg, and heart rate of 100/min. There is marked tenderness in her left flank. CBC reveals polymorphonuclear leukocytosis. The basic metabolic panel is normal, except for a BUN level of 33mg/dl (normal 10-20 mg/dl) and glucose level of 320 mg/dl. Urine examination shows 1+ protein, 3 + glucose, many WBCs, WBC casts and positive leucocyte esterase. Blood cultures are drawn. CT scan of the abdomen shows a fluid collection in the left flank, with extensive perinephric stranding. Air is seen in the left kidney and perinephric space. Besides IV hydration and glycemic control, what is the best next step in this patient's care? A. Parental antibiotics B. Parental antibiotics and percutaneous drainage C. Parental antibiotics and immediate nephrectomy D. Parental antibiotics and surgical drainage E. Oral antibiotics Explanation: There are a number of infections which predominantly occur in diabetic individuals. Emphysematous pyelonephritis is one of them. E. coli and K. pneumoniae are the most common organisms involved.

Treatment is guided by the extent of involvement of the perinephric space. If abscess and gas extend into the perinephric space, an immediate nephrectomy is necessary. (Choice B) Mild pyelonephritis is managed with a combination of parental antibiotics and percutaneous drainage. (Choices A and E) Parental antibiotics alone are not generally sufficient, and oral antibiotics are usually ineffective. (Choice D) Surgical drainage does not offer a significant advantage over percutaneous drainage. Emphysematous cholecystitis also occurs predominantly in diabetics. As compared to emphysematous pyelonephritis, which occurs more commonly in females, emphysematous cholecystitis is seen commonly in males. Clostridia and E. coli are the most common organisms responsible for emphysematous cholecystitis. The most common symptoms are fever, right upper quadrant pain, nausea and vomiting. Crepitus on abdominal examination is very suggestive of emphysematous cholecystitis. The diagnosis is made on CT scan, which depicts gas within the gall bladder. All patients require urgent removal of the infected gall bladder and parental antibiotics. Open surgical removal of the infected gall bladder is preferred over laparoscopic removal. Educational Objective: Parenteral antibiotics and immediate nephrectomy is indicated in patients with advanced emphysematous pyelonephritis. Case 57 A 45-year-old female patient of yours comes to see you in the office. She is accompanied by her 14year-old daughter. She has been your patient for the last 20 years, and you have an excellent rapport with her. Her daughter became sexually active three months ago. She is aware of the risk of cervical cancer with sexual activity, and wants you to convince her daughter to have a cervical Papanicolaou smear done. What is the most appropriate response in this setting? A. Reassure the mother & tell her that a Pap smear is not necessary at this time. B. Reassure the mother and tell her that you will do the Pap smear at this time. C. Tell the mother that you will do the Pap smear if she has been exposed to multiple partners. D. Tell the mother that you will do the Pap smear if she has engaged in unprotected intercourse. E. Tell her that she does not need a Pap smear until she is 18 years of age. Explanation: Human papilloma virus (HPV) infection is a sexually transmitted infection strongly linked with the development of cervical intraepithelial neoplasia (CIN) and cervical cancer. Early initiation of sexual activity, multiple new sexual partners, and high-risk partners (partners with HPV infection), are the main risk factors for the acquisition of HPV infection and cervical cancer. Screening for cervical cancer by cytologic examination/Pap smear is an effective way of detecting early pre-invasive and invasive carcinoma. According to the guidelines, screening for cervical intraepithelial neoplasia (CIN) or cancer should be started three years after the onset of sexual intercourse, or at the age of 18 years (recently changed to 21 years) whichever is earlier. This is due to the fact that high-grade cervical cytologic abnormalities due to HPV usually do not occur until three to five years after HPV exposure. The daughter in the above vignette started engaging in sexual activities only three months ago. There is no reason to start screening her for cervical cancer by Pap smear at this time. The mother should be assured and informed of the facts regarding the timing of exposure and initiation of screening. (Choices C and D) The initiation of cervical cancer screening does not depend on the history of multiple partners or unprotected intercourse. Educational Objective: Cervical cancer screening should be started three years after initiation of sexual intercourse or at the age of 18 years (some say 21 years). Case 58 A 30-year-old female presents with tingling and numbness in both of her distal lower extremities for the last three months. Her symptoms are gradually worsening. She also complains of intense fatigue. She denies weight change, constipation, skin dryness, headaches or visual changes. She was

diagnosed with primary hypothyroidism secondary to Hashimoto’s thyroiditis when she was 10 years old. She is currently on 100 mcg per day of levothyroxine orally. Her mother also has hypothyroidism. She does not smoke or drink alcohol. She denies the use of recreational drugs. Her menstrual cycles have been irregular for the last six months. Her last menstrual period was two months ago. She is sexually active with one partner. Her blood pressure is 114/78 mm Hg and heart rate is 72/min. She weighs 152 pounds and is 5’4" tall. Her thyroid is nonpalpable. Her mucous membranes are moist and pale. Her tongue appears to be bald. She does not have any scleral icterus. Vibration and proprioception are decreased in both of her lower extremities distally. Pain and temperature sensations appear normal. Her ankle jerks are absent, but other reflexes are brisk. Babinski and Romberg’s signs are positive. The rest of the physical examination is unremarkable. Stool obtained from the rectal examination is negative for occult blood. Lab investigations reveal a hemoglobin of 8 g/dL and hematocrit of 24%. Her total WBC count is 3,200 per cubic millimeter. Her platelet count is 300,000 per cubic millimeter. Basic serum chemistries are within normal limits. Her Total T4 level is 8.2 mcg/dL (normal 4-12 mcg/dL) and TSH level is 2.4 micro IU/mL (normal 0.5 to 5 micro IU/mL). Item 1 of 2 What is the next best step in the management of this patient? A. Increase her levothyroxine dose to improve her fatigue B. Perform MRI of the spine to look for spinal cord compression C. Serum protein electrophoresis D. Measure serum B12 levels E. Measure folate levels Explanation: The patient's primary hypothyroidism predisposed her to other autoimmune disorders such as pernicious anemia. Involvement of the posterior column (loss of proprioception), lateral column (brisk reflexes) and peripheral nerves (loss of ankle jerks) indicate neurologic involvement due to Vitamin B12 deficiency, which resulted from atrophic gastritis (pernicious anemia). Dietary vitamin B12 binds to the intrinsic factor secreted by the parietal cells in the gastric mucosa. The intrinsic factor-vitamin B12 complex is then carried to the terminal ileum for receptor-mediated absorption. Pernicious anemia results from the autoimmune destruction of parietal cells, leading to achlorhydria and decreased production of the intrinsic factor. The lack of intrinsic factor leads to vitamin B12 deficiency. The red blood cells are macrocytic due to ineffective erythropoiesis secondary to defective nucleic acid synthesis. Neurologic involvement is characterized by involvement of posterior and lateral columns in the spinal cord known as 'subacute combined degeneration' and leads to ataxia, loss of proprioceptive and vibratory sensations. Severe spasticity, weakness, and peripheral nerve involvement can occur. Symptoms and signs are more prominent in the lower than the upper extremities. (Choice A) Increasing the levothyroxine dose is not the appropriate choice for this patient because her thyroid function tests are within normal limits. The fatigue in this patient is most likely due to anemia. (Choice B) Although neurologic involvement can indicate cord compression, the history and other features of this patient suggest otherwise; therefore, MRI is unnecessary. (Choice C) Endocrinopathy with polyneuropathy can be seen with POEMS (polyneuropathy, organomegaly, endocrinopathy, M-proteins and skin changes) syndrome. SPEP is done to look for an M-spike. This patient does not have organomegaly or skin changes. (Choice E) Folate levels may be obtained to determine the cause of marocytic anemia; however, the patient's history of autoimmune hypothyroidism and characteristic neurological picture is more suggestive of vitamin B12 deficiency. Educational Objective: Patients with primary hypothyroidism are predisposed to get other autoimmune diseases such as pernicious anemia. Vitamin B12 deficiency in pernicious anemia is due to a deficiency of intrinsic factor secreted by the stomach. Neurologic involvement in Vitamin B12 deficiency is characterized by subacute combined degeneration of the spinal cord and peripheral neuropathy. 87% of people answered this question correctly.

Item 2 of 2 Which of the following is most crucial to monitor during the first few days of treatment in the above patient? A. TSH level B. Hemoglobin level C. Serum potassium level D. Platelet count E. White blood cell count Explanation: Treatment with vitamin B12 in patients with moderate to severe megaloblastic anemia can cause hypokalemia, which can sometimes be very severe and life threatening. Hypokalemia results following the uptake of potassium by newly forming red blood cells. The patients' serum potassium levels should therefore be monitored during the first 48 hours. Potassium is replaced depending on the measured serum potassium levels. Some physicians transfuse packed red blood cells in patients with severe megaloblastic anemia before Vitamin B12 supplementation to prevent hypokalemia. (Choices A, B, D & E) These parameters are not necessary to monitor during the first few days of B12 therapy. Educational Objective: Patients with moderate to severe megaloblastic anemia can have severe hypokalemia during the first 48 hours of treatment with vitamin B12. Close monitoring and supplementation of potassium is required during this period. Case 59 A 36-year-old female comes to see you in the office for smoking cessation counseling. She has been struggling to quit cigarette smoking for the past year. She has a 50-pack-year history of smoking, and currently smokes two to three packs of cigarettes daily. She works as a legal assistant in a very reputable law firm in the city. She recently got married, and her husband does not smoke at all. They are planning to have a child next year, and she is really interested in quitting before that time comes. She has multiple failed attempts to quit smoking in the past. She says, "Every time I try to quit smoking, I start shaking really bad. I can't sleep, and I get very anxious and frustrated. I can't seem to work, which is bad because my job is very demanding. Eventually, I can't take it anymore, and I start smoking again. But this time, I'm really determined to quit. I need your help, doctor." Which of the following is the most appropriate next step in her management? A. Prescribe a low-dose nicotine spray. B. Prescribe bupropion. C. Encourage her to continue her attempts at smoking cessation. D. Ask her to change her job. E. Prescribe a high-dose nicotine patch. Explanation: Cigarette smoking causes significant physiological and psychological dependence. Withdrawal symptoms are extremely common in patients attempting to quit, and are especially pronounced in patients with a long history of heavy cigarette smoking. Some of the common symptoms include restlessness, anxiety, irritability, frustration, decreased concentration, insomnia, or depressed mood. These generally peak in the first three to four days after smoking cessation, and usually resolve in the next three to four weeks. These symptoms and the associated intense craving for cigarettes are responsible for the majority of relapses seen in patients. Most smokers typically make several attempts to quit before they are able to quit successfully. Current recommendations for the management of smoking cessation involve a combination of behavioral therapy and a pharmacologic intervention, such as buproprion or nicotine replacement. Nicotine replacement therapy (gum, patch or inhaler) acts by maintaining a low level of nicotine in the

blood. It is preferred for the amelioration of withdrawal symptoms during the early stages of smoking cessation. (Choice A) A low-dose nicotine inhaler is unlikely to ameliorate the withdrawal symptoms of a patient with a history of heavy smoking. She should initially receive high-dose nicotine replacement therapy. (Choice B) Nicotine replacement therapy is preferred to buproprion for the management of withdrawal symptoms in the early stages of smoking cessation. (Choices C and D) The patient needs a specific intervention to ameliorate her withdrawal symptoms in addition to moral support and behavioral therapy. Educational Objective: Nicotine replacement therapy is extremely helpful in ameliorating the symptoms of nicotine withdrawal in the early stages of smoking cessation.

Case 60 A 7-year-old Caucasian boy is brought to your office by his mother because of a sudden acceleration of growth and a ‘very annoying rash.’ His past medical history is insignificant. Physical examination reveals severe cystic acne involving his face and shoulders, enlarged testicles and penis, and coarse pubic and axillary hair. His height corresponds to 98 percentile and his weight to 85 percentile for his age. Which of the following is the most likely diagnosis in this patient? A. Hypothalamic dysfunction B. 21-hydroxylase deficiency C. Severe hypothyroidism D. Abnormal karyotype 47, XXY E. Abnormal karyotype 47, XYY Explanation: It is very important to distinguish between the two most common presentations and causes of precocious pubarche in order to facilitate the proper treatment. Precocious puberty is caused by premature activation of the hypothalamus-pituitary-gonad (HPG) axis, while precocious pseudopuberty is caused by a gonadotropin-independent process, typically an excess of sex steroids. This patient presents with signs of severe androgen excess (e.g. severe cystic acne, significant growth acceleration) which suggests precocious pseudo-puberty. It can be caused by late-onset congenital adrenal hyperplasia. (Choice A) Hypothalamic dysfunction leading to precocious puberty is usually less dramatic in presentation. Sequential development of the following is typically present: testicular enlargement, penis enlargement, pubic hair growth, and lastly, a growth spurt. (Choice C) Severe hypothyroidism is a rare cause of precocious puberty that is characterized by the slowing, not acceleration, of growth. (Choice D) Klinefelter’s syndrome may present with a height that is higher than normal, but signs of androgen excess are not typical. (Choice E) 47, XYY karyotype may manifest as severe acne, but precocious puberty is not characteristic. Educational Objective: Precocious pubarche with signs of severe androgen excess is suggestive of precocious pseudopuberty that is caused by a gonadotropin-independent process, typically an excess of sex steroids.

Case 61 A 74-year-old Chinese-American woman is brought to the emergency department. She has had fever, chills, productive cough, and shortness of breath for the past 24 hours. Her other medical problems include hypertension and rheumatoid arthritis. Her medications include enalapril and naproxen. Her temperature is 38.9 C (102 F), blood pressure is 150/90 mmHg, pulse is 106/min., and respirations are 22/min. Examination showed decreased breath sounds and crackles at the left base. Fecal occult blood per rectum was negative. Her laboratory tests reveal the following: CBC Hb 9.4 g/dL Platelet count 450,000/cmm Segmented neutrophils 84%

MCV 84 fl Leukocyte count 10,000/cmm Lymphocytes 16%

The chest x-ray showed an alveolar infiltrate in the left base. The patient was started on intravenous ceftriaxone and oral azithromycin. On the third day of treatment, additional hematologic tests are done, and these show the following: Ferritin 300 ng/mL (NV 15-120) Iron 44 mcg/dL (NV 50-170) TIBC 200 mcg/dL (NV 250-450) Which of the following is the most appropriate diagnostic test for this patient’s anemia? A. Erythropoietin levels B. Bone marrow biopsy C. Serum Transferrin D. Serum Transferrin saturation E. Erythrocyte sedimentation rate (ESR) Explanation: This patient has rheumatoid arthritis (RA), left lower lobe pneumonia, and normocytic, normochromic, anemia. Although RA usually causes anemia of chronic disease, a reliable diagnosis in this setting can only be established by performing a bone marrow biopsy. (Choices C and E) Because of the patient's acute infection, the ESR and ferritin levels will be abnormally increased, while the serum transferrin level and TIBC will be decreased. (Choice D) The serum transferrin saturation (serum iron/TIBC) and iron levels will not distinguish between anemia of chronic disease and iron deficiency anemia because both will present with diminished levels. (Choice A) Measurement of erythropoietin levels can help with decisions regarding the most adequate treatment of anemia of chronic disease in a patient with RA. This is not used to diagnose the anemia itself. Educational Objective: Anemia of chronic disease (ACD) can be difficult to distinguish from iron deficiency anemia in the setting of an acute bacterial infection. In this setting, the best test to confirm the diagnosis and rule out iron deficiency anemia is a bone marrow biopsy. Case 62 A 32-year-old Caucasian female comes to the emergency department (ED) and complains of a generalized, throbbing, headache that began during the night and got worse in the morning. She has also brought in her 8-year-old son, who has a similar type of headache and nausea. She says that her husband went to work today, although he was not feeling well. She denies any previous illness, and

has no idea what might have brought these symptoms on. Her temperature is 36.7C(98F), blood pressure is 120/76 mm Hg, pulse is 90/min, and respirations are 16/min. No neck rigidity or meningeal signs are present on physical examination. Which of the following tests can confirm the most probable diagnosis in this patient? A. Non-contrast CT scan of the head B. Lumbar puncture C. CBC with differential D. Carboxyhemoglobin level E. Serum acetaminophen level Explanation: The clinical scenario described is highly suggestive of carbon monoxide (CO) poisoning. The most important feature that should be emphasized is the involvement of several people that have similar symptoms. Interestingly, accidental CO poisoning has seasonal and regional variations, and are more common during cold winters in cold climates. The potential causes of the problem may be poorly functioning heating systems or improperly vented fuel-burning devices (e.g., kerosene heater). Clinical symptoms of CO poisoning include throbbing headaches, nausea, malaise and dizziness. Severe poisoning may result in seizures, syncope and coma. Delayed neuropsychiatric syndrome develops in up to 40% of patients with significant CO poisoning. The diagnosis is confirmed by carboxyhemoglobin level measurement. (Choicees A and B) Other diagnoses such as subarachnoid hemorrhage and acute meningitis are unlikely in this case. (Choice C) CBC with differential would not assist in the differential diagnosis of this case. (Choice E) Acetaminophen intoxication may initially manifest as nausea and vomiting, but eventually leads to liver damage. The absence of acetaminophen intake in the history also makes the diagnosis unlikely. Educational Objective: Clinical symptoms of CO poisoning include throbbing headache, nausea, malaise and dizziness. Several people simultaneously presenting with a headache is an important clue. The diagnosis is confirmed by carboxyhemoglobin level measurement. Case 63 A randomized, double-blinded clinical trial was conducted to assess the role of multidrug chemotherapy in the treatment of patients with stage III and IV stomach cancer. 150 patients in the treatment group and 100 patients in the placebo group were followed for 24 months. 120 patients in the treatment group (80%) and 80 patients in the placebo group (80%) died during the follow-up period. The investigators conclude that the treatment is effective. Which of the following is the most likely explanation of such a conclusion? A. Observer’s bias may be present B. Selective survival may be an issue C. The results are confounded D. Time-to-event data were analyzed E. Two-year risk was calculated Explanation: Time-to-event data analysis is becoming more popular in the analysis of follow-up studies and clinical trials. This type of analysis is called ‘survival analysis.’ It accounts not only for the number of events in both groups, but also for the timing of the events. (Choice E) Interestingly, patients in the treatment group may live longer than the patients in the placebo group despite the fact that a two-year mortality risk is the same for both groups. For example, the median survival time may be three months for the placebo group and nine months in

the treatment group; therefore, time-to-event analysis in the study presented may explain the conclusion that the treatment group did better. (Choice A) Observer’s bias is reduced by the double-blinding technique. (Choice C) Randomization is an effective tool to control for confounding. (Choice B) Selective survival is not a likely explanation of the conclusion given by the investigators. Educational Objective: Survival analysis is used to analyze follow-up studies and clinical trials. It accounts not only for the number of events in both groups, but also for the timing of the events. Case 64 The following Vignette applies to the next 2 items A 50-year-old male comes in to the emergency department complaining of unremitting pain in an area of his right buttock near the anus. Although he is uncertain when the discomfort first began, he notes that the pain has increased significantly over the past 24 hours. He finds he is unable to pass stool because of the pain. He has not experienced symptoms like this before. Ten years ago, he was diagnosed with diabetes mellitus. He is currently on insulin. He smokes half a pack of cigarettes per day but would like to quit. He denies drinking alcohol or using recreational drugs. His temperature is 38.1C (100.6F), blood pressure is 128/86 mm Hg, and pulse is 90/min. There is a hot, tender, mass located between the anus and the left ischial tuberosity. The mass fluctuates upon palpation.

Item 1 of 2 What is the most appropriate next step in the management of this patient? A. Increase the insulin dose and discharge home B. Prescribe bupropion and discharge home C. Prescribe oral antibiotics and discharge home D. Admit to the hospital and start intravenous antibiotics E. Incise and drain the mass Explanation: Anal abscesses arise when one or more of the several glands that encircle the anus become blocked and the bacteria within grow unchecked. Patients with anal abscesses typically present with severe, constant pain that may be accompanied by fever or malaise. Physical examination commonly reveals erythematous, indurated skin or a fluctuant mass over the perianal or ischiorectal space. Purulent material may be seen if the abscess has begun to drain spontaneously. It is widely agreed that the most important aspect of treatment is prompt incision and drainage of the abscess (Choice E). Perianal and small ischiorectal abscesses are often drained in the office, but larger ischiorectal abscesses typically require surgical intervention. Increasing the insulin dose (Choice A) or prescribing bupropion (Choice B) are inappropriate because they do not treat the abscess. However, strict control of diabetes is very important in reducing the complications associated with wound healing. Oral antibiotics (Choice C) should also be prescribed to those patients who have diabetes mellitus, immunosuppression, extensive cellulitis, or valvular heart disease. However, antibiotics are an adjunct to and not a substitute for incision and drainage of the abscess. Admission to the hospital for intravenous antibiotics (Choice D) is unnecessary in a patient with localized infection.

Educational Objective: Anal abscesses should be treated with prompt incision and drainage. Antibiotics should also be prescribed in those patients who have diabetes mellitus, immunosuppression, extensive cellulitis, or valvular heart disease. Item 2 of 2 The patient is treated appropriately. To minimize risk of complications, he is advised to promptly schedule a follow-up visit with his primary care physician. Patients with anal abscesses are at greatest risk of developing which of the following conditions? A. Soft tissue infection B. Fistula C. Fecal impaction D. Incontinence E. Hemorrhoids Explanation: While an anal abscess is an infection within one or more of the anal spaces, an anal fistula (Choice B) is a tunneling between the anus or rectum and another epithelial lined space (eg, the skin overlying the drainage site). Fifty percent of patients with anal abscesses will go on to develop a chronic fistula from the involved anal gland to the overlying skin. Patients with fistulas typically present with an anal abscess that persists after incision and drainage, or with a pustule-like lesion in the perianal or ischiorectal area that continually drains. Surgical repair is usually necessary to eliminate the fistula while preserving fecal continence. Soft tissue infection (Choice A) is rarely a concern when the initial mass has been properly incised and drained. Fecal impaction (Choice C) could be of concern in a patient with an anal abscess who had been unable to pass stool for a long period before presenting for treatment. It does not commonly occur subsequent to treatment of anal abscess, however. Incontinence (Choice D) and hemorrhoids (Choice E) are not commonly associated with anal abscesses. Educational Objective: Fifty percent of patients with anal abscesses will go on to develop a chronic fistula from the involved anal gland to the overlying skin. Such fistulas require surgical repair. Case 66 The following vignette applies to the next 2 items A 45-year-old Caucasian man is brought to the emergency department by his wife. She states that he has been having fever, loss of appetite, nausea, vomiting, abdominal pain with distention, and hallucinations. He has chronic hepatitis C; he was diagnosed eight years ago. Treatment with ribavirin and pegylated interferon failed two years ago. He does not drink alcohol. His temperature is 38.3 C (101 F), blood pressure is 100/50 mmHg, pulse is 108/min, and respirations are 20/min. He is currently awake, but irritable. Physical examination reveals dry, icteric mucus membranes and clear lung fields. His abdomen is tender and distended; bowel sounds are present; shifting dullness is observed. There is no rebound tenderness and hepatomegaly. Fecal occult blood is negative. There are no focal motor or sensory deficits. Mild intention tremors are present. The patient’s laboratory tests reveal the following: CBC Hb 8.8 g/dL Ht 26% Platelet count 80,000/cmm Leukocyte count 8,000/cmm Segmented neutrophils 80%

Bands 3% Lymphocytes 17% LFT Total bilirubin 5.2 mg/dL Direct bilirubin 2.5 mg/dL Alkaline phosphatase 520 U/L Aspartate aminotransferase 98 U/L Alanine aminotransferase 85 U/L Albumin 2.8 g/dL Coagulation Profile: Prothrombin time 20 sec

PTT 38 sec

A paracentesis is performed. The results are the following: Leukocytes 900/mm3 Neutrophils 65% RBC 3/cmm Item 1 of 2 Which of the following is the most appropriate pharmacotherapy? A. Therapeutic paracentesis B. Blood transfusion and antibiotics C. Mannitol and lactulose D. Peritoneo-venous shunt E. Lactulose and antibiotics Explanation: This patient is developing hepatic encephalopathy (HE). The presence of tremors is suggestive of stage 2 HE. This is most likely due to spontaneous bacterial peritonitis (SBP), which is diagnosed when there are more than 250 neutrophils/mm3 in the ascitic fluid of a patient with nephrotic syndrome or cirrhosis, as in this case. Management involves the administration of lactulose. Cultures are not always positive in this condition; however, antibiotic therapy must not be delayed while the results are pending. (Choice C) Mannitol is used with lactulose only in the more advanced stages of HE (Choices A and D) Repeated paracentesis and peritoneo-venous shunt are treatments for refractory ascites, not for SBP. (Choice B) Blood transfusion is not indicated now, as there is no evidence of ongoing blood loss. Blood transfusion is usually reserved for patients with hemoglobin levels less than 8 g/dL. Educational Objective: SBP is a serious condition and is diagnosed when > 250 neutrophils/mm3 is found in the peritoneal fluid. Empiric antibiotic therapy must be started immediately while waiting for the culture results, especially if there are other clinical indicators of infection. Item 2 of 2 The patient responds well to the treatment. He is currently less agitated and more oriented. He is discharged from the hospital. Four weeks later, he returns with his wife for a follow-up visit. His new laboratory tests show the following: Total bilirubin 2.8 mg/dL Direct bilirubin 1.3 mg/dL Alkaline phosphatase 380 U/L Aspartate aminotransferase 78 U/L Alanine aminotransferase 43 U/L Albumin 2.6 g/dL Prothrombin time 19 sec The patient’s wife is very concerned and asks about his prognosis. Which of the following is the most appropriate response?

A. His prognosis is good B. He has a very high risk of hepatocellular carcinoma C. He needs a liver transplant D. There is nothing much we can do E. We have to observe him for a couple of months for an accurate prognosis Explanation: The patient has possible cirrhosis secondary to chronic hepatitis-C infection. His prothrombin time remains prolonged, hepatic enzymes are mildly elevated and AST is higher than ALT, which points towards profound liver damage. In addition, he has hyperbilirubinemia and hypoalbuminemia. With the laboratory data available, the Child-Pugh score of liver damage can be calculated. Child-Turcotte-Pugh (CTP) score components are used to select the candidates for liver transplantation. Points 1 2 3

Bilirubin (mg/dl) Encephalopathy 3

Ascites

Prothrombin time

Albumin

Absent Slight/responsive

3.5 2.8-3.5

None

17 sec

(g%)

3-5

. For cholestatic disorders the bilirubin categories are < 4, 4-10, and > 10 Class A: CTP score 6 Class B: CTP score 7-9 Class C: CTP score > 10 Liver transplantation should be considered in all cirrhotic patients with CTP > 7 (Choice A) The patient's prognosis is not good. He needs a liver transplant. (Choice E) Because the acute event has been controlled, the score can be calculated and a prognosis given without an observation period. (Choice D) His disease is not terminal at this point. (Choice B) Studies have shown that approximately 7% (over a period of five years) of patients with hepatitis-C and cirrhosis die from hepatocellular cancer. Furthermore, sharing this information rather than advising the patient to undergo a liver transplantation is inappropriate. Educational Objective: A clinical and biochemical evaluation of a patient with cirrhosis and chronic hepatitis is important to determine if he/she is a candidate for surgery. Liver transplantation is a treatment option when the disease has progressed and there is no other treatment available, and when the patient is young and is not currently using alcohol. Case 67 A 23-year-old Caucasian college student comes to the student health center for the evaluation of lowgrade fever, malaise, and dry cough for the past two weeks. She also complains about a recurrent dry and itchy rash over her face and axilla. She has had recent problems with maintaining her weight. She has lost 15 pounds in the past two months, despite her attempts to gain weight to stay in the college weightlifting team. She admits that she has been under a lot of stress recently, and was working three jobs at one time to keep up with her finances. Which of the following is the most important in the medical history, and should be obtained in detail to help diagnose this patient? A. The episodes will diminish with age, but treatment is not usually effective. B. Sexual history C. History of allergies

D. Family history E. We have to observe him for a couple of months for an accurate prognosis Explanation: The index of suspicion for HIV infection should be high in any young patient who presents with nonspecific or vague symptoms and a significant, unintentional weight loss in the past few weeks. This patient should be screened for risk factors for HIV infection, which includes a history of sexual exposure and the use of intravenous drugs in the past. A detailed sexual history must be obtained, and this should include the patient's sexual orientation and past history of unprotected sexual intercourse. The physical examination should also be focused, and the search for clues of HIV infection or Acquired Immunodeficiency Syndrome (AIDS) must be exhaustive. New-onset seborrheic dermatitis (erythematous and pruritic rash on the face and axilla) is commonly seen in HIV-infected patients, and can be one of the presenting complaints of the disease. (Choices A, C, and D) The occupational history, family history, and previous history of allergies are not as important as a detailed sexual history in this patient. Educational Objective: Obtain a detailed sexual history in all young patients who present with weight loss and nonspecific complaints. Case 68 A 4-year-old Caucasian boy is brought to your office by his mother because of frequent staring spells that last about 10-20 seconds. His past medical history is significant for three episodes of otitis media and one episode of severe diarrhea that required hospitalization. The neurologic examination is normal. A staring spell is provoked at the EEG lab, and a generalized 3/second spike and wave activity is observed. No myoclonic activity is noted. The mother is concerned about the child’s condition, and asks you about the treatment and prognosis. Which of the following is the best response to her concern? A. The episodes will diminish with age, but treatment is not usually effective. B. The episodes will diminish with age, and treatment is usually effective. C. The episodes will not change with age, but treatment is usually effective. D. The episodes will not change with age, and treatment is not usually effective. E. The episodes will increase with age, and treatment is not usually effective. Explanation: The clinical scenario described is consistent with childhood absence epilepsy (CAE): age of onset (4-8 years), no neurologic signs, typical EEG pattern, and no myoclonic activity. This condition is usually responsive to ethosuximide or valproate, although high doses may be required to control it effectively. Interestingly, many traditional anti-epileptic drugs are not effective in absence epilepsy, and can even exacerbate the condition (e.g. gabapentin). The prognosis in patients with CAE, especially if generalized tonic-clonic seizures are absent, is good. Staring spells will disappear in the teenage years. The risk of persistence of the condition is higher in patients who develop generalized tonicclonic seizures. Unlike CAE, juvenile myoclonic epilepsy (JME) is characterized by a late onset of absence seizures with myoclonic activity, and is associated with life-long seizures. Educational Objective: The prognosis in patients with childhood absence epilepsy, especially if generalized tonic-clonic seizures are absent, is good. Case 69 A 42-year-old Caucasian woman comes to the emergency department because of right-sided neck pain, persistent cough, and dyspnea for the past three days. Her past medical problems include hypertension and obesity. She takes enalapril and hydrochlorothiazide. She has smoked one pack of cigarettes daily for the past 25 years. She does not drink alcohol. She works as an executive secretary. Her temperature is 37.8C (100 F), blood pressure is 130/80 mm Hg, pulse is 92/min and

respirations are 22/min. Examination shows decreased breath sounds in the upper part of the right lung. There is moderate swelling and erythema of the neck, moderate right jugular vein dilatation, and increased venous marks in the right arm. A chest x-ray reveals a right apical lung mass. Which of the following is the most appropriate immediate step in the management of this patient? A. Start broad-spectrum antibiotic therapy B. Order a neck and chest computerized tomography (CT) scan with contrast C. Prepare for endotracheal intubation D. Order thyroid tests, CBC and chemistry panel, and reevaluate with the results E. Order magnetic resonance imaging (MRI) of the neck and chest Explanation: The patient’s history, physical findings, and radiologic findings suggest superior vena cava syndrome (SVCS) secondary to a lung malignancy. At least 80% of the cases of SVCS are due to bronchogenic carcinoma. The classic presentation of SVCS begins with dyspnea, persistent cough, fascial fullness and neck pain, and progresses into hoarseness, dysphagia, chest pain and syncope. Pertinent physical findings are edema and erythema of the neck (which may sometimes compromise the face), and dilated veins of the arms and neck. Advanced disease is manifested by cyanosis, collateral veins in the thorax, ocular proptosis and lingual edema. The best diagnostic test for SVCS is a contrast CT scan of the chest and neck, which will reveal an obstruction of the superior vena cave due to the pulmonary mass, the metastatic nodes, or an intravenous thrombus. CT scan is very useful because it can reveal the extent of obstruction and provide a histopathologic diagnosis (via percutaneous biopsy), which will determine the particular therapeutic regimen required to manage the underlying malignancy. (Choice E) Magnetic resonance imaging (MRI) is a more expensive procedure, and does not offer any visual advantage over computed tomography. It is only used when a patient is allergic to the contrast dye used in CT scan, or when venous access cannot be obtained for contrast-enhanced studies. (Choice C) Immediate endotracheal intubation may be performed if there exists a risk of sudden airway occlusion, such as in patients with mediastinitis and deep neck infection; however, this patient does not have a history of esophageal disease, upper respiratory tract infection, fever, or chills, to make us suspect mediastinitis. (Choice A) The erythema of the patient is due to venous occlusion and possible thrombosis, not to facial or neck cellulitis. Antibiotic therapy may be subsequently needed if the patient is found to have an infection; however, it is not necessary at this point. (Choice D) Biochemical and hematological tests may be obtained after CT scan evaluation. Educational Objective: Superior vena cava syndrome must be suspected in any patient who has a high risk of malignancy (e.g., family history or smokers) and presents with dyspnea, orthopnea, neck pain and swelling, and has cervical and upper extremities venous dilatation. The best diagnostic test to perform is computerized tomography of the neck and chest. 63% of people answered this question correctly.

Case 70 The following vignette applies to the next 2 items A 24-year-old Italian man is brought to the emergency department by his girlfriend for the evaluation of difficulty in breathing. He has been vacationing in the United States along with his girlfriend for the past two months. His shortness of breath started four days ago and is progressively getting worse. He has to pause several times during your interview to breathe. His girlfriend tells you that he had suffered from an episode of gastroenteritis two weeks ago. He also had bloody bowel moments at that time. The diarrhea resolved spontaneously, but he felt very weak and fatigued afterwards. One week later, he felt a fine tingling sensation in his toes and feet. Three days later, he was extremely weak and could not get up from his bed and started to have some difficulty in breathing. The breathing difficulty got much worse when he woke up this morning, and he decided to come to the emergency room. He has no significant past medical history or family history. He denies any history of intravenous drug abuse, alcohol or tobacco use, or risk factors for HIV disease. His temperature is 37.6C (99.8F), blood pressure is 142/62 mmHg, respirations are 28/min, and heart rate is 96/min. His lungs and cardiovascular examination are unremarkable. His neurological examination reveals a flaccid paralysis of the bilateral lower extremities. Deep tendon reflexes are also absent bilaterally. The rest of the neurological examination, including a detailed cranial nerve examination, is normal. Item 1 of 2 What is the next best step in the management of this patient? A. Start intravenous antibiotics B. Send blood cultures C. Give botulinum antitoxin D. Provide respiratory support E. Obtain stat serum chemistry and electrolytes Explanation: The patient in the above vignette has a classic presentation of acute Guillain-Barre syndrome (GBS), which is an acute inflammatory demyelinating ascending polyneuropathy characterized by progressive flaccid paralysis and areflexia. The pathogenesis is immunologically mediated, with approximately 2/3 of the patients presenting with a history of upper respiratory or gastrointestinal infection in the proceeding 2 to 4 weeks (Campylobacter jejuni, cytomegalovirus, Epstein-Barr virus, or herpes simplex viruses infections). The patient usually presents with progressive lower extremity weakness that may ascend rapidly over days to involve the upper extremities, cranial nerves and eventually, the muscles of respiration. The physical examination reveals bilateral symmetric weakness or flaccid paralysis and absent deep tendon reflexes in the extremities. Approximately 25 to 30% of patients with GBS will eventually develop neuromuscular respiratory failure and need respiratory support with mechanical ventilation. All patients with suspected Guillain-Barre syndrome and clinical signs and symptoms of progressive respiratory compromise should be hospitalized immediately and closely monitored for signs of respiratory failure. This includes continuous pulse oximetry monitoring, bedside spirometry, arterial blood gas measurement and frequent clinical evaluation for signs of respiratory muscle fatigue. Measurements of bedside vital capacity provide the most useful information regarding the degree of respiratory impairment. Respiratory failure is an important and common cause of death in these patients. Urgent intubation and mechanical ventilation should be considered for the patients with rapidly progressive respiratory compromise to decrease the risk of complication and mortality. Most patients with GBS have spontaneous remission; they either recover completely or have only minor neurological deficits. Some of the factors associated with a poor outcome include older age, rapidly progressive disease, prolonged mechanical ventilation, and the presence of associated comorbidity. (Choices A and B) Blood culture and intravenous antibiotics have no role in the management of patients with Guillain-Barre syndrome.

(Choice C) Therapy with botulinum antitoxin is indicated in patients with progressive symptoms of suspected botulism. These patients classically present with an acute onset of bilateral cranial nerve neuropathies with symmetric descending flaccid paralysis. (Choice E) There is no indication for obtaining urgent electrolytes and serum chemistry at this point. Educational Objective: Campylobacter jejuni infection is the most frequent precipitant of Guillain-Barre syndrome (GBS). All patients with suspected GBS should be hospitalized, regardless of the severity of the illness. Measurements of bedside vital capacity provide the most useful information regarding the degree of respiratory impairment. Patients with signs of respiratory failure should be promptly intubated and supported with mechanical ventilation until the resolution of respiratory muscle weakness. Item 2 of 2 What is the therapy of choice for the patient's condition? A. Plasmapheresis B. High dose intravenous steroids with a gradual taper C. Antitoxin alone is enough D. Antitoxin and immunoglobulin E. Clindamycin and Vancomycin Explanation: The first step in the management of patients with Guillain-Barre syndrome (GBS) is to triage the severity and provide appropriate respiratory support for patients with signs of respiratory failure. After the airway is secured, specific therapy should be instituted as early as possible. Plasmapheresis and/or administration of intravenous immunoglobulin (IVIG) are the mainstays of therapy for patients with severe and progressive GBS. The indications for plasmapheresis are: severe flaccid paralysis, bulbar palsy, progressive respiratory failure, and patients on mechanical ventilation. It is most effective when it is started within seven days of the onset of symptoms; however, some improvement still occurs even when it started late in the course of disease. Plasmapheresis is not usually indicated for ambulatory patients with mild or non-progressive disease. Daily administration of IVIG for two weeks may also be used to treat patients with GBS because it has been shown to be as effective as plasmapheresis. (Choices B and E) Antibiotics and corticosteroids have no role in the management of patients with GBS. (Choices C and D) Botulinum antitoxin is extremely useful in patients with progressive weakness and suspected botulism. It has no role in the management of patients with GBS. Educational Objective: Plasmapheresis or intravenous immunoglobulin is the mainstay of therapy for patients with severe and progressive GBS. Corticosteroids have no role in the management of patients with GBS 72% of people answered this question correctly. Case 71 The following vignette applies to the next 2 items A follow-up study was conducted to assess the role of different treatment regimens on the risk of cardiovascular outcomes. As a subset of the study, two treatment regimens were evaluated: highdose hydrochlorothiazide (100 mg) and low-dose hydrochlorothiazide (25 mg). According to the study results, the mean systolic blood pressure (BP) in the high-dose group was 139 mmHg, and the mean diastolic BP was 88 mmHg; in the low-dose group, the mean values were 143 mmHg and 92 mmHg, respectively. A two-sample t test gave a p value of 0.03 for systolic BP and 0.04 for diastolic blood

pressure differences between the two groups. The relative risk (RR) of sudden cardiac death comparing the low-dose group to the high-dose group was 0.4 (95% Confidence Interval 0.25 - 0.55) Item 1 of 2 Which of the following is the best statement concerning the results of the study? A. High-dose hydrochlorothiazide has no additional benefit in lowering BP. B. Low-dose hydrochlorothiazide reduces the risk of sudden death in the cohort by 40%. C. High-dose hydrochlorothiazide doubles the risk of sudden death in the cohort. D. The risk of sudden death decreases by 60% with a lower dose of hydrochlorothiazide. E. The dose of the drug is not significantly related to the risk of sudden death. Explanation: The scenario described emphasizes the pitfalls in interpreting the measures of effect. Be very careful in determining which groups are being compared, as the resulting interpretation will differ strikingly. Relative risk is a measure of effect that can be calculated in follow-up studies. It compares the risks between two groups; in this case, a low-dose group is compared to a high-dose group. Note that in this computation, the high-dose hydrochlorothiazide group is set as a reference group! Compared to high-dose hydrochlorothiazide treatment, low-dose hydrochlorothiazide treatment appears to protect from the occurrence of sudden death. The relative risk of 0.4 therefore indicates a protective effect of a lower dose of the drug, which reduced the risk of sudden death by 60%. (Choice E) The effect is statistically significant because the confidence interval does not contain the null value (1.0). (Choices B and C) The results presented have no information on the ‘baseline’ risk of sudden death in the cohort; therefore, no conclusions about risk reduction in the cohort (no treatment) would be valid. (Choice A) High-dose hydrochlorothiazide treatment seems to be more effective in reducing BP than the low-dose regimen (Note the p values for the two-sample t test). Educational Objective: During the interpretation of the measures of effect, be careful in determining which groups are being compared, as the resulting interpretation will differ strikingly. Item 2 of 2 Another subset of patients in this cohort were given placebo, and their risk of sudden death was assessed. This group's RR of sudden death is very close to 1.0 when compared to the low-dose hydrochlorothiazide group, although the mean systolic and diastolic BP are higher when compared to the low-dose hydrochlorothiazide group. Which of the following is the best statement concerning the risk of sudden death in this cohort? A. The risk of sudden death in the cohort is high. B. High-dose hydrochlorothiazide increases the risk of sudden death approximately 2.5 times. C. High-dose hydrochlorothiazide increases the risk of sudden death by 60%. D. Low-dose hydrochlorothiazide decreases the risk of sudden death almost 3 times. E. Hydrochlorothiazide is a safe drug regardless of the dose. Explanation: The scenario assesses your understanding of risk and relative risk (RR) concepts. It mentions that the RR of sudden death when comparing low-dose hydrochlorothiazide treatment to placebo is close to 1 (null value). This means that the patients taking a low dose of the drug have an almost equal risk of sudden death with the patients taking placebo. On the other hand, the RR of sudden death in the lowdose drug group is 0.4, when compared to the high-dose group. To switch the groups when computing for the RR, the inverse of the estimate is obtained. For instance, the risk of sudden death for the highdose group is 2.5 times higher, when compared to the low-dose group and placebo group (2.5 is the inverse value of 0.4).

RR (low-dose group vs. high-dose group) = 0.4 RR (high-dose group vs. low-dose group) = 1/0.4 = 2.5 RR (low-dose group vs. placebo) = 1.0 Summarizing the results of the study, high-dose hydrochlorothiazide treatment seems to increase the risk of sudden death by 2.5 times, when compared to low-dose treatment and placebo. Educational Objective: Switching the groups when computing for the RR means obtaining the inverse of the estimate

Case 72 A 54-year-old Caucasian male is hospitalized for an episode of life-threatening angioedema. He was treated with corticosteroids and fresh frozen plasma infusion. He never had such an episode before. His past medical history is significant for diabetes diagnosed two years ago and hypertension diagnosed two weeks ago. His family history is significant for diabetes mellitus and stroke in his mother, and prostate cancer in his father. His blood pressure is 150/95 mmHg and heart rate is 80/min. Which of the following is the best treatment option for hypertension in this patient? A. Captopril B. Clonidine C. Terazosin D. Losartan E. Metoprolol Explanation: The episode of angioedema is most likely related to the treatment of recently diagnosed hypertension with an ACE inhibitor. Although the exact mechanism of ACE inhibitor-induced angioedema is unclear, it is likely that both kinin-dependent and kinin-independent mechanisms have a role. The incidence of this complication is 0.2 to 0.7%. Selective beta-blockers have been shown to be well-tolerated and equally beneficial in diabetic patients as in non-diabetics; therefore, metoprolol is the most appropriate treatment option in this patient. (Choice A) Re-trial of an ACE inhibitor is not recommended, although switching to an angiotensin receptor blocker (ARB) may be considered in selected patients. (Choice D) Many cases of angioedema have been described with ARBs including losartan, although the incidence is much lower than with ACE inhibitors. This patient experienced a life-threatening episode of angioedema, and has no compelling indication for ARB therapy (e.g., heart failure, diabetic nephropathy). (Choices B and C) Clonidine and terazosin are not considered first-line agents for the treatment of hypertension. Educational Objective: ARBs can be used very cautiously in selected patients with ACE inhibitor-induced angioedema; however, it is best to use alternative drugs, if possible.

15% of people answered this question correctly. Case 73 The following vignette applies to the next 4 items A 24-year-old female is admitted to the hospital after a motor vehicle accident, where she sustained a right humerus fracture, multiple rib fractures and a contusion over her right lower leg. The next morning, a nurse calls to inform you that she is complaining of severe pain in her right lower leg. Her temperature is 37.2 C (99 F), heart rate is 96/min, respiratory rate is 18/min, and blood pressure is 140/82 mmHg. Physical examination reveals a tense swelling around the right calf region. The pain is worsened on palpation and passive movements of the foot. Neurological examination reveals motor weakness and hypoesthesia of the distal right leg. Item 1 of 4 Which of the following is the most likely diagnosis? A. Fat embolism B. Deep venous thrombosis C. Acute compartment syndrome D. Acute vascular occlusion E. Neural compression Explanation: Acute compartment syndrome refers to ischemic tissue damage secondary to elevated pressures in the enclosed compartments of the lower legs or forearm. When the tissue pressure in an enclosed compartment exceeds the perfusion pressure, the resulting diminished tissue perfusion and compromised blood flow to the muscles and nerves inevitably lead to ischemic tissue necrosis. Majority of the cases involving the lower extremities are due to a traumatic event, most commonly tibial fractures. Other causes include a crush injury or other long bone fractures in a motor vehicle accident, a tight cast or dressing after trauma, and drug overdose. Patients usually present with severe pain which is out of proportion to the extent of injury. The pain is typically worsened by passive movements of the involved muscles. Sensory nerves are usually affected earlier than the motor nerves, and the neurologic deficit presents as decreased vibration sense, decreased two-point discrimination, numbness or hypoesthesia. Late features include extremity paralysis and absent distal pulsation (pulseless paralysis). (Choice A) Fat embolism is infrequently seen in patients with long bone or pelvic fractures. Patients usually present with a triad of hypoxemia, neurological abnormalities (i.e., confusion), and a petechial rash (involving the head, neck, anterior chest wall, or axilla). (Choice B) Deep venous thrombosis usually does not present acutely in a setting of motor vehicle accident in an otherwise healthy young patient. It is a rare cause of compartment syndrome and does not compromise blood circulation and neuromuscular function. (Choice D) Vascular occlusion secondary to a motor vehicle accident usually presents more suddenly and dramatically. Acute compartment syndrome usually has a lag period of a few hours before irreversible nerve injury and muscle necrosis occurs. (Choice E) Nerve compression may occur in a patient after a motor vehicle accident and possible bone fracture; however, it does not lead to blood flow compromise and muscle necrosis (pain with passive movements of the involved muscles). Educational Objective: Acute compartment syndrome usually occurs after a traumatic event and causes pain, paresis, hypoesthesia, and diminished to absent pulses in the involved limb. 87% of people answered this question correctly.

Item 2 of 4 Which of the following is the most common life-threatening complication of the above condition? A. Disseminated intravascular coagulation B. Rhabdomyolysis and renal failure C. Pulmonary embolism D. Gangrene of the limb E. Thrombocytopenia Explanation: Acute compartment syndrome results in markedly diminished to absent tissue perfusion within hours of the inciting event, causing tissue necrosis, muscle infarction, and rhabdomyolysis, which releases myoglobin into the peripheral circulation. Myoglobin is directly toxic to the renal tubules, and subsequently causes acute tubular necrosis and acute renal failure. Acute renal failure and its complications (electrolyte disturbances) are one of the most common life-threatening complications of acute compartment syndrome. Laboratory studies typically reveal markedly elevated creatinine kinase levels and the presence of myoglobin in the urine (positive dipstick for blood in the absence of RBC’s in the urine). (Choices A, C, D, E) Thrombocytopenia, disseminated intravascular coagulation, pulmonary embolism, and gangrene of the limb are not usually seen in patients with acute compartment syndrome.

Educational Objective: Rhabdomyolysis and subsequent development of acute renal failure is one of the most common and severe life-threatening complications of acute compartment syndrome. 24% of people answered this question correctly. Item 3 of 4 Which of the following is the most appropriate next step in the management of this patient? A. Administer oxygen B. Start the patient on anticoagulation C. Order venous Doppler ultrasonography D. Check the tissue pressure E. Order a nerve conduction study Explanation: Compartment syndrome is characterized by an increase in the tissue pressure in the enclosed myofascial compartments of the extremities. When the elevated tissue or compartment pressure reaches its threshold level, the capillaries collapse, and this eventually leads to tissue and muscle necrosis. The exact value for the tissue pressure at which blood flows to the muscle and nerve tissue stops is controversial. The current general consensus for the threshold value is greater than 30 mmHg. It is therefore important to measure the tissue or compartment pressure early in the course of management, especially if the diagnosis is in question. (Choice A) Oxygen is usually used in the supportive treatment of patients with fat or cholesterol emboli. (Choice B) Anticoagulation is usually required for patients with deep venous thrombosis. (Choice C) Venous Doppler ultrasonography is useful for the diagnosis of deep venous thrombosis. It is not helpful for the diagnosis of compartment syndrome. (Choice E) Nerve conduction studies are helpful in the diagnosis and localization of the site of nerve damage.

Educational Objective: Direct measurement of the compartment or tissue pressure is the diagnostic procedure of choice for patients with suspected acute compartment syndrome. 80% of people answered this question correctly. Item 4 of 4 The appropriate step was taken for the patient. Which of the following is the best next step in management? A. Continue with oxygen therapy B. Obtain a hypercoagulable panel C. Consult a vascular surgeon D. Perform urgent fasciotomy E. Review the results of venous Doppler ultrasonography Explanation: Acute compartment syndrome is a surgical emergency. Any delay in treatment leads to irreversible muscle and nerve damage. A compartment pressure of 30 mmHg or greater warrants an emergent fasciotomy (also known as compartment release). Surgical decompression aims to relieve the pressure within the enclosed compartment and to restore the blood flow to muscles and other tissues within 6-10 hours of the initial symptoms. Some patients may develop a persistent sensory or motor deficit after an episode of acute compartment syndrome despite early fasciotomy. (Choice A) Oxygen therapy has no role in the management of patients with acute compartment syndrome. (Choice B) Obtaining a hypercoagulable panel may be useful in patients with idiopathic deep venous thrombosis. It is not indicated in patients with acute compartment syndrome. (Choice C) A consultation with a vascular surgeon is necessary in patients with vascular injury or occlusion. (Choice E) Venous Doppler ultrasonography is useful to diagnose deep venous thrombosis. It has no role in the diagnosis or management of patients with acute compartment syndrome. Educational Objective: An emergent fasciotomy is the definitive treatment in patients with acute compartment syndrome. Surgical decompression aims to restore the capillary blood flow and tissue perfusion. Case 74 The following vignette applies to the next 3 items A six-month-old well-nourished African-American boy is brought to the pediatrician by his mother. She is concerned because on multiple occasions in the past 48 hours, she has discovered evidence of rectal bleeding in her son’s diaper. The mother reports that her son has no history of vomiting, excessive crying, or demonstration of discomfort when passing stool. She denies any history of trauma or foreign body ingestion. The boy was born at term via normal spontaneous vaginal delivery and has no pertinent medical history. He is now sitting very quietly in his mother’s lap, and appears to be experiencing mild discomfort. His temperature is 37.0C (98.6F), blood pressure is 84/56 mm Hg, pulse is 132/min, and respirations are 28/min. His anterior fontanelle is slightly depressed. His abdomen is soft and nontender. There is no hepatosplenomegaly. The rectal examination is unremarkable. Immediately after being examined, the child passes stool that looks like currant jelly, and contains gross amounts of blood and mucous. Item 1 of 3

Which of the following diagnostic tests would be most useful in the management of this patient’s condition? A. Abdominal plain films B. Barium enema C. CT scan of abdomen D. Technetium 99m scan E. Ultrasound of abdomen Explanation: The second most common cause of small bowel obstruction in infants and toddlers, intussusception is the telescoping of a proximal portion of intestine into a more distal portion of intestine. The resulting venous and lymphatic congestion can cause significant intestinal edema and may ultimately result in ischemia, perforation, and peritonitis. The most common presentation involves intermittent, severe, crampy abdominal pain accompanied by intense crying and vomiting. A sausage-shaped mass may be palpated on the right side of the abdomen. Interestingly, the classic triad of pain, a palpable mass, and "currant-jelly" stool (containing blood and mucous) is observed in less than 15% of patients, and up to 20% of patients may have no evident pain. The diagnosis is most commonly confirmed with a barium enema contrast study. This study is preferred because it is both diagnostic and therapeutic. Reduction of the intussusception through this method is successful 75-90% of the time. (Choice A) Abdominal plain films occasionally demonstrate a soft-tissue density extending into the gaseous pockets within the large bowel, a finding known as the "crescent sign." Frank intestinal obstruction and pneumoperitoneum may also be found. (Choice C) CT scans of the abdomen may allow for the diagnosis of intussusception, but are less preferred because they require sedation of young children and are time-consuming. (Choice D) A Meckel scan with technetium 99m is often used to highlight ectopic gastric mucosa. It is not commonly used in the diagnosis of intussusception. (Choice E) Ultrasonography is also an excellent choice as its sensitivity and specificity near 100%. The classic image seen on ultrasound is that of a "bull?s eye," signifying layers of intestine nestled within intestine. Although there have been some reports of reducing the intussusception under ultrasound guidance, this is an unusual and not well-studied technique; therefore, barium enema remains as the preferred means of diagnosis and therapy. Educational Objective: The barium enema (not barium swallow) contrast is a preferred means of diagnosing intussusception because it is also therapeutic; reduction of the intussusception through this method is successful 7590% of the time. Ultrasonography is also an excellent choice as its sensitivity and specificity near 100%. * If you don’t find barium enema in the choices choose ultrasonography. *Extremely important question for USMLE step-3 Item 2 of 3 The appropriate diagnostic test is selected and the mother agrees to the proposed treatment. The boy’s father is not present. Which of the following is the most appropriate next step? A. Document the mother’s informed consent and proceed with diagnostic testing and treatment B. Have the father brought in for written consent C. Have the father telephoned for oral consent D. Have another adult relative brought in for written consent E. Request a nurse to witness the informed consent

Explanation: Obtaining informed consent formally ensures that the patient is an informed participant in his health care decisions. To properly equip the patient with adequate knowledge to make such decisions, several concepts must be conveyed, including the nature, alternatives, risks, and benefits of the procedure. For a patient’s consent to be deemed valid, he must be considered competent, and his consent must be voluntary. The same standards apply if the patient is a minor, with the exception that the custodial parent or legal guardian retains the legal authority to provide consent. It is generally considered sufficient to document the informed consent of one parent and then proceed with diagnostic testing and treatment. (Choices B and C) It is not necessary to contact the child’s father for written or oral consent, unless the mother does not have the legal authority to act on the child’s behalf. (Choice D) Contacting another adult relative to provide consent is only appropriate if that individual has the legal authority to act on the child’s behalf while the mother does not. (Choice E) If the procedure is a simple one (eg, a blood draw), it may be appropriate to have the nurse obtain informed consent from the patient’s mother. However, more complex procedures warrant an in-depth discussion between the physician and patient. Once such a discussion occurs, the physician should document the consent and include it in the medical chart. It is rarely necessary to have support staff witness the informed consent if the physician is already present. Educational Objective: When providing health care to minors, it is generally sufficient to document the informed consent of one parent who has legal custody of the child and then proceed with diagnostic testing and treatment. 88% of people answered this question correctly. Item 3 of 3 The risks and benefits of the preferred curative procedure are discussed, and the child’s mother asks about potential complications. Which of the following is most likely? A. Adhesions B. Fistula C. Perforation D. Renal failure E. Stricture Explanation: The complication of greatest concern when reducing an intussusception with a barium or air contrast study is perforation of the bowel, which occurs in 10 WBCs on microscopy of the first voided urine. The diagnosis is confirmed with a Gram stain smear and culture of a urethral swab specimen. All the patients diagnosed with gonococcal urethritis should also be treated empirically with an agent active against C. trachomatis, such as doxycycline 100 mg orally twice a day for 7 days, or azithromycin 1 gm orally as a single dose. (Choices A, B, and E) Ceftriaxone (125 mg intramuscularly) is one of the preferred and most commonly used agents against gonococcal infection. Neisseria gonorrhoeae is extremely susceptible to broad-spectrum cephalosporins, including ceftriaxone. The current recommended dose is 125 mg intramuscularly, which is effective in more than 99% of the cases. (Choice C) Repeat exposure to N. gonorrhoeae is possible, but it is too early to have symptomatic disease from reexposure after adequate treatment. Coinfection with Chlamydia is more likely. Educational Objective: All the patients diagnosed with gonococcal urethritis should also be treated empirically with an agent active against C. trachomatis, such as doxycycline 100 mg orally twice a day for 7 days, or azithromycin 1 gm orally as a single dose. 75% of people answered this question correctly. Case 94 Two Caucasian children are brought to the physician by their mother. The older child is an eight-yearold boy and the younger is his 6-year-old sister. Ever since the family dog died, the boy has been crying uncontrollably, while his sister looks calm and believes that everything will be fine. Both children do not have any medical problems, and are not on any medication. Their developmental and growth milestones are normal. What is the best way to approach this situation? A. Refer the 8-year-old child to psychotherapy B. Refer the 6-year-old child to psychotherapy C. Refer both children to psychotherapy D. Start both children on sedative medications E. Both children are normal Explanation: Bereavement in children can present in different ways, according to the child’s age. Children who are between 3 and 7 years old usually react to the loss of a loved person or pet with disbelief. At this age, these children think that death is only temporary (e.g., sometimes they think that the dead person or animal is only sleeping and will later wake up or come back to life). This reaction usually lasts for some days, after which the child gradually accepts the fact that death is not reversible. On the other hand, older children (greater than 7 years) are already aware that death is final. They can react in different manners: with sadness, uncontrollable tearing, inability to concentrate, rage, guilt, nightmares, regressive behavior or social isolation. (Choices A, B, and C) Each of the two siblings has reacted appropriately, according to their ages, and do not need to be referred to psychotherapy. (Choice D) Sedative medications are indicated only for those children who develop unmanageable hysteric crisis or panic attacks. Educational Objective: Bereavement in children can present in different ways, according to the child’s age. Those who are younger than 7 years tend to react with disbelief, and those younger than 5 years have magical

thoughts about death and can feel guilty or responsible. Children older than 7 years accept death as final and can experience depression, anxiety or have regressive behavior. Case 95 A father brings his 4-year-old girl to the emergency department of the local children’s hospital because she has developed scattered ecchymoses and petechiae all over her body within the last few hours. Laboratory evaluation reveals a platelet count of 4,000/?L. Her hemoglobin and WBC counts are normal. The father believes he had a similar episode as a child. What is the best means of managing this girl?s condition? A. Emergency splenectomy B. Administration of prednisone C. Administration of packed red blood cells D. Administration of platelets E. Administration of cryoprecipitate Explanation: Platelet-specific autoantibodies are the presumed pathogenesis of idiopathic thrombocytopenic purpura. In children, the condition is typically characterized by a sudden onset of bleeding, manifested as petechiae, purpura, epistaxis, and gingival bleeding. More severe bleeding is rare. Commonly, there is a history of infection in the several weeks prior to presentation. Symptomatic patients with moderate to severe thrombocytopenia ( 55 years 2. WBC count > 16,000/cmm 3. LDH levels > 350 IU/L 4. Glucose levels > 200 mg/dl 5. AST > 250 IU/L B. After 48 hours: 1. Po2 < 60 mmHg 2. Calcium < 8 mg/dl 3. BUN increase > 5 mg/dl 4. Hematocrit decrease > 10% 5. Albumin < 3.2 mg/dl 6. Estimated fluid deficit > 4 L The presence of three or more of these criteria is associated with a grave prognosis in acute pancreatitis.

Another scoring system entitled "Acute Physiology and Chronic Health Evaluation (APACHE II) score" is gaining popularity. The APACHE II score is based on the evaluation of clinical data (i.e., blood pressure, pulse, temperature), biochemical data (i.e., urea and electrolytes), and renal and pulmonary function parameters. (Choices A, B, D and E) These are important predictors of prognosis after the first 48 hours of acute pancreatitis. Educational Objective: Know the Ranson?s criteria used to assess the prognosis in patients with acute pancreatitis. 42% of people answered this question correctly. A healthy, 29-year-old African-American woman comes to the physician for a routine health maintenance examination. She has a history of migraine headaches. She has been smoking one pack of cigarettes daily for the last three years. She drinks alcohol only on weekends. Her father died of colon cancer at the age of 50, and her mother is diabetic. She is currently taking no medications. Her vital signs are within normal limits. Examination shows no abnormalities. Her annual pap smear shows high-grade squamous intraepithelial lesion (LSIL). Satisfactory colposcopy examination confirms CIN II. Which of the following is the most appropriate next step in the management of this patient? A. Expectant management B. Cryosurgery C. Laser ablation D. Cold knife conization E. LEEP Explanation: High-grade squamous intraepithelial lesions include CIN II, CIN III, moderate and severe dysplasia, and carcinoma in situ. High-grade squamous intraepithelial lesions are more likely to be progressive than low-grade lesions, and these should always be treated with ablation or excision. Ablation can be done using cryosurgery or laser, and excision can be done using knife conization, laser conization or Loop Electrosurgical Excision Procedure (LEEP). Loop Electrosurgical Excision Procedure (LEEP) is the treatment of choice for high-grade squamous intraepithelial lesion. LEEP is preferred because of its low cost, accuracy, and easiness to perform. It is a very successful procedure and can be performed in an office setting. (Choice A) Expectant management is not the appropriate choice for high-grade lesions. (Choices B, C and D) Ablation is done when a histologic diagnosis is accurate, when there is no evidence of invasion, and glandular lesion, and when colposcopy is satisfactory. Otherwise, excision is preferred. Educational Objective: LEEP is the treatment of choice for high-grade squamous intraepithelial lesions. 50% of people answered this question correctly. A 30-year-old Caucasian man with no significant past medical history presents to his primary care physician complaining of nausea, vomiting, diarrhea, abdominal cramping, and fever to 38.3C (100.9F). The symptoms started 12 hours after the man consumed a beverage that contained pureed fruit, powdered protein, and three raw eggs. Physical examination of the man is unremarkable. Salmonella enteritidis is subsequently isolated from his stool culture. What is the best means of managing this patient?s care? A. Treatment with ampicillin

B. Treatment with ciprofloxacin C. Treatment with trimethoprim/sulfamethoxazole D. Treatment with ceftriaxone E. Supportive therapy and observation Explanation: Symptomatic individuals found to be infected with Salmonella enteritidis should be given replacement fluid and electrolytes, as they are at risk for becoming dehydrated. Since the gastroenteritis is usually self-limited and antibiotic use has not been shown to hasten the resolution of symptoms or improve the rate of Salmonella clearance from stool, antibiotic usage is not recommended for immunocompetent adults or children older than one year of age. Therefore, supportive therapy and observation (Choice E) would be most appropriate. In the United States, Salmonella enteritidis is becoming increasingly resistant to ampicillin (Choice A). Even if antibiotic therapy were indicated, ampicillin would be a poor choice. Preemptive therapy is warranted in patient groups at greater risk for complications. These groups include children younger than twelve months of age (with special attention paid to neonates) and immunocompromised adults. Therapy may be considered for adults at least 50 years old with known atherosclerotic disease, as they are more prone to developing bacteremia and endovascular infection. Effective antibiotics include ciprofloxacin (Choice B), trimethoprim/sulfamethoxazole (Choice C), and ceftriaxone (Choice D). The latter antibiotic is only available intravenously, so its usage is restricted. Educational Objective: Salmonellosis does not need to be treated with antibiotics in immunocompetent individuals age 12 months or older. 48% of people answered this question correctly. A 22-year-old Caucasian female is hospitalized after a car accident. She sustained a hip fracture, fracture of several ribs, and a blunt abdominal injury that required a laparotomy. The laparotomy revealed a liver laceration and extensive hemoperitoneum. In the early postoperative period, the patient is noted to have hyperactive deep tendon reflexes. Which of the following electrolyte abnormalities may be responsible for this condition? A. Hypokalemia B. Hyperkalemia C. Hyponatremia D. Hypocalcemia E. Hypermagnesemia Explanation: Hypocalcemia is the most probable diagnosis in this patient. Hypocalcemia can occur during or immediately after surgery, especially in patients undergoing major surgery and requiring extensive transfusions. Usually, hypocalcemia occurs due to volume expansion and hypoalbuminemia, and is therefore asymptomatic; however, sometimes it may manifest as hyperactive deep tendon reflexes, muscle cramps and, rarely, convulsions. Hypomagnesemia may mimic hypocalcemia, but is associated with alcoholism, prolonged nasogastric suction or diarrhea, and diuretic use. (Choice E) Mild hypermagnesemia results in decreased deep tendon reflexes. A severe form causes loss of the deep tendon reflexes and muscle paralysis, thereby leading to flaccid quadriplegia, decreased respiration, and eventual apnea.

(Choice B) Hyperkalemia typically results in gastrointestinal disturbances (nausea, vomiting), ECG changes, and asystole, if severe. It may be associated with severe burns, crush injuries, and renal insufficiency. (Choices A and C) Hypokalemia and hyponatremia are unlikely to manifest as hyperactive deep tendon reflexes. Educational Objective: Hypocalcemia can occur during or immediately after surgery, especially in patients undergoing major surgery and requiring extensive transfusions. Hyperactive deep tendon reflexes may be the initial manifestation. Hypermagnesemia, on the other hand, results in loss of the deep tendon reflexes. 45% of people answered this question correctly. An 18-year-old high school sophomore is brought to the emergency room after she was involved in a motor vehicle accident. Upon her arrival to the emergency room, she is minimally responsive, and there is evidence of bleeding from the right side of her head. Her vital signs are as follows: temperature 37.6?C (99.6F), blood pressure 182/98 mmHg, heart rate 52 per minute and respiratory rate 6 per minute. Her oxygen saturation is 96% on two liters of oxygen. Her breath smells of alcohol. Physical examination reveals a right pupillary size of 7 mm with minimal response to light. The left pupil is 3 mm with normal pupillary light reflex. The right eye is deviated outwards and downwards. There is bilateral papilledema on funduscopic examination. While in the ER, the patient had an episode of generalized body extension, with adduction and pronation of her upper extremities. There is no evidence of any thoracic or abdominal injuries. Which of the following is the most appropriate next step in the management of this patient? A. Intravenous thiamine B. CT scan of the brain C. Intravenous mannitol D. Hyperventilation to achieve a PaCO2 of 25-30 mmHg E. Endotracheal intubation Explanation: The above patient has the typical clinical features of elevated intracranial pressure (ICP), most likely secondary to an intracranial hemorrhage from head trauma sustained in the motor vehicle accident. Other causes of elevated intracranial pressure include an intracranial hemorrhage from ruptured aneurysms, bleeding AV malformations, CNS infections, CNS neoplasm, and hydrocephalus. Patients with intracranial hypertension have been classically described to have bradycardia, hypertension and respiratory depression (Cushing's triad). In the early stages, patients complain of headaches, vomiting, blurred vision, and have papilledema on funduscopic examination. Further elevation in the intracranial pressure leads to transtentorial herniation of brain tissue, causing altered levels of consciousness (stupor progressing to coma), dilation of the ipsilateral pupil, third cranial nerve palsy, hemiparesis, decerebrate posturing, and eventually, respiratory arrest. The patient in the above vignette has signs of marked respiratory depression and intracranial hypertension. The most important next step in the management is to rapidly intubate the patient to protect and maintain her airway, in case of a respiratory arrest. (Choice A) Intravenous thiamine is useful in alcoholic patients with severe malnutrition to prevent the development of Wernicke's encephalopathy. It has no role in the lowering of an elevated intracranial pressure. (Choice B) CT scan of the brain should be done urgently to look for an intracranial hemorrhage or any other potentially reversible causes of elevated intracranial pressure; however, the patient?s airway should be secured before sending her for neuroimaging.

(Choice C) Intravenous mannitol is an osmotic diuretic which reduces brain volume by drawing water out of the cells. It can be used to acutely lower elevated intracranial pressures in emergent situations; however, securing and maintaining a patent airway should still remain the first priority in the management of such patients. (Choice D) Lowering the PaCO2 by hyperventilating the patient causes cerebral vasoconstriction, which decreases the cerebral blood flow, and reduces intracranial pressure. Hyperventilation is therefore used to decrease the PaCO2 to 25-30 mmHg and to rapidly lower intracranial pressure in emergent situations; however, it is contraindicated in patients with traumatic brain injury and an acute stroke. This is due to the fact that a significant decrease in cerebral blood flow caused by vasoconstriction can lead to worsening of neurological injury in these patients. Educational Objective: Securing and maintaining a patent airway is the first priority in the management of trauma patients with symptomatic intracranial hypertension. 73% of people answered this question correctly. A 41-year-old woman comes to see you in the office for the first time. She has recently moved to your town and wants to establish her medical care with a primary care physician. She is also requesting for an influenza vaccination for this year. She tells you that she had a "really bad" episode of flu last year and had to take a week off from work. She denies any active medical problems. She does not have a history of any chronic medical illness. She smokes approximately one pack of cigarettes a week and drinks alcohol occasionally. Her mother and father both had diabetes mellitus; her father also had significant coronary artery disease. She works as a secretary in the local law firm. She is married and has two sons, who are 8 and 11 years old. Her husband and sons are all in good health, and do not have any medical problems. Which of the following is the most appropriate response to this patient's request for vaccination?

A. You should be fine because you have antibodies from last year's infection. B. You do not have any risk factors, so vaccination is not advisable. C. You should wait for the influenza epidemic to start, and then return for vaccination. D. I agree. Lets give you the vaccine. E. You have two young kids at home. You should get the vaccine now and every year. Explanation: Influenza is an acute viral respiratory illness caused by influenza A and B virus. It occurs in epidemics nearly every year, usually in the winter season. It is responsible for a substantial morbidity in the general population. Mortality occurs due to an associated influenza pneumonia and occurs mainly in patients with chronic underlying cardiac, pulmonary, and other medical illnesses. Vaccination of the high-risk groups is the major means of preventing influenza and its associated mortality. The United States Preventive Health Services and Center for Disease Control and Prevention (CDC) have devised guidelines for the vaccination of people who are at high risk of developing complications from influenza. Certain groups of people at high risk include the following: Persons 50 years of age or older (age limit has been increased to 65 or older for 2004-2005 season secondary to vaccine shortage). All adults and children with a chronic health condition (heart disease, diabetes, kidney disease, asthma, COPD, HIV/AIDS, and malignancy, etc). All residents of nursing homes and long-term care facilities. Children between 6 and 23 months of age. Children and young kids 6 months to 18 years of age on long-term aspirin treatment. Women who will be in second or third trimester of pregnancy in the influenza season. Healthcare workers or workers at long-term care facilities who may transmit influenza to high-risk persons. Household members of persons at high risk of developing influenza and its complications. Out-of-home caregivers and household contacts of children less than 6 months of age. The woman in the above vignette does not have any risk factors or any high-risk contacts at home or work. Although she may contract influenza, she is unlikely to have the associated complications; therefore, it is not advisable to vaccinate her at this point. (Choice A) The influenza virus (influenza A virus in particular) undergoes periodic changes in its antigenic characteristics; therefore, prior infection or vaccinations are not helpful in preventing future infections. (Choices C and D) Vaccination of a patient without any known risk factors is not recommended. (Choice E) Household members of children or adults with chronic medical illnesses should receive an influenza vaccine annually. Since this patient?s husband and two sons do not have any chronic medical illness, vaccination is not recommended. Educational Objective: Influenza vaccination is generally recommended only for those people who are at high risk for influenza-related complications. 38% of people answered this question correctly. A 72-year-old, male, nursing home patient is admitted to the hospital with a three-day history of nausea, vomiting, diarrhea, and confusion. The nurses at the nursing home tell you that he has been drinking a lot of water for the past three days. He has a history of tobacco abuse, diabetes mellitus, hypertension, coronary artery disease, multiple myeloma, and schizophrenia. His medications include aspirin, insulin, metoprolol, and chlorpromazine. His laboratory findings in the hospital reveal a serum sodium level of 120 mEq/L, urine sodium concentration of 80 mEq/L, serum osmolality of 258 milliosmoles/kg, and urine osmolality of 400 milliosmoles/kg. A chest x-ray showed a 3 cm mass

around the right hilar region. Which of the following is most likely cause of the patient's laboratory findings? A. Syndrome of inappropriate ADH secretion B. Chlorpromazine toxicity C. Hyperproteinemia secondary to multiple myeloma D. Volume depletion from excessive vomiting and diarrhea E. Excessive water ingestion Explanation: Hyponatremia is characterized by an excess of water in relation to the total body sodium concentration. It is caused by excessive free water intake (primary polydipsia), endocrine disorders (i.e., adrenal insufficiency and hypothyroidism), or impaired water excretion from advanced renal failure and excessive antidiuretic hormone (ADH) release. Excessive ADH secretion can be due to decreased effective circulating volume (true volume depletion secondary to vomiting and diarrhea, congestive heart failure, cirrhosis or overuse of diuretics) and syndrome of inappropriate ADH secretion (SIADH). SIADH can be seen in a variety of CNS disorders, pulmonary diseases, HIV infection, tumors (i.e., small cell carcinoma of the lung), use of drugs such as carbamazepine, cyclophosphamide and selective serotonin re-uptake inhibitors, and in postoperative patients. Patients with hyponatremia secondary to SIADH usually have a decreased plasma or serum osmolality, elevated urinary osmolality (due to excessive fluid retention), urinary sodium concentration of more than 40 mEq/L and normal renal, adrenal and thyroid functions. The patient in the above vignette appears to have SIADH secretion secondary to lung cancer, most likely small cell carcinoma of the lung. (Choice B) The drugs that can cause excessive ADH release include selective serotonin re-uptake inhibitors (fluoxetine and sertraline), chlorpropamide, carbamazepine, and cyclophosphamide. Chlorpromazine is an anti-psychotic agent and is not associated with SIADH. (Choice C) Pseudohyponatremia secondary to hyperlipidemia or hyperproteinemia from any cause is usually associated with a normal or elevated plasma osmolality. (Choice D) True volume depletion due to gastrointestinal (vomiting, diarrhea) or renal causes can cause hyponatremia with a low plasma osmolality and elevated urine osmolality; however, the urinary sodium concentration is typically less than 20 mEq/L in these patients. (The urinary sodium concentration is usually more than 40 mEq/L in patients with SIADH.) (Choice E) Patients with hyponatremia secondary to primary polydipsia or excessive water ingestion excrete a very dilute urine and have a urine osmolality of less than 100 milliosmoles/kg. Educational Objective: The patients with hyponatremia secondary to SIADH typically have a low plasma osmolality, elevated urine osmolality, and a high urinary sodium concentration. 89% of people answered this question correctly. A 76-year-old male with a history of hypertension, diet-controlled type-2 diabetes mellitus, coronary artery disease, and atrial fibrillation comes to the office for a follow-up. The patient?s medication includes aspirin, metoprolol, hydrochlorothiazide, warfarin, amiodarone, and multivitamins. He has been on all these medications for one year. He complains of tiredness, weight gain, and occasional swelling in his feet for the last three months. He denies chest pain, dyspnea, orthopnea, or dizziness. Physical examination is unremarkable, except for a 10-lbs.-weight gain since his last visit six months ago. Which of the following is the most appropriate next step in the management of this patient?

A. Discontinue hydrochlorothiazide and add furosemide B. Decrease the dose of metoprolol C. Check thyroid-stimulating hormone level D. Discontinue amiodarone E. Add digoxin

Explanation: The next best step in the management of this patient is to check serum TSH levels. The patient is experiencing symptoms of hypothyroidism, which may be attributed to amiodarone use. Amiodarone causes thyroid dysfunction due to its high iodine content. Hypothyroidism (85%) is more common than thyrotoxicosis (10-15%). Two mechanisms by which amiodarone-induced-thyrotoxicosis may occur are: activation of Graves? disease (type-1 thyrotoxicosis) and destructive thyroiditis (type-2 thyrotoxicosis). Graves' disease induced by amiodarone is generally treated with high-dose thioamides (methimazole or propylthiouracil). Perchlorate can be used to decrease further iodine uptake by the thyroid gland. Steroids are generally required for treating type-2 thyrotoxicosis. Thyroid functions are monitored every six months in euthyroid patients on amiodarone. It is prudent to start with a lower dose in elderly patients or those who have significant coronary artery disease. (Choices A and E). The patient does not have congestive heart failure (no orthopnea, no SOB, no JVD, and normal lung exam); therefore, starting a stronger diuretic or adding digoxin would not help. (Choice B) Fatigue could be a side effect of a beta-blocker; however, in this case, fatigue is more likely due to amiodarone-induced hypothyroidism. (Choice D) It is not necessary to discontinue amiodarone if a patient becomes hypothyroid . Generally, patients who have amiodarone-induced hypothyroidism required a higher dose of levothyroxine to bring their TSH within normal range because amiodarone inhibits conversion of T4 to its active form T3. The hypothyroidism is treated with levothyroxine. Educational Objective: Amiodarone can cause thyroid dysfunction, corneal deposits, skin discoloration, pulmonary fibrosis (lipoid pneumonitis), and liver toxicity. 74% of people answered this question correctly. A 29-year-old Caucasian man is brought to the emergency department after he was found lying on the street, unconscious, and bleeding from his right leg. He was involved in a fight, and was stabbed twice in the right thigh. On arrival, his clothes are partially covered with snow. He is extremely pale, and his extremities are cold. His temperature is 35C (95F), blood pressure is 90/60 mm Hg, pulse is 120/min and respirations are 12/min. Examination shows decreased bilateral pulses, superficial breathing effort, clear lungs on auscultation, and decreased heart sounds. There is slow bleeding through one of the two penetrating wounds in his right thigh. The patient is stuporous, but does not have any focal neurological abnormality. He is emergently intubated and infused with warm fluids. His body is covered with warm blankets. The patient?s laboratory tests reveal: CBC Hb: 6.3 g/dL Ht: 19% MCV: 92 fl Platelet count: 150,000/cmm Leukocyte count: 4,500/cmm Segmented neutrophils: 65% Lymphocytes: 29% Monocytes: 6% Serum chemistry Serum Na: 134 mEq/L Serum K: 3.8 mEq/L Chloride: 100 mEq/L Bicarbonate: 14 mEq/L BUN: 15 mg/dL Serum Creatinine: 0.7 mg/dL

Calcium: 9.8 mg/dL Blood Glucose: 52 mg/dL Because of the acute anemia, the patient receives a transfusion of packed red blood cells. He also receives parenteral glucose and thiamine, and his mental status slightly improves. He suddenly develops muscle spasms of the face and upper extremities, diaphoresis, and bilateral hand contracture. Fundoscopy reveals papilledema. He soon develops involuntary, repetitive, generalized tonic-clonic seizures. He is given intravenous lorazepam, which temporarily controls the abnormal movements. He then has another seizure episode. Which of the following is the most appropriate immediate step in the management of this patient? A. Start bicarbonate infusion B. Start potassium replacement C. Start calcium replacement D. Start fresh frozen plasma infusion E. Order a head CT scan Explanation: In general, symptomatic hypocalcemia resulting from transfusion of citrated blood is rare, because normal individuals rapidly metabolize citrate in the liver and kidney; however, patients with renal failure, hepatic failure, hypothermia or shock who receive blood transfusions have a high risk of hypocalcemia. This is due to their inability to metabolize citrate, which is concomitantly transfused with every blood transfusion. The conversion of citrate into lactate is impaired, which leads to an excess amount of citrate in the blood. The excess citrate then binds calcium, and this leads to hypocalcemia. The measured serum calcium levels may remain normal despite this occurrence, because the deficit of ionized calcium is not reflected in the total calcium levels. Prophylactic administration of at least 10 cc of 10% calcium gluconate is therefore recommended for every 500 ml of packed red blood cells transfused. (Choice A) Sodium bicarbonate is only indicated if the patient?s serum bicarbonate levels are extremely low (5 mEq/L or less). As the patient?s circulation improves, there will be a rebound or increase in the concentration of alkali. This may be aggravated by exogenous administration. Some of the excess bicarbonate will also be transformed into carbon dioxide in the brain, which may cause paradoxical acidification of the central nervous system. (Choice B) Patients with hypothermia usually develop hypokalemia. This patient is currently normokalemic; however, as he recovers from the metabolic acidosis induced by hypothermia, his potassium level may become slightly low. At this point, an urgent potassium replacement is not necessary. (Choice D) There is no need for transfusion of fresh frozen plasma because there is no evidence of coagulopathy. (Choice E) The patient?s seizures and new neurologic findings are due to a metabolic problem. Transient papilledema is a usual finding in significant hypocalcemia that leads to seizures. Unless there is strong evidence or suspicion of head trauma, there is no urgent need for a head CT scan. Educational Objective: Patients with hypothermia or shock who receive blood transfusions are predisposed to hypocalcemia because of their impaired ability to metabolize citrate into lactate. In these patients (renal failure, hepatic failure, shock, lactic acidosis) prophylactic administration of at least 10 cc of 10% calcium gluconate is recommended for every 500 ml of packed red blood cells transfused. 33% of people answered this question correctly.

A 23-year-old African-American woman, gravida 2 para 1, presents to the emergency department at 32 weeks gestation with regular menstrual-like cramping and low back pain. Her past medical history is significant for preterm labor at 28 weeks gestation, which resulted in infant death in the early postnatal period. Physical examination shows regular uterine contractions and cervical dilation (3 cm). No evidence of membrane rupture is present. The contraction stress test is negative. You decide to proceed with adequate hydration and tocolysis. Which of the following steroids is preferred for antenatal corticosteroid therapy? A. Prednisone B. Betamethasone C. Hydrocortisone D. Fluticasone E. Fludrocortisone Explanation: Antenatal corticosteroid therapy has been proven to be effective in reducing perinatal morbidity and mortality associated with preterm labor. It reduces the risk of infant respiratory distress syndrome by stimulating phospholipids (surfactant!) synthesis and accelerating morphologic lung development. In addition, antenatal corticosteroid therapy appears to reduce the risk of intraventricular hemorrhage in infants. For these reasons, it should be given to any pregnant woman from 24 to 34 weeks of gestation with intact membranes at high risk for preterm delivery. Two regimens of therapy are available: betamethasone and dexamethasone (some authors believe that betamethasone is preferred over dexamethasone). (Choice C) Hydrocortisone is not considered effective because it is extensively metabolized in the placenta and only a little proportion reaches the fetus. (Choice A) Prednisone, the most popular agent for corticosteroid therapy, is not used for antenatal therapy. (Choice D) Fluticasone is an inhalational agent used for the treatment of allergic and inflammatory disorders. (Choice E) Fludrocortisone is a synthetic mineralocorticoid, not glucocorticoid agent. Educational Objective: Antenatal corticosteroid therapy has been proven to be effective in reducing perinatal morbidity and mortality associated with preterm labor. Betamethasone (and sometimes dexamethasone) is used for antenatal corticosteroid therapy. 79% of people answered this question correctly. The following vignette applies to the next 2 items A 65-year-old female is seen for acute mid-thoracic back pain. Seven days ago, she slipped over ice, and has had the back pain ever since. The pain is described as deep-seated, boring in quality, nonradiating, and without any aggravating or relieving factors. The pain has not improved, despite the use of over-the-counter analgesics. Her past medical history is significant for chronic obstructive pulmonary disease and hypertension. She is currently on fluticasone, salmeterol, and ipratropium inhalations, as well as oral lisinopril and hydrochlorothiazide. She denies prolonged use of oral glucocorticoids. She has a 30-pack-year history of smoking, but quit approximately five years ago, when she was diagnosed with chronic obstructive pulmonary disease. She had menopause when she was 48 years old. Her mammogram and Pap smear were negative six months ago. Her mother and two sisters have osteoporosis. Examination reveals tenderness over the mid-thoracic vertebrae. The rest of the examination is unremarkable. The complete blood count, basic metabolic panel, calcium

and phosphorus are within normal limits. Plain radiographs of the thoracic spine reveal a compression fracture in the 10th and the 11th vertebrae.

Item 1 of 2 What is the next best step in the management of this patient? A. Check bone mineral density of the hip and spine using dual energy X-ray absorptiometry (DEXA) B. Measure the heel bone mineral density by using ultrasound C. Bone biopsy D. Check parathyroid hormone levels E. Perform a technetium bone scan Explanation: The patient has multiple risk factors for bone loss. These factors are: Caucasian race, family history of osteoporosis, poor lifestyle, and postmenopausal status. Measurement of her bone density using a central device (preferably DEXA) is recommended. Although she qualifies for treatment irrespective of her bone mineral status due to her compression fracture, baseline DEXA will still be useful for monitoring her response to osteoporosis treatment. (Choice B) Peripheral devices such as heel ultrasounds can predict fracture risks; however, monitoring the patient's response to treatment using a peripheral device is not advocated. Even if low bone mass is documented by a peripheral device, baseline bone mineral density by DEXA will still be necessary for monitoring. Directly proceeding with measurement of central bone mineral density is therefore advised. (Choice C) Bone biopsy is almost never required for the diagnosis of primary osteoporosis. (Choice D) This patient has normal calcium and phosphorous levels. She does not have any clinical features that suggest primary hyperparathyroidism. Measurement of the PTH levels are not required. (Choice E) Bone scan is a sensitive but very nonspecific test for a variety of bone diseases. It can be useful in differentiating old from new vertebral fractures (an increased uptake is only seen in new compressive fractures.). It can also be useful if metastatic disease is suspected. In this patient, a bone scan is likely to show an increased uptake in the region of the compressive fracture, but this will not be very useful in managing the patient. Educational Objective: The evaluation of patients with suspected osteoporosis is best performed by performing central bone density measurement (hip and lumbar spine) using DEXA. 88% of people answered this question correctly. Item 2 of 2 Her 25-hydroxyvitamin D level is 25 mcg/L (normal 18 to 68 mcg/L), and TSH level is 2.1 microunits/mL (normal 0.35 to 5 microunits/mL). Bone density at the spine and hip shows a T-score of ?2.7 at the lumbar spine and ?2.0 at the hip. What is the best way to treat this patient? A. Start calcium and vitamin D supplementation B. Calcium and vitamin D supplementation with alendronate C. Calcium and vitamin D supplementation with daily nasal calcitonin D. Subcutaneous parathyroid hormone injections E. Calcium and vitamin D supplementation with hormone replacement therapy Explanation: Once osteoporosis is suspected, bone mineral density (BMD) should be measured. T- and Z- scores are routinely used to report BMD. The T-score is the BMD score of a patient in comparison to young healthy adults around 25-30 years of age. The Z-score is the BMD score of a patient compared to age-matched normal controls.

The World Health Organization categorizes BMD in postmenopausal Caucasian women (WHO criteria) according to the T-score (BMD score in comparison to young white females), as follows: normal osteopenic osteoporosis

(T-score > 1.0) (T-score between ?1.0 and 2.5) (T-score 1.0) osteopenic (T score between ?1.0 and 2.5) osteoporosis (T score 220/120). If blood pressure reduction is required, labetalol is the drug of choice. (Choice B) Overzealous hydration and D5W solution are usually avoided. (Choice D) Despite initial encouraging evidence, large-scale studies demonstrated no improvement in the outcome of patients with thrombotic stroke receiving nimodipine. (Choice E) Lumbar puncture is indicated when a patient has a negative CT scan and the clinical suspicion of subarachnoid hemorrhage remains high. Educational Objective: The reduction of blood pressure in patients with acute ischemic stroke can deteriorate their condition, and should be used only if the blood pressure is extremely high (>220/120). Despite initial encouraging evidence, large-scale studies demonstrated no improvement in the outcome of patients with thrombotic stroke receiving nimodipine. 48% of people answered this question correctly. Item 2 of 2 The patient receives the appropriate therapy. Two days after the event, he does not show any progression of neurological deficit. The EKG shows no rhythm abnormalities, and carotid duplex scanning is normal. What is the best next step in the management of this patient? A. Discharge the patient B. Order echocardiography C. Repeat CT scan D. Schedule carotid angiography E. Obtain EEG Explanation: Determining the stroke subtype is important in patients with ischemic stroke in terms of short-term as well as long-term treatment decisions. Thrombotic, embolic and lacunar strokes may require different management approaches that underscore the importance of the underlying pathophysiology. Cardiac evaluation should be performed in almost all patients with brain ischemia. Echocardiography is indicated in patients with suspected embolic stroke and in patients in whom the mechanism of stroke is not clear, especially if carotid studies are negative. Left ventricular systolic dysfunction appears to be more common in patients who present with stroke than in the general population, and many patients with stroke have asymptomatic left ventricular dysfunction that can be detected on echocardiography.

(Choice C) In stable patients with no progression of neurological deficits, repeat CT scan has little diagnostic significance. (Choice D) Conventional angiography is rarely performed nowadays. It may be helpful in patients with fibromuscular dysplasia and carotid dissection, as well as patients undergoing preoperative evaluation of carotid artery disease. Educational Objective: Echocardiography is indicated in patients with suspected embolic stroke and in patients in whom the mechanism of stroke is not clear, especially if carotid studies are negative. 42% of people answered this question correctly.

A 55-year-old Caucasian female is brought to the emergency department by her son because of slurred speech and confusion. These symptoms started several days ago and gradually progressed. Her past medical history is significant for hypertension, bipolar disorder, and deep vein thrombosis. Her son says that the ?mood disorder? runs in the family because his grandmother had the same problem. She takes lithium, which seems to control her symptoms well. One month ago, she was diagnosed with deep vein thrombosis and hospitalized. Warfarin treatment was started at the hospital, and she continues to take this drug with periodic ?blood tests?. The physical examination reveals a mild tremor of her hands and an unstable gait. She is oriented to person, but not to time and space. If medication toxicity is responsible for this patient?s current symptoms, which of the following is the most likely additional finding? A. Hormone replacement therapy (HRT) for hot flushes B. Hydrochlorothiazide for hypertension treatment C. Low INR value on routine coagulation studies D. Right-sided hemiparesis with slight motor aphasia E. Bilateral hemianopsia on visual field testing Explanation: Lithium is widely used as an effective agent to treat bipolar disorder. It has a low therapeutic index. An overdose may result due to intentional poisoning or other factors. The mortality is 25% in patients with acute lithium poisoning and 8% in patients on maintenance dose. Manifestations of lithium toxicity are diverse: gastrointestinal, renal, endocrine, dermal, cardiovascular, hematological, as well as neurological abnormalities have been described. A significant number of patients suffers irreversible neurological damage. Typical neurological symptoms include lethargy, confusion, tremors, fasciculations and ataxia. This scenario demonstrates how various factors can affect serum lithium levels and precipitate lithium toxicity. Any factor that decreases renal excretion, such as renal insufficiency or effective volume depletion (e.g., diuretics or congestive heart failure) may be responsible. (Choice A) HRT may be responsible for deep vein thrombosis in this patient, but the question asked about the current neurological symptoms. (Choice C) A low INR value on routine coagulation studies indicates inadequate anti-coagulation. (Choices D and E) Right-sided hemiparesis would suggest a stroke, and bilateral hemianopsia on visual field testing would most probably indicate the presence of an intracranial mass lesion in this patient. Educational Objective: Any factor that decreases renal excretion may be responsible for the increase of serum lithium levels. 25% of people answered this question correctly. The following vignette applies to the next 2 items A 61-year-old male with hypercholesterolemia, diabetes mellitus type 2 and hypertension comes to the office because of constipation. He has hyperlipidemia, hypertension and diabetes mellitus controlled with atorvastatin, hydrochlorothiazide and rosiglitazone, respectively. He denies alcohol intake, but admits smoking two packs of cigarettes daily for the last several years. Colonoscopy performed one year ago was normal. Lab investigations reveal a serum calcium level of 14.2 mg/dL and an albumin level of 4.0 gm/dL. Item 1 of 2 What is the most likely cause of this patient?s hypercalcemia? A. Primary hyperparathyroidism

B. Secondary hyperparathyroidism C. Drug-induced D. Malignancy E. Diabetic nephropathy Explanation: This patient has a longstanding history of smoking; therefore, he has higher chances of developing lung cancer. Patients with squamous cell carcinoma of the lung can produce parathyroid hormone related protein (PTHrP), which can cause hypercalcemia. It is sometimes difficult to differentiate primary hyperparathyroidism from hypercalcemia seen during malignancy. Typically, the serum calcium level in patients with malignant lung tumors is much higher than those with primary hyperparathyroidism. This patient?s serum calcium level is 14.2 mg/dL, and is most likely due to a malignancy. (Choice A) Primary hyperparathyroidism is one of the most common causes of hypercalcemia in an outpatient setting. As stated above, the level of serum calcium is lower in patients with primary hyperparathyroidism compared to patients with malignancy-induced hypercalcemia. Calcium levels above 12mg/dL are rare. (Choice B) Secondary hyperparathyroidism due to renal failure or vitamin D deficiency is not typically associated with hypercalcemia. Patients with secondary hyperparathyroidism can sometimes develop autonomously functioning nodules of the parathyroid gland, which is then called tertiary hyperparathyroidism. Hypercalcemia can occur in the presence of very high serum PTH levels in tertiary hyperparathyroidism. (Choice C) HCTZ can cause hypercalcemia, but the elevations in serum calcium levels are usually minimal. (Choice E) Diabetic nephropathy per se is not associated with hypercalcemia. Educational objectives: The three most common causes of hypercalcemia include primary hyperparathyroidism, malignancyinduced hypercalcemia and vitamin D-induced hypercalcemia. In hypercalcemia secondary to malignancy, the serum calcium levels are generally very high. 34% of people answered this question correctly. Item 2 of 2 The physical examination of the patient is essentially unremarkable. What is the most appropriate next step in management? A. Serum PTH measurement B. Bone scan C. CT scan of the chest D. Chest x-ray E. Renal biopsy Explanation: Hypercalcemia can be divided in to two broad categories: (1) parathyroid dependent and (2) parathyroid independent. Examples of parathyroid dependent hypercalcemia include primary or tertiary hyperparathyroidism, lithium-induced hypercalcemia, and familial hypocalciuric hypercalcemia. Parathyroid independent causes include malignancies, vitamin D toxicity, granulomatous disease, and milk-alkali syndrome. Malignancy-induced hypercalcemia is generally due to the secretion of PTHrP, but other mechanisms have also been described, particularly in hematological malignancies.

These categories can be distinguished by measuring the serum parathyroid hormone levels; therefore, the most appropriate next step in this case is to do a parathyroid estimation to help classify this patient's hypercalcemia as parathyroid dependent or independent. Specific PTHrP assays are commercially available and are usually ordered with PTH if malignancy is highly being considered in the differential diagnosis. (Choices B, C, D and E) Bone scan, CT scan, chest x-ray, and renal biopsy are not indicated at this point. In majority of endocrinological disorders, biochemical confirmation is required before any imaging is performed. Educational Objective: Hypercalcemia can be divided in to two broad categories: (1) parathyroid dependent and (2) parathyroid independent. The most appropriate next step in management is to do a parathyroid estimation. A 36-year-old African-American man was brought to the hospital because of fever, chills, productive cough, and shortness of breath. He also has night sweats, loss of appetite, and weight loss. He is unemployed and homeless; he was found in the street and brought to the hospital by some concerned policemen. He has no other medical problems. He drinks 6-8 bottles of beer daily. His family history is not significant. He has no medications. He has no known drug allergies. His temperature is 38.9 C (102 F), pulse is 104/min, and respirations are 22/min. His pulse oximetry is 92% at room air. His chest x-rays show a left upper lobe alveolar infiltrate. He is admitted and started on clindamycin and gatifloxacin. On his fourth hospital day, he is breathing better and has more appetite, but he continues to be febrile. His laboratory results reveal that one of his sputum samples is positive for acid-fast bacilli (AFB), and he is started on anti-tuberculosis (TB) therapy. The nurses that were taking care of him in the unit approach you to ask about their risk of acquiring tuberculosis (TB). They want you to tell them what they can do about it. Which of the following is the most appropriate course of action? A. Place a PPD in three weeks and evaluate B. Place a PPD now and repeat it in three weeks C. Take a chest x-ray in three weeks D. Start prophylactic therapy with isoniazid (INH) for those who attended him for more than two consecutive days without using a mask E. Take a chest x-ray and place a PPD in three weeks Explanation: The Center for Disease Control (CDC) recommends immediately giving a PPD test to all health care workers (HCW) exposed to a contagious patient with tuberculosis. If the result is negative, the PPD test will be repeated after three weeks to check for any changes that could have been induced by the recent exposure to the bacteria. (Choice D) Prophylactic therapy with INH is not needed at this point, as it is not known if the HCWs were infected or not. (Choices C and E) PPD test must be given before ordering a chest x-ray, unless the HCW has specific respiratory symptoms. (Choice A) The initial PPD must always be given immediately because the HCW can be PPD positive from before, and not due to the most recent exposure. Educational Objective: When a HCW is exposed to a patient with a contagious form of tuberculosis (i.e., laryngeal, bronchial, or pulmonary), the CDC recommends immediate PPD testing to determine the baseline immunologic status; three weeks after, PPD retesting will be given to check for any changes due to the recent TB exposure.

The following Vignette applies to the next 3 items A 25-year-old Caucasian man comes to the emergency department because of persistent pain and limited range in motion of the right wrist. Five hours ago, while playing football, he fell to the ground and landed on his outstretched right hand. He went home after the incident because his hand did not look too swollen and the pain was tolerable. A few hours later, he noticed that he was unable to move his hand, and decided to come to the hospital. He has no other medical problems. He denies the use of tobacco, alcohol or drugs. Examination shows mild swelling in the dorsum of the right hand. The hand cannot be fully flexed, and can be extended only by passive motion due to pain. There is severe tenderness on palpation of the anatomical snuffbox. Item 1 of 3 Which of the following is the most likely diagnosis? A. Colles? fracture B. Chondral fracture C. Wrist sprain D. Scaphoid fracture E. Triangular cartilage injury Explanation: A history of falling on an outstretched hand is characteristic of a scaphoid fracture. It is most commonly seen in young adults between the ages of 15 and 30 years. It results from a fall on the outstretched hand, resulting in severe hyperextension and slight radial deviation of the wrist. Accompanying symptoms are complete (or greater than 50%) loss in range of motion of the wrist joint, severe pain, and stiffness. Physical examination reveals tenderness on palpation of the scaphoid within the anatomic snuffbox. (Choice C) Wrist sprain is characterized by mild pain, stiffness, normal range of motion of the wrist joint, minimal tenderness on palpation of the lunate and scaphoid (navicular) bones, and mild swelling of the dorsum. (Choice B) Chondral fracture is characterized by moderate pain, stiffness, and approximately 20% loss in range of motion of the wrist joint. (Choice E) Triangular cartilage injury is characterized by tenderness of the region distal to the ulnar styloid, which increases in severity with forced ulnar deviation. (Choice A) Patients with Colles? fracture have tenderness in the region located 2 cm below the radioulnar joint. In almost half of the cases, the ulnar styloid separates from the rest of the bone, and lateral view of the wrist reveals a dinner fork deformity. Educational Objective: A history of falling on an outstretched hand, complete (or greater than 50%) loss in range of motion of the wrist joint, severe pain, and tenderness on palpation of the scaphoid within the anatomic snuffbox is diagnostic of scaphoid fracture until proven otherwise. Item 2 of 3 An x-ray of the wrist reveals no evidence of dislocation or fracture. After receiving analgesic therapy, the patient reports that the pain in his wrist has improved. Which of the following is the most appropriate next step in the management of this patient? A. Prescribe oral analgesics, rest for two weeks, and discharge the patient home B. Order a computerized tomography (CT) scan of the right wrist C. Order a magnetic resonance imaging (MRI) study of the right wrist D. Place a cast brace with the hand on dorsal hyperextension

E. Place a cast brace with the hand on dorsal flexion Explanation: Patients with a non-displaced scaphoid fracture can have normal radiographs for up to two weeks after a traumatic incident. Overlooking this lesion may lead to complications such as traumatic arthritis (which results from nonunion of the fracture) and avascular necrosis of the scaphoid bone. A high clinical suspicion for scaphoid fracture warrants the use of further diagnostic studies (i.e., CT scan of the hand, bone scan), even if the initial x-ray results are negative. (Choice A) Analgesic therapy and resting the affected hand are indicated in the management of wrist sprain. (Choice C) MRI studies are used in the diagnosis of triangular cartilage injury. Although it may also be used in this case, its higher cost makes it less appealing than a CT scan, which is enough to make the diagnosis. (Choice E) Cast placement in the management of a patient with uncomplicated Colles? fracture involves immobilization of the hand in the neutral position, or ideally, in the normal volar tilt position. (Choice D) Cast placement with the hand in a hyperextended position is not beneficial for both Colles? and scaphoid fractures. The uncomplicated, undisplaced scaphoid fractures are treated with immobilization in a long or short arm thumb spica cast with the wrist in slight radial deviation and in neutral flexion.

Educational Objective: Although displaced fractures of the scaphoid can be identified in x-rays immediately after trauma, some scaphoid fractures can take one or two weeks before it becomes apparent in x-rays. A high clinical suspicion for scaphoid fracture therefore warrants the use of further diagnostic studies (i.e., CT scan of the hand, bone scan), even if the initial x-ray results are negative. The uncomplicated, undisplaced scaphoid fractures are treated with immobilization in a long or short arm thumb spica cast with the wrist in slight radial deviation and in neutral flexion. 9% of people answered this question correctly. Item 3 of 3 The patient is concerned about the complications that he might develop. Which of the following is the most common complication of his injury? A. Avascular necrosis B. Nonunion C. Malunion D. Infection E. No complications Explanation: The most common complication of scaphoid fractures is nonunion. The other important complication is avascular necrosis. These two complications often result because the blood flows from the distal to proximal portion of the scaphoid bone, and this proximal portion is completely dependent on the distal blood supply. Proximal fractures of the scaphoid therefore require longer immobilization (up to 12 weeks) to ensure adequate healing. Educational Objective: The most common complication of scaphoid fracture is nonunion.

9% of people answered this question correctly. A follow-up study was conducted to assess the effect of circumcision on the incidence of penile carcinoma. A large cohort was investigated. The rate in uncircumcised males was 8/1000, and the rate in circumcised males was 6/1000. The investigators concluded that circumcision may be markedly effective in reducing the incidence of the disease in the population. According to the study results, how many patients should be circumcised to prevent one case of penile carcinoma? A. 100 B. 250 C. 500 D. 1000 E. 5000 Explanation: The "number needed to treat" (NNT) is the number of patients who need to be treated in order to prevent one additional bad outcome. It is an important way to present the results of a study or assess the usefulness of treatment or prophylaxis. Sometimes, it is more convenient for practitioners to use NNT than measures of association (which represents the strength of association, not the practical aspects of treatment efficacy). The calculation of NNT is easy: it is actually the inverse of absolute risk reduction (ARR). ARR = Control group event rate ? Experimental group event rate = 8/1000 ? 6/1000 = 2/1000 NNT = 1/ARR = 1/0.002 = 500 Therefore, according to the study results, 500 males should be circumcised in order to prevent one case of penile carcinoma. Educational Objective: NNT is the number of patients who need to be treated in order to prevent one additional bad outcome; it is actually the inverse of absolute risk reduction. The following vignette applies to the next 2 items A 21-year-old Caucasian male presents to your office with a non-pruritic rash. He says that he noticed it in the spring season, and it got worse during the summer. His past medical history is insignificant. He does not smoke or consume alcohol, and denies any recreational drug use. He has been sexually active with two partners during the last year, and he uses condoms occasionally. The physical examination reveals multiple circular hypopigmented macules on the chest, back and upper arms. Item 1 of 2 Which of the following is the most likely diagnosis in this patient? A. Pityriasis rosea B. Tinea versicolor C. Secondary syphilis D. Eczema E. Rocky mountain spotted fever Explanation: Tinea versicolor is a fungal infection of the skin that is caused by the dimorphic yeast Pityrosporum orbiculare (also known as Malassezia Furfur). The clinical picture is usually very characteristic: multiple small circular maculae are observed that may vary in color (white, pink or brown). The rash is typically more prominent in the summer time because the yeast inhibits pigment transfer to keratinocytes and makes the affected skin more demarcated from unaffected tanned skin. The lesions are usually asymptomatic, although mild pruritus may be present. The typical location of the lesions is the upper trunk, but the rash may also involve the upper arms, neck and abdomen.

(Choice A) Pityriasis rosea is characterized by the presence of a herald patch, confinement of the lesions to the central trunk, orientation of the lesion along the lines of cleavage of the skin, and a typical pink color of the lesions. (Choice C) Secondary syphilis commonly involves the hands and feet. (Choice D) Eczema typically affects the extremities and produces scaling and severe pruritus. (Choice E) RMSF is an infectious disease that is characterized by an acute onset and severe systemic symptoms. Educational Objective: In patients with tinea versicolor, multiple small circular maculae are observed which may vary in color (white, pink or brown). 75% of people answered this question correctly. Item 2 of 2 Which of the following is the best treatment for this patient? A. Oral erythromycin B. Topical terbinafine C. Penicillin IM D. Topical corticosteroids E. Oral doxycycline Explanation: The treatment of choice for patients with tinea versicolor is topical anti-fungal therapy. Any anti-yeast topical agent can be used, including terbinafine, clotrimazole and ketoconazole. The success rate with topical anti-fungal agents exceeds 80%. With extensive disease or recalcitrant infection, oral antifungals are preferred (ketoconazole, itraconazole or fluconazole). (Choice A) Oral erythromycin is effective in patients with pityriasis rosea. (Choices C) Penicillin is used for the treatment of syphilis. (Choice E) Doxycycline is used in the treatment of patients with RMSF. (Choice D) Topical corticosteroids can be used in patients with eczema. Educational Objective: The treatment of choice for patients with tinea versicolor is topical anti-fungal therapy. 70% of people answered this question correctly. A 65-year-old Caucasian female presents to the emergency department with progressive shortness of breath on minimal exertion and fatigue. She was diagnosed with myelodysplasia two years ago, and has been receiving supportive therapy with frequent RBC transfusions. Her last transfusion was 3 months ago. She experienced an episode of severe pneumonia one year ago that required hospitalization and IV antibiotic therapy. She is taking no medications currently and has no known allergies. Her blood pressure is 120/70 mmHg and heart rate is 95/min. Physical examination reveals pallor. Systolic murmur with intensity of II/VI is heard over the cardiac apex. Laboratory findings are significant for hematocrit of 24% and hemoglobin level of 5.7 mg/dL. You consider RBC transfusion in this patient. Blood grouping and cross-matching are done, but the blood bank is unable to find suitable blood. This is the first time such an incompatibility has occurred. What is the most likely reason for this incompatibility?

A. Autoantibodies B. Alloantibodies C. Anti-Rh (D) antibodies D. Anti-HLA antibodies E. ABO incompatibility Explanation: After blood is ordered for transfusion, the following compatibility testing is usually performed. First, the patient?s ABO and Rh types are determined. After this, the patient's serum is screened for unexpected antibodies, a procedure called pretransfusion antibody screening. Pretransfusion antibody screening is intended to detect any of all clinically significant RBC antibodies. If negative, the patient can be safely transfused. If positive, further investigation is usually warranted to evaluate the identity of the antibody. The major problem that leads to difficulties finding cross-matched blood in patients with a history of multiple transfusions is alloantibodies (e.g., in patients with sickle cell anemia or myelodysplasia). The most commonly implicated RBC antigens in that case are E, L and K. Moreover, these patients tend to develop multiple alloantibodies that make finding compatible blood even more difficult. (Choices C and E) Finding ABO and Rh-compatible blood is usually not a big challenge. (Choice A) Autoantibodies are less likely to cause difficulties in cross-matching in this patient; they are commonly implicated as a cause of the incompatible cross-match in patients with autoimmune anemia and taking certain drugs (e.g., methyldopa and procainamide). (Choice D) HLA allosensitization increases risk of graft rejection in patients awaiting organ or bone marrow transplantation and platelet refractoriness in those requiring subsequent platelet transfusion support. Remember that RBCs do not express HLA antigens. Educational Objective: The major problem that leads to difficulties finding cross-matched blood in patients with a history of multiple transfusions is alloantibodies. 52% of people answered this question correctly.

A 31-year-old multigravid woman presents to your office for counseling on the various methods of contraception. She has four children, and had three elective abortions in the past. She wants the most effective method of contraception, but does not want it to be "permanent". Her past medical history is insignificant. Which of the following is the most appropriate method of contraception for this patient? A. Oral contraceptives B. Implantable or injectable contraceptives C. Diaphragms/cervical caps D. Male condoms E. Spermicides Explanation: Implantable and injectable contraceptives, including implantable levonorgestrel and depot medroxyprogesterone acetate, have the lowest rate of pregnancy (does not exceed 2-3%). The actual pregnancy rate may be as low as 0.3%. (Choice A) Oral contraceptives are also very effective, but the actual pregnancy rate is higher than with implantable and injectable contraceptives due to inconsistent or incorrect use. (Choices C and D) Barrier methods of contraception such as diaphragms, cervical caps, and male condoms are moderately effective due to inconsistent or incorrect use. The actual pregnancy rate is 12-14%. (Choice E) Spermicides, if used alone, have a high failure rate. Educational Objective: Implantable and injectable contraceptives, including implantable levonorgestrel and depot medroxyprogesterone acetate, have the lowest rate of pregnancies among nonpermanent methods of contraception. 62% of people answered this question correctly. A 26-year-old white woman who is a long distance runner comes to your office. She trains daily and participates frequently in marathons and other long-distance competitive events all around the country. She denies smoking or alcohol drinking, and says that she follows a healthy, low-fat diet. Her last menstrual period was three years ago, but this is not a problem for her as she does not want to get pregnant "at least in the next five years." After an extensive work-up, her amenorrhea was attributed to exercise. She is 168 cm (5?6??) tall and weighs 50 kg (110 lbs). Her blood pressure is 100/60 mmHg, pulse is 58/min, and respirations are 14/min. Physical examination shows no abnormality. You alert the patient that her amenorrhea poses her in risk of another condition. Which of the following conditions are you referring to? A. Thyroid disease B. Hyperprolactinemia C. Early menopause D. Osteoporosis E. Hypocholesterolemia Explanation: Females who maintain a lower weight or body mass index (BMI) due to the sport or activity that they regularly engage in (e.g. ballet dancers, gymnasts, and runners) may become hypoestrogenic and present with exercise-induced amenorrhea. They are at special risk of developing osteopenia, and even osteoporosis. Spontaneous fractures have been reported in these types of patients, with osteoporosis or osteopenia confirmed through bone densitometry. Treatment consists of improving

caloric intake; if this is not possible, patients are started on hormonal replacement with oral contraceptives and supplementation with calcium and vitamin D. (Choice E) Paradoxically, there is a tendency toward hypercholesterolemia, not the opposite. (Choice C) There is no risk of early menopause, but exercise-induced amenorrhea can mimic early menopause because of estrogen deficiency. (Choices A and B) Exercise-induced amenorrhea is a diagnosis of exclusion. Thyroid hormone and prolactin level determinations are necessary in the evaluation of this patient because one/both of these may be the underlying cause (but not a complication or aftermath) of the amenorrhea. Educational Objective: Exercise-induced amenorrhea is due to a decrease in the pulsatile secretion of LH, which leads to a decline in estrogen production. It can lead to osteopenia, osteoporosis, breast and vaginal atrophy, mild hypercholesterolemia, and infertility. 72% of people answered this question correctly. The following vignette applies to the next 2 items A 62-year-old Caucasian man comes to the physician because of progressive weight loss and right lower extremity weakness for the last two months. On further questioning, he tells you that he has been having frequent leg cramps in the right leg. These cramps are most prominent in the morning, after he wakes up. He also has difficulty with swallowing and chewing. He denies any problems with bowel or bladder function. His other medical problems include hypertension and hypercholesterolemia. He does not use tobacco. His family history is not significant. His vital signs are within normal limits. The physical examination reveals weakness in his right lower leg, atrophy and fasciculation of the thigh and calf muscles, a hyperactive knee jerk, and tongue fasciculations. The rest of his neurological examination, including the sensory examination, is normal. Item 1 of 2 Which of the following is the most likely diagnosis? A. Binswanger?s disease B. Vascular dementia C. Multiple sclerosis D. Amyotrophic lateral sclerosis E. Brain stem glioma Explanation: Amyotrophic lateral sclerosis (ALS) is the most common form of progressive motor neuron disease. It is a relentlessly progressive disorder that involves both the lower motor neurons (consisting of anterior horn cells in the spinal cord and brainstem neurons innervating the bulbar muscles) and upper or corticospinal motor neurons. At its onset, ALS may involve selective loss of function of only upper or lower motor neurons, but it ultimately causes progressive loss of both upper and lower motor neurons. The initial sign of the disease with lower motor neuron involvement is an insidiously developing asymmetric weakness, usually first evident distally in one of the limbs. Patients may disclose a history of recent development of cramping with volitional movements that typically occur in the early morning hours (e.g. while stretching in bed). Weakness is associated with progressive wasting, atrophy of muscles, and spontaneous twitching or fasciculations of motor units. Involvement of the bulbar muscles leads to difficulty with chewing and swallowing, as well as fasciculations of the face and tongue.

ALS with prominent corticospinal involvement is characterized by hyperactivity of muscle-stretch reflexes (tendon jerks) and frequent spastic resistance to passive movements of the affected limbs. Ocular motility, sensory, bowel, bladder, and cognitive functions are preserved, even with advanced disease. (Choice B) Vascular dementia is characterized by the presence of behavioral disturbances and cognitive deficits associated with clinical or radiographic evidence of a stroke. The decline in the level of cognition is relatively abrupt and progresses in a stepwise fashion. (Choice A) Binswanger?s disease is a type of vascular dementia that involves white matter infarcts. Patients with this disease usually present with apathy, agitation, and bilateral corticospinal or bulbar signs. (Choice C) Multiple sclerosis is usually seen in younger females with two or more clinically distinct episodes of CNS dysfunction. Patients typically have sensory, visual, or bladder and bowel dysfunction. (Choice E) Brain stem tumors may compress the cervical cord and produce weakness, fasciculations in the upper limbs, and spasticity in the legs. The presentation may closely resemble ALS; however, absence of pain or sensory changes and normal bowel and bladder function in the patient favor ALS. Educational Objective: ALS should be suspected in patients who present with progressive weakness accompanied by both upper and lower motor neuron deficits. Ocular motility, sensory, bowel, bladder, and cognitive functions are preserved, even with advanced disease. 79% of people answered this question correctly. Item 2 of 2 Which of the following treatment has been shown to be beneficial in these patients? A. Riluzole B. Corticosteroids C. IV immunoglobulins D. Plasmapheresis E. Surgery Explanation: Riluzole is a glutamate inhibitor that is currently approved for the management of amyotrophic lateral sclerosis. Although it cannot arrest the underlying pathological process, it may prolong survival and delay the need for a tracheostomy. (Choices B, C, and D) Amyotrophic lateral sclerosis is a neurodegenerative disease. Agents such as corticosteroids, intravenous immunoglobulins, and cyclophosphamide, have no role in its management. These agents may be useful in immunologically-mediated neurological diseases such as multiple sclerosis and Guillain-Barr 頳 yndrome. (Corticosteroids are used to treat acute exacerbations of multiple sclerosis, while plasmapheresis and intravenous immunoglobulins are the main treatment modalities of Guillain-Barr 頳 yndrome.) Educational Objective: Riluzole is a glutamate inhibitor that is currently approved for the management of amyotrophic lateral sclerosis. 42% of people answered this question correctly. A 65-year-old female is admitted to the intensive care unit because of unstable angina. She has a history of hypertension, diabetes, hyperlipidemia, and osteoporosis. Cardiac catheterization is

performed and shows severe, triple-vessel, coronary artery disease. She has had recurrent angina while in the hospital. A coronary artery bypass graft procedure is planned. Your intern reported to you that her thyroid-stimulating hormone level in a blood sample drawn in the emergency department is 36 mU/mL (0.35 ?5.0 mU/mL is normal). Which of the following is the most appropriate next step in the management of this patient? A. Call the cardiac surgeon to cancel the surgery B. Proceed with cardiac surgery C. Start her on liothyronine (T3) orally and postpone surgery D. Start liothyronine (T3) intravenously and proceed with cardiac surgery E. Start her on a usual dose of levothyroxine (T4) and perform surgery when euthyroid

Explanation: The patient needs to undergo cardiac surgery on an emergent basis. Hypothyroidism, even if severe, is not a contraindication to emergency surgical procedures. Postoperative mortality rate is not increased in patients with hypothyroidism undergoing major surgeries. These hypothyroid patients, however, are at a slightly higher risk for developing postoperative ileus and hyponatremia, and are more prone to oversedation with narcotic medications. (Choice A) There is no need to cancel the scheduled surgery. (Choices C and D) Liothyronine is not indicated in the treatment of hypothyroidism . (Choice E) Levothyroxine (T4) is the treatment of choice in patients with hypothyroidism; however, due to this patient's age and significant coronary artery disease, levothyroxine should be started on a lower dose. Higher doses of levothyroxine in elderly patients or in patients with significant coronary artery disease will increase myocardial oxygen demand and can precipitate myocardial infarction. In such cases, lower doses of levothyroxine should be started postoperatively when the coronary blood flow improves. The dose is then gradually increased to achieve euthyroidism. Educational Objective: Hypothyroidism, even if it is severe, is not a contraindication to an emergency surgical procedure. 40% of people answered this question correctly. A 27-year-old Caucasian man comes to the physician for a routine health maintenance examination. He has no other medical problems. His father had colon cancer. His vital signs are within normal limits. Examination shows no abnormalities. Which of the following information is the most important to determine the patient's risk for colon cancer? A. Age of onset of colon cancer in his father B. History of cigarette smoking C. Type of diet D. Associated symptoms and signs E. History of alcohol use Explanation: The history of colon cancer in a first-generation relative is the most important risk factor. This risk is further increased if more than one first-generation relatives are affected, or if colon cancer develops at an age younger than 55. Colon cancer occurs at an earlier age in patients with familial adenomatous polyposis and familial nonpolyposis colorectal cancer, which are very important risk factors for colon cancer. (Choice C) A diet rich in fiber, folate, and calcium has a protective effect against colon cancer. (Choice B and E) Both alcohol use and cigarette smoking have been associated with an increased risk of colon cancer; however, early development of colon cancer in a first-generation relative is a more important risk factor. (Choice D) Associated signs and symptoms give little information regarding the risk of colon cancer in the above patient. Educational Objective: Early development of colon cancer in a first-generation relative is a very important risk factor for evaluating a patient for colon cancer prevention. 73% of people answered this question correctly.

A 42-year-old Caucasian man comes to see you in the office with symptoms of dyspepsia and heartburn for the last two years. His symptoms are progressively getting worse. He has tried lifestyle changes, over-the-counter antacids, and ranitidine earlier, but these have not significantly relieved his symptoms. He denies any history of dysphagia, odynophagia, weight loss, or gastrointestinal bleeding. He has no other medical problems. He works as an assistant to a lawyer. He smokes a pack of cigarettes daily and drinks 6 to 8 cups of coffee daily. His physical examination is unremarkable. Which of the following is the most appropriate next step in the management of this patient? A. Start him on a trial of a proton pump inhibitor B. Obtain esophageal manometry C. Schedule him for an ambulatory pH monitoring D. Refer to a gastroenterologist E. Refer to a surgeon for antireflux surgery Explanation: The patient has symptoms consistent with gastroesophageal reflux disease. This results from a combination of excessive gastric acid reflux and impaired clearance of the acid by the esophagus. Mild symptoms can usually be managed by simple lifestyle and dietary modifications, antacids, and non-prescription H2 blockers. Patients with more severe and prolonged symptoms, and those who fail initial management usually require more aggressive therapy, which involves a trial of proton pump inhibitors for at least eight weeks. All the available agents in this class have similar efficacy when used in equivalent doses. (Choices B and C) Esophageal manometry and pH monitoring have very limited and specific roles. Manometry is used to facilitate the placement of ambulatory pH probes and to guide antireflux surgery. Ambulatory pH monitoring helps to confirm the diagnosis in patients with persistent symptoms without the endoscopic evidence of mucosal damage. It also helps to evaluate refluxassociated pulmonary and upper respiratory symptoms. (Choice D) The patient should be referred to a gastroenterologist for an upper endoscopy if he fails a trial of proton pump inhibitors. Endoscopy should be considered early in the course if patients present with symptoms of dysphagia, odynophagia, weight loss, or gastrointestinal bleeding. (Choice E) Antireflux surgery is too invasive, and should not be recommended unless all other modalities have failed. Educational Objective: A trial of proton pump inhibitors should be used in patients who have failed other conservative therapies. Endoscopy should be done early if the patient complains of dysphagia, odynophagia, significant weight loss, and GI bleeding. 61% of people answered this question correctly. The following vignette applies to the next 2 items An 82-year-old Caucasian female comes to the office for the evaluation of a low-grade fever and rash on her left flank area for the past two days. She has a past medical history of hypothyroidism, diabetes mellitus, rheumatoid arthritis, osteoporosis, and a recent hospital admission for urosepsis. Her medications include aspirin, ciprofloxacin, alendronate, levothyroxine, glipizide, low-dose prednisone, calcium, and vitamin D supplementation. She has no known drug allergies. She does not smoke or drink alcohol. On physical examination, her temperature is 37.8C(100F), heart rate is 92/min, blood pressure is 124/72 mmHg, and respiratory rate is 14/min. Examination of the rash reveals the presence of multiple small vesicles over an erythematous base in the thoracic 9-10 nerve root distribution. Item 1 of 2

Which of the following is the most appropriate step in the management of this patient? A. Perform a Tzanck smear. B. Obtain a complete blood count with differential count. C. Start the patient on acyclovir. D. Obtain a skin biopsy to confirm the diagnosis. E. Obtain blood for varicella-zoster virus serology. Explanation: The presentation and physical findings of the patient are classic for herpes zoster, which occurs as a result of reactivation of latent varicella-zoster virus (VZV) infection in the sensory or dorsal nerve root ganglion. The population at greatest risk of developing herpes zoster includes the elderly and immunosuppressed patients (i.e., patients on cancer chemotherapy or chronic steroid therapy, transplant recipients, and HIV-infected patients). The disease is characterized by the development of a painful vesicular rash in the distribution of specific nerve roots. Most patients have a prodromal phase consisting of fever, malaise, headache, and localized pain, followed by the development of a vesicular eruption several days later. The vesicles typically appear along the thoracic or lumbar dermatomes, and these evolve into pustules before crusting in about 7-10 days. The pain is the most consistent symptom, and can persist for days to months after the rash resolves. The treatment of choice is oral acyclovir. The goal of antiviral treatment is to promote early healing, and to prevent or reduce the duration and severity of complications. If started within 48 hours of the onset of rash, acyclovir has been shown to improve acute pain and prevent the development of postherpetic neuralgia in a significant number of patients. (Choices A, B, D and E) Herpes zoster is a clinical diagnosis. Treatment should be started based on the typical signs and symptoms. Diagnostic workup such as Tzanck smear, skin biopsy or serologies is not necessary. Educational Objective: Antiviral therapy with acyclovir is the mainstay of treatment for herpes zoster. Treatment should be initiated based on clinical suspicion within 48 hours of the onset of rash for maximum efficacy. 70% of people answered this question correctly. Item 2 of 2 The appropriate step was taken for the patient. Over the next 24 hours, she developed a similar rash in other parts of her body. Due to severe pain, she was admitted to the hospital. Which of the following is the most appropriate next step in the management of this patient? A. Isolate the patient until the lesions are crusted. B. Isolate the patient until the pain resolves. C. There is no need to isolate the patient. D. Isolate the patient only if varicella-zoster virus serology is positive. E. Isolate the patient only if she has extensive or disseminated herpes zoster. Explanation: All major organizations, including the Center for Disease Control and Prevention (CDC) have recommended guidelines regarding the isolation of patients with varicella-zoster virus (chicken pox) infections. Varicella-zoster virus is a highly contagious virus that is transmitted from person to person by droplets and close contact. The risk of transmission and development of varicella is extremely high in susceptible patients. Most people, including healthcare workers, have a history of varicella and have immunity against the infection; however, nosocomial transmission of the infection can occur in all susceptible adults, healthcare workers, and immunocompromised patients. This can result in severe disease and complications such as bacterial superinfection of the lesions, varicella pneumonia, cerebellar ataxia, meningoencephalitis, and death. Infection in pregnant woman may lead to the development of congenital varicella syndrome in newborn infants.

Herpes zoster is also considered infectious, although the risk of transmission is not as high as that of varicella. According to the current recommendations, all patients with disseminated herpes zoster should be maintained on contact and airborne isolation until all skin lesions are dry and crusted. All healthcare workers without a history of varicella-zoster virus infection are strongly encouraged to avoid contact with patients with varicella or herpes zoster infection. (Choice B) The pain of herpes zoster may persist for weeks to months after all the lesions have resolved. It is not necessary to isolate the patient until that point. (Choice C) All patients with herpes zoster should be isolated to prevent nosocomial spread to susceptible healthcare workers. (Choice D) There is no need to check VZV serology in patients with herpes zoster. (Choice E) All hospitalized patients with localized dermatomal herpes zoster should be isolated to prevent nosocomial spread. Educational Objective: Airborne and contact isolation should be maintained for all patients with disseminated herpes zoster infection until all their lesions are dry and crusted. Contact precautions are indicated in hospitalized patients with localized herpes zoster, but these are not currently recommended in a community setting. 53% of people answered this question correctly. A 35-year-old white male comes to the emergency department. He is complaining of a severe, rightsided abdominal pain that started two hours ago, and has been gradually worsening. The pain now radiates to his groin, and is described as 10/10 in intensity and sharp in nature. The patient feels nauseated, but has not actually vomited. He denies any history of trauma to his flank. His past medical history is not significant. His family history is significant for three first-degree relatives with kidney stones. On examination, he has very mild tenderness on the right lumbar region. Murphy?s sign is negative. Urinalysis results are as follows: Blood: moderate RBC: 25-30/HPF WBC: 0-2/HPF Nitrite: negative Leukocyte esterase: negative After relieving the patient?s pain with intravenous hydromorphone (Dilaudid), important next step in the management of this patient?

what is the most

A. Order ultrasonogram B. Order x-ray of kidney, ureter, and bladder (KUB) C. Order an intravenous pyelogram (IVP) D. Order a CT scan of the abdomen and pelvis E. Order a urine culture and sensitivity Explanation: Nephrolithiasis should strongly be suspected in any patient who presents with atraumatic flank pain, particularly if abdominal tenderness is absent, and urinalysis shows hematuria. The history of the patient is very suggestive of nephrolithiasis. Studies have shown the superior sensitivity and specificity of non-contrast helical CT scan over other diagnostic modalities in the diagnosis of nephrolithiasis. The standard procedure is to obtain 8 mm slices using the CT scan, however decreasing the size to 3-5 mm slices will further increase sensitivity and specificity. An added advantage is that this modality is fast, widely available, and enables doctors to search for other

possible etiologies of the pain without unnecessary delay, if the CT scan results turn out to be negative. (Choice A) USG of the kidneys, ureters and bladder (KUB) is the procedure of choice for cases wherein IVP or CT scan cannot be obtained or is contraindicated (i.e. pregnant patients); however, it may miss small stones, and sometimes miss even ureteral stones. (Choice B) X-Ray of KUB without IVP detects calcium-containing stones only. Since 20% of kidney stones do not contain calcium, failure to visualize these types of stones makes this procedure unfavorable. (Choice C) IVP is a very sensitive and specific procedure. However, it has a few disadvantages compared to CT scan. IVP is relatively more time-consuming, and is only slightly less expensive than CT scan. These reasons, along with the possible risk for allergic reactions occurring secondary to the dye, have resulted to IVP being used as a second line investigation after CT scan. (Choice E) The rapid onset of symptoms and urinalysis results (negative for WBC, leukocyte esterase and nitrites) has made this case unlikely to be a urinary tract infection. Ordering a urine culture and sensitivity at this point is not needed, and may add an unnecessary delay to the treatment of this patient. Educational Objective: Non-contrast helical CT scan is the gold standard for the diagnosis of nephrolithiasis, followed by an IVP. In pregnant patients, or in individuals where you want to avoid the radiation exposure, choose the USG as the procedure of choice. 30% of people answered this question correctly. A 65-year-old Caucasian male comes to the clinic in September for a follow-up visit regarding an episode of acute sinusitis he developed after a lingering cold. He reports some improvement in his symptoms of mild headache, cough, nasal congestion, and increased sinus drainage. He attributes this improvement in symptoms to his usage of over-the-counter decongestants and the amoxicillin prescribed a few days ago. He received his last dose of tetanus toxoid 5 years ago. Given his recent susceptibility to illness, one of his close friends suggested he seek out any recommended vaccinations at this visit. To address this issue, which of the following is the most appropriate next step? A. Provide reassurance but do not vaccinate today B. Administer tetanus toxoid C. Administer tetanus toxoid and influenza vaccine D. Administer influenza vaccine and pneumococcal vaccine today E. Administer influenza vaccine today and pneumococcal vaccine next week Explanation: The three most commonly administered vaccinations for adults include the influenza vaccine, the pneumococcal vaccine, and tetanus toxoid. Influenza season includes the winter months (with a peak incidence in January), and the vaccine is only effective for the year in which it is given. The indications for influenza vaccination in adults include the following: 1. 50 years of age and older 2. chronic respiratory, cardiac, or metabolic conditions 3. immunosuppressed state 4. residents of long-term care facilities 5. pregnant women 6. healthcare workers and people in prolonged contact with adults in these high-risk categories

On the other hand, the pneumococcal vaccine provides protection against Streptococcus pneumoniae, a pathogen responsible for a substantial proportion of community-acquired pneumonias and subsequent hospitalizations. Moreover, pneumococcal pneumonia is a known common complication of influenza infection. For these reasons, pneumococcal vaccination is recommended for all adults 65 years of age and older. Adults who were vaccinated more than five years ago (while younger than 65 years of age) are eligible for one additional re-vaccination. Concurrent administration of the pneumococcal and influenza vaccines is safe and effective; therefore, this patient should receive both vaccines. (Choices B and C) Tetanus toxoid should be administered to the following individuals: 1. those who have serious or dirty wounds (e.g., puncture wounds or wounds contaminated with soil, feces, or saliva) and who received their last dose more than five years ago 2. those who have clean minor wounds and who received their last dose more than ten years ago Since this patient has not incurred a wound recently, he does not need to receive tetanus toxoid. (Choice E) Resolving sinusitis and antibiotic usage are not contraindications to vaccination; therefore, there is no need to postpone administration of the pneumococcal vaccine until next week. (Choice A) Providing only reassurance is not advisable, since this patient is clearly prone to upper respiratory tract infections, and is in the recommended age group for both influenza and pneumococcal vaccinations. Educational Objective: Pneumococcal vaccine should be administered to adults 65 years of age and older. Influenza vaccine should be administered to adults 50 years of age and older, as well as those adults in various high-risk categories (e.g., pregnant women, healthcare workers, the immunosuppressed or chronically ill). 72% of people answered this question correctly. A 7-year-old Caucasian boy is brought to the emergency department due to a two-day history of fever, chills, cough and progressive shortness of breath. The cough is productive of greenish-yellow sputum. The patient was diagnosed with cystic fibrosis after three episodes of severe pneumonia two years ago. He is currently not taking any medications. He has no known allergies, and all his immunizations are up-to-date. His blood pressure is 110/60 mmHg, pulse is 130/min, temperature is 38.9C(102F) and respirations are 32/min. Physical examination reveals decreased breath sounds over the left lower lobe and scattered wheezing bilaterally. Chest x-ray shows an infiltrate in the lower left lobe. Which of the following is the best treatment option for this patient? A. Vancomycin B. Azithromycin C. Piperacillin plus tobramycin D. Ampicillin plus clindamycin E. Dicloxacillin Explanation: Severe exacerbations of pulmonary disease in patients with cystic fibrosis usually require IV antibiotic therapy. Ideally, the selection of antibiotics should be done according to the results of the sputum culture. Empirical therapy should cover Pseudomonas aeruginosa as a common pathogen, and it usually includes at least two drugs. The most popular combination is an aminoglycoside (e.g., tobramycin) with an anti-pseudomonal semi-synthetic penicillin (e.g., piperacillin or ticarcillin). Sometimes, a third generation cephalosporin or a carbapenem is used. (Choice A) If methicillin-resistant S. aureus infection is diagnosed, vancomycin is employed.

(Choices B and E) Azithromycin or dicloxacillin can be used as oral agents in less severe cases in patients with susceptible flora. Azithromycin may be a useful long-term agent to prevent acute exacerbations. (Choice D) Ampicillin plus clindamycin is not a good option for empirical therapy in this patient. Educational Objective: The most commonly used empiric antibiotic therapy in cystic fibrosis patients with severe exacerbations of pulmonary disease is an aminoglycoside with anti-pseudomonal semi-synthetic penicillin. 78% of people answered this question correctly. A 7-year-old Caucasian boy is brought to the emergency department due to a two-day history of fever, chills, cough and progressive shortness of breath. The cough is productive of greenish-yellow sputum. The patient was diagnosed with cystic fibrosis after three episodes of severe pneumonia two years ago. He is currently not taking any medications. He has no known allergies, and all his immunizations are up-to-date. His blood pressure is 110/60 mmHg, pulse is 130/min, temperature is 38.9C(102F) and respirations are 32/min. Physical examination reveals decreased breath sounds over the left lower lobe and scattered wheezing bilaterally. Chest x-ray shows an infiltrate in the lower left lobe. Which of the following is the best treatment option for this patient? A. Vancomycin B. Azithromycin C. Piperacillin plus tobramycin D. Ampicillin plus clindamycin E. Dicloxacillin Explanation: Severe exacerbations of pulmonary disease in patients with cystic fibrosis usually require IV antibiotic therapy. Ideally, the selection of antibiotics should be done according to the results of the sputum culture. Empirical therapy should cover Pseudomonas aeruginosa as a common pathogen, and it usually includes at least two drugs. The most popular combination is an aminoglycoside (e.g., tobramycin) with an anti-pseudomonal semi-synthetic penicillin (e.g., piperacillin or ticarcillin). Sometimes, a third generation cephalosporin or a carbapenem is used. (Choice A) If methicillin-resistant S. aureus infection is diagnosed, vancomycin is employed. (Choices B and E) Azithromycin or dicloxacillin can be used as oral agents in less severe cases in patients with susceptible flora. Azithromycin may be a useful long-term agent to prevent acute exacerbations. (Choice D) Ampicillin plus clindamycin is not a good option for empirical therapy in this patient. Educational Objective: The most commonly used empiric antibiotic therapy in cystic fibrosis patients with severe exacerbations of pulmonary disease is an aminoglycoside with anti-pseudomonal semi-synthetic penicillin. 78% of people answered this question correctly. A healthy, 44-year-old, Middle-eastern, male taxi driver had a motor vehicle accident. He was a restrained driver. You come to help him, along with the paramedics, and find him unresponsive with spontaneous respirations. His blood pressure is 100/60 mmHg, pulse is 112/min, and respirations are 12/min. Examination shows clear lung fields and normal first and second heart sounds. He could not

be extricated from the car. Suddenly, he starts to cough and spit some blood. Which of the following is the most appropriate course of action? A. Intubate immediately B. Suction the mouth C. Do cricothyrotomy D. Do the jaw lift maneuver E. Stabilize the spine first Explanation: There is a considerable risk of neck trauma during most motor vehicle accidents. The first thing to always do in such trauma cases is to secure the airway. Whenever the airway needs to be protected, it is advisable to use the jaw lift maneuver to avoid further complications if there is any neck injury. (Choice B) After securing the airway, the paramedics can proceed to suction the mouth to prevent any possible aspiration. (Choices A and C) The patient has spontaneous respirations. There is no indication for immediate intubation or cricothyrotomy at this point. (Choice E) Stabilization of the spine is also done after securing the airway. Educational Objective: Whenever there is an accident with possible neck trauma, the airway must be secured through the jaw lift maneuver to avoid further strain in the neck area. Cricothyrotomy may be necessary if there is complete obstruction of the airway. If the patient is unconscious, completely unresponsive, unable to protect the airway, not breathing or coughing, he or she will need to be intubated. 20% of people answered this question correctly.

A group of investigators is exploring trends in the incidence of coronary artery disease (CAD) in a large population. They studied several factors in relation to the incidence of the disease. The relationships between the cigarette sales in the population (average number of cigarette packs sold per person per day) and the incidence of CAD (per 1000 people) over time are given on the figure below. Which of the following is the best statement concerning the results of the study? A. No stable trend is present in the incidence of CHD over time. B. Quitting smoking decreases the risk of CHD for a person living in this population. C. Cigarette sales were stable over time in the population. D. No individual-level conclusions can be ascertained from the given plot . E. Discordant changes in cigarette sales and incidence of CHD are present. Explanation: No individual-level conclusions can be ascertained from the given plot. The study described is a typical ecologic (correlational) study. It is important to remember that ecologic studies give population-level information, not individual-level information! Applying population-level information to an individual level may lead to a bias called ecologic fallacy. (Choices A and C) The given plot illustrates the temporal trends of cigarette sales and incidence of CHD; it is clear that the incidence of CHD, as well as cigarette sales, steadily declined over time in the population. (Choice E) Because both the incidence of CHD and cigarette sales declined progressively, the changes over time are concordant, not discordant. (Choice B) It cannot be concluded directly from this plot that quitting smoking decreases the risk of CHD for a person living in this population. A similar trend could be possibly observed by plotting personal computer usage in this population (increasing over time); however, it also cannot be concluded that personal computer use decreases the risk of CAD. Educational Objective: It is important to remember that ecologic studies give population-level information, not individuallevel information! 35% of people answered this question correctly. A 2-year-old boy is brought to the physician because of a delay in language development. According to his mother, the only words that he says are "mama" and "dada." He runs well and walks up and down stairs without assistance. He builds a tower of seven blocks and turns pages one at a time. He was delivered vaginally at 39 weeks? gestation; his mother's pregnancy was uncomplicated, but prenatal visits were not regularly followed. His past medical history is unremarkable. His mother is worried and asks if such development is normal. Which of the following is the most appropriate next step in the boy's management? A. Reassurance B. Watchful waiting C. Hearing test D. Screening for TORCH organisms E. CT scan of the head Explanation: This child has normal developmental milestones, except for language which is delayed. A 2-year-old child is supposed to know a 50+ word vocabulary, use 2- to 3-word phrases, and follow two step commands. Delayed speech development may indicate an underlying hearing impairment; therefore, such children should be referred for audiologic assessment. Other candidates for hearing evaluation

include children with a family history of hereditary childhood hearing loss, history of meningitis, history of recurrent or persistent otitis media with effusion for more than three months, documented intrauterine infections, craniofacial anomalies, and use of ototoxic medications such as aminoglycoside. Hearing loss early in life, even mild or unilateral, may affect speech and language development, social and emotional development, and academic achievements. Early identification is thus mandatory for a better prognosis; hence, screening programs have been widely and strongly advocated. Although the American Academy of Pediatrics endorses universal screening, at present, systematic screening is implemented in 32 states only. The etiology of hearing impairment depends on whether the hearing loss is conductive or sensorineural. The most common cause of conductive hearing loss in children is presence of fluid in the middle ear. Other causes include tympanic membrane perforation (trauma or infection), ossicular discontinuity (infection, cholesteatoma, and trauma), tympanosclerosis, and congenital anomalies of the external ear canal or middle ear components. As for sensorineural hearing loss (SNHL), it may be congenital or acquired. The most common infectious cause of congenital SNHL is CMV. Other less common congenital infectious causes include toxoplasmosis, syphilis, and rubella. Postnatal infectious causes include group B streptococcus and bacterial meningitis. Hearing loss can also be genetically determined; it may be autosomal dominant or recessive, isolated or in a syndromic association with other anomalies. (Choices A and B) Reassurance and watchful waiting are inappropriate because hearing impairment should be identified promptly. (Choices D and E) The other diagnostic methods may be more or less useful for evaluation of this child; however, hearing impairment should first be confirmed, and its type identified by a hearing test. Educational Objective: Delayed speech development may indicate an underlying hearing impairment and should prompt referral of the patient for audiologic assessment. Other candidates for hearing evaluation include children with a family history of hereditary childhood hearing loss, history of meningitis, history of recurrent or persistent otitis media with effusion for more than three months, documented intrauterine infections, craniofacial anomalies, and use of ototoxic medications such as aminoglycoside. 61% of people answered this question correctly. A 78-year-old Caucasian man with end stage esophageal cancer is admitted to the hospital with severe malnutrition and failure to thrive. The patient's caretaker tells you that he has not been able to eat or drink for the last six weeks. His weight dropped from 160 pounds to 120 pounds during that time. The cancer has spread to his lungs and liver. The patient expresses his wishes to not receive any further treatment for the cancer, and specifies that he does not want any heroic measures or interventions done to keep him alive. Keeping his current clinical condition in mind, you think about offering hospice care to the patient. When is the most appropriate time to refer this patient to hospice care? A. Tell the patient that you would not be able to treat her if she refuses to sign the consent. B. He should be referred to hospice care two weeks prior to his anticipated death. C. He should be referred to hospice care two months before his anticipated death. D. He should be referred to hospice care after he has cleared his hospital bills. E. CT scan of the head Explanation: Hospice care is usually provided to terminally ill patients who have a predicted life expectancy of six months or less. The largest population of patients receiving hospice care consists of cancer patients.

Other patients receiving hospice care have terminal medical conditions such as endstage cardiomyopathy, endstage chronic obstructive pulmonary disease, or pulmonary fibrosis. Hospice care is based on the principle of providing compassionate and comprehensive support and care, which includes, but is not limited to psychological, social, nursing, and palliative medical care to a dying patient. It also provides support and respite care to the family members or caregivers of the terminally ill patient. Hospice care is provided by a multidisciplinary team, which includes a registered nurse, nurse?s aide, social workers, chaplains, and a hospice physician who closely coordinates with the patient?s attending physician. It is usually provided at the patient?s own home (home hospice care), but it can also be given as an inpatient hospice care for patients who are not functionally independent. (Choices B and C) There is a general tendency to delay hospice care to eligible patients by their caregivers or attending physicians. This leads to a shorter length of stay in hospice programs, and deprives the patients of the full benefits available to them. The patient in the above scenario has advanced esophageal cancer with an expected survival of less than six months. He should be referred to hospice care program now, or as soon as possible. Educational Objective: Patients with advanced metastatic cancers or other terminal illnesses with an expected life expectancy of less than six months should be evaluated for hospice care. 95% of people answered this question correctly. A 32-year-old pregnant woman who recently migrated from Eastern Asia is brought to the hospital by her husband. She is on her 37th week of gestation. She has been having regular uterine contractions for the last four hours. Examination reveals that she is in active labor. She agrees to be admitted for delivery. You then explain to the patient and her husband the various possible methods of delivery and the complications that can arise from them. The couple appears to understand everything that you explain. When you ask the patient to sign the consent sheet for treatment, she becomes reluctant, and tells you that women in their culture are not allowed to sign any papers if their husbands are alive. She asks you to have her husband sign the consent sheet for her treatment. Which of the following is the most appropriate reaction to the patient's request? A. If the child does not cry after she finishes feeding him. B. Ask the hospital not to admit the patient if she refuses to sign the consent. C. Tell her that you would not be responsible for any malpractice claims if her husband signs the paper. D. Obtain a witness to the above conversation, and treat the patient. E. CT scan of the head Explanation: Informed consent involves a process whereby a patient or his/her surrogate decision-maker makes a healthcare decision based on the physician?s recommendation. Before arriving at a decision, the patient and the healthcare professional work must together as a team. It is the physician?s obligation to provide all the medical facts accurately to the patient or the surrogate decision maker in accordance with good medical practice. It is important to ensure that the information provided to the patient is adequate, and is easily and completely comprehended. The patient or surrogate then exercises the right of autonomy and makes the healthcare decision. In the above vignette, the patient was provided with adequate information. She and her husband appear to understand the information completely. She has a right to choose a surrogate decision maker, such as her husband, to make healthcare decisions on her behalf. Both the patient and her husband want to proceed with the planned treatment as well. The patient should be treated appropriately even if she refuses to sign the consent for treatment herself. (Choices A and B) Refusal of admission or treatment of the patient is unethical and inappropriate.

(Choice C) This statement is not true. The physician has to assume responsibility for all actions which directly involves the management of his patient. Educational Objective: Informed consent is considered effective when sufficient information is provided by a physician, and is easily and adequately comprehended by the patient or their surrogate decision maker. 80% of people answered this question correctly.

A healthy 28-year-old Middle-Eastern woman comes to the physician for a routine health maintenance examination. She has just delivered a 3.5 Kg (7.7 lb) baby boy three days ago. This was her first pregnancy. She has no complaints. She is a housekeeper. She does not use tobacco, alcohol, or drugs. Examination shows no abnormalities. She is breastfeeding, and she asks you what the best way is to know if the baby is being fed properly. Which of the following is the most accurate answer? A. If the child does not cry after she finishes feeding him. B. Make sure you feed him using both breasts. C. Make sure you feed him every 2 hours. D. Make sure you feed him every 4 hours. E. Make sure you feed him anytime he cries. Explanation: Most breastfeeding authorities, as well as the CDC (Center for Disease Control), recommend feeding infants at least every four hours, as well as to look for early signs of hunger (like hand or finger suckling or arms movement toward the mouth) to determine proper breastfeeding. (Choice A) Breastfeeding has a soporific effect in the infant. If the baby does not cry, this does not mean that he was properly fed. (Choice C) Some infants may need feeding every two hours; however, there is no scientific evidence to support this method. (Choice E) Newborns cry for many different reasons, and not always because they need to be fed. Furthermore, it is not advisable to wait until the newborn cries to breastfeed him. (Choice B) It is a good practice to feed the newborn using both breasts, as complete emptying of the breast will increase the milk?s nutritional qualities. Unfortunately, sometimes there may be a decreased production of milk, and the baby has to be fed using artificial formula. Feeding the baby using both breasts will therefore not guarantee the infant?s satiety. Educational Objectives: Every primary care physician should encourage breastfeeding. It is recommended that the infant be fed at least every four hours. The mother should be able to identify early signs of hunger, like suckling of the hand or fingers or arm movements towards the mouth, as preterm or debilitated infants may not be able to cry vigorously or show agitation. 26% of people answered this question correctly. A 14-year-old Caucasian male presents to the pediatrician for a follow-up visit regarding his acne. On his face are numerous comedones and many inflamed papules and pustules, and there is some evidence of mild scarring. At his first visit three months ago for this problem, he was prescribed topical retinoid and benzoyl peroxide, which he has been using regularly. He feels that the acne has improved slightly with this treatment regimen. What is the most appropriate means of managing his condition? A. Add topical erythromycin to the regimen B. Add oral corticosteroids to the regimen C. Begin treatment with oral tetracycline only D. Begin treatment with oral isotretinoin only E. No change in treatment; re-evaluate in three months

Explanation: Although there are no definitive evidence-based guidelines on the treatment of acne, some general principles are observed by most clinicians. Treatment regimens usually progress through several stages, based upon how severe the acne is at presentation and how effective the treatment options are in resolving the symptoms. Mild acne is initially treated with topical retinoid. Moderate acne (or refractory mild acne) is treated with topical retinoid and either benzoyl peroxide or a topical antibiotic. More severe acne (or refractory moderate acne) is treated with: 1) topical retinoid, benzoyl peroxide, and topical antibiotic (Choice A); or 2) systemic antibiotic plus topical retinoid or benzoyl peroxide. Note that systemic antibiotics (Choice C) are not typically used exclusively. The most severe cases of acne should be treated with a systemic antibiotic and topical retinoid or benzoyl peroxide; if there is no response after 3-6 months, then oral isotretinoin (Choice D) is indicated. Corticosteroids (Choice B) are used in patients with excessive adrenal androgen production (e.g., cases of congenital adrenal hyperplasia). Microcomedones take eight weeks to mature, so any acne therapy regimen must be continued for at least this length of time before determining its efficacy. Since this boy has already waited three months, it would be unreasonable to ask him to wait another three months (Choice E) before reevaluating the treatment regimen. Educational Objective: Topical products (e.g., retinoid, benzoyl peroxide, antibiotics) are first-line treatments for mild to moderate acne. Systemic products such as oral antibiotics are reserved for more significant acne, and oral isotretinoin is used for the most severe, refractory forms of acne. 36% of people answered this question correctly. The following vignette applies to the next 2 items A 16-year-old Caucasian boy is brought to the ED via ambulance by his school nurse. The nurse tells you that the boy has not been feeling well for the past two days. She first saw him two days ago when he was complaining of headache and generalized body aches. At that time, she evaluated him, concluded that he was suffering from a viral prodrome, and prescribed him acetaminophen for his symptoms. Earlier this morning, he had two episodes of vomiting while sitting in his classroom, and he was extremely drowsy. In the ED, he is minimally responsive. He experiences another episode of emesis and is emergently intubated by the emergency room resident for airway protection. Subsequent evaluation, including a lumbar puncture for CSF analysis, reveals that he is suffering from acute meningococcal meningitis. Item 1 of 2 Which of the following contacts should be offered antibiotic prophylaxis to prevent the development of meningococcal meningitis? A. A single intramuscular injection of penicillin G B. His teachers at school C. Emergency room physicians and other staff members D. Paramedics who brought the patient to the emergency room E. No change in treatment; re-evaluate in three months Explanation: Neisseria meningitidis is the most common cause of acute bacterial meningitis in children and adolescents from ages 2-18 years. The portal of entry of this bacteria is via the nasopharynx.

Although colonization of Neisseria meningitidis in the nasopharynx is relatively common and affected patients are usually asymptomatic, it can cause mucosal invasion with systemic bacteremia, seeding of the meninges, and subsequent development of potentially fatal meningitis in predisposed individuals. Meningococcal meningitis is a devastating illness and is associated with significant morbidity and mortality. The mortality rates are even higher in adult patients with meningitis. It is therefore recommended that antibiotic chemoprophylaxis be used to prevent the spread of meningococcal meningitis. All close contacts must receive antibiotic prophylaxis. These include people who live in the same household and persons who have had prolonged, close contact with the patient. In addition, all healthcare workers (including physicians) who have had direct exposure to the respiratory secretions of the patient (e.g., while suctioning or intubating the patient) should receive chemoprophylaxis. (Choices B, C, & D) Those persons with brief exposure to the patient (e.g., either at school or work, or during transportation) do not require antibiotic prophylaxis. Antibiotic prophylaxis is not indicated if the exposure to the patient is brief or short, and if there is no direct exposure to the respiratory or nasopharyngeal secretions. It is only required for persons who have had prolonged contact in a closed environment with the patient. Educational Objective: All persons with prolonged, close contact or direct exposure to the respiratory secretions of a patient with meningococcal meningitis should receive antibiotic prophylaxis to prevent the spread and development of meningitis. 37% of people answered this question correctly. Item 2 of 2 You identify and contact the persons in whom the antibiotic prophylaxis is indicated. Which of the following is the most appropriate therapy for chemoprophylaxis against meningococcal meningitis? A. A single intramuscular injection of penicillin G B. Inhaled tobramycin for two days C. A single dose of intravenous vancomycin D. Oral rifampin E. Two days of cefazolin Explanation: As described above, antibiotic prophylaxis is indicated in all the close contacts of patients with meningococcal meningitis. Neisseria meningitidis tends to colonize the nasopharynx and invade the mucosa, thereby causing systemic bacteremia and subsequent meningitis in predisposed patients. Antibiotic prophylaxis is used to eliminate the nasopharyngeal colonization of meningococci, and is effective in preventing the development of meningitis in close contacts. Oral rifampin (600 mg every 12 hours for a total of four doses) is the recommended regimen for chemoprophylaxis. Other antimicrobials that can be used for chemophrophylaxis include fluoroquinolone (a single 500 mg dose of ciprofloxacin), sulfonamides, ceftriaxone, and minocycline. (Choices A, B, & C) Penicillin G, inhaled tobramycin, cefazolin, or intravenous vancomycin is not recommended for chemoprophylaxis to prevent the spread of meningococci. Educational Objective: Oral rifampin is the recommended antimicrobial for chemoprophylaxis against the spread of meningococcal meningitis. The other commonly used regimens are single oral dose of ciprofloxacin or single intramuscular injection of ceftriaxone. 87% of people answered this question correctly.

A 26-year-old Caucasian woman who is suffering from cystic fibrosis comes to the physician. She has recently changed her boyfriend and is trying to conceive these days. Her boyfriend does not have any medical problem. She is a non-smoker and non-alcoholic. Which of the following statements is true regarding fertility in a patient with cystic fibrosis? A. Prazosin B. There is a 20% chance that you may not become pregnant. C. There is a 50% chance that you may not become pregnant. D. Only males are infertile; you will not have any problem. E. Air pollution

Explanation: Chances of infertility in a female with cystic fibrosis are 20% percent. This increased risk is due to secondary amenorrhea caused by malnutrition, and is also due to thick, tenacious cervical mucus. Chances of infertility in a male with cystic fibrosis are 95%. Spermatogenesis is normal in a patient with cystic fibrosis, but sperm transport is impaired because of impaired development of the Wolffian duct. Educational Objective: Chances of infertility in a female with cystic fibrosis are 20% percent. Chances of infertility in a male with cystic fibrosis are 95%. 27% of people answered this question correctly. The following Vignette applies to the next 2 items A 33-year-old Caucasian female presents to your office with a six-month history of cold intolerance. When the weather is cold, her third and forth fingers on both hands get numb and turn blue. She says, ?I wear gloves to keep my hands warm, but it does not always work?. Her past medical history is significant for non-ulcer dyspepsia treated with omeprazole. She does not smoke or consume alcohol, and denies any recreational drug use. She has no known drug allergies. She works as a nurse at a local hospital and considers her job moderately stressful. She is sexually active with her husband and uses condoms for contraception. Physical examination findings are within normal limits. Item 1of 2 Which of the following medications is the best initial treatment for this patient? A. Prazosin B. Nifedipine C. Verapamil D. Aspirin E. Topical nitroglycerin Explanation: This patient presents with symptoms suggestive of Raynaud phenomenon, an exaggerated vascular response to cold temperature or emotional stress. Calcium antagonists are considered the first-line pharmacological therapy for these patients, but it should be mentioned that not all calcium antagonists are equally effective. Most dihydropyridine calcium channel blockers (e.g., nifedipine and amlodipine), as well as diltiazem, are proven to work well in these patients. Many other vasodilator agents have been tried to treat this condition. (Choice C) Verapamil does not seem to be effective or at least equally effective as calcium antagonists.

(Choice A) Alpha-adrenergic blockers such as prazosin are effective; however, patients eventually become refractory to these agents after prolonged usage. (Choice E) Nitroglycerin is usually used as an adjunct agent to calcium antagonist therapy in patients with severe Raynaud phenomenon. It is not recommended as the first-line agent. (Choice D) Some authors recommend adding aspirin to the treatment regimen in patients with severe ischemia and necrosis to prevent platelet aggregation; however, aspirin should be used with caution because it can actually worsen vasospasm by blocking the synthesis of vasodilatory prostaglandins. Educational Objective: Calcium antagonists, typically nifedipine or amlodipine, are considered the first-line pharmacological therapy for patients with Raynaud phenomenon. 59% of people answered this question correctly.

Item 2 of 2 The patient returns in one month and complains that the treatment ?does not help much.? In addition, she has started to experience periodic joint and muscle pain. She says that she has been adherent to the medication and has tried to avoid going out in cold weather as much as possible. Her husband even gave up smoking because you had warned her that passive smoking may worsen her condition. Which of the following is the best next step in the management of this patient? A. Increase the dose of the medication B. Add a second agent C. Order arterial Doppler ultrasonography D. Obtain ANA and RF E. Reassure and observe Explanation: Raynaud phenomenon may be idiopathic (also called Raynaud disease) or secondary to other conditions such as connective tissue diseases, vascular lesions, medications, etc. When managing patients with Raynaud phenomenon, potential precipitating factors such as medications and environmental factors (e.g., frostbite, vibration) should first be excluded. The presence of symptoms suggestive of systemic disease (arthralgias and myalgias) and resistance to treatment (such as in this patient) warrant further evaluation; this may include ANA, RF, CBC, blood chemistry, urinalysis and measurement of complement levels. (Choice E) Patients are diagnosed with primary Raynaud phenomenon when there are no historic clues to a secondary condition, normal physical findings and no ischemic digital lesions. No specialized studies are indicated in these patients. (Choice C) In patients who have symptoms suggestive of vascular lesions (e.g., asymmetric involvement and deficient pulses), further evaluation with digital plethysmography, arterial Doppler ultrasonography or angiography should be considered. (Choices A and B) It is not reasonable to change the treatment without first excluding a systemic disease that may be responsible for the condition. Educational Objective: The presence of symptoms suggestive of systemic disease (arthralgias, myalgias, fever, and weight loss) in patients with Raynaud phenomenon warrants further evaluation; this may include ANA, RF, CBC, blood chemistry, urinalysis and measurement of complement levels. 67% of people answered this question correctly. A 26-year-old male who was initially admitted in the intensive care unit three days ago for diabetic ketoacidosis (DKA) is transferred to the medical floor after getting the appropriate initial treatment. He has had type-1 diabetes for the past two years, and is currently on glargine (14 units at bedtime) and lispro (8 units three times a day before meals). His blood glucose levels during the last two days have ranged from 120?200 mg/dl. He started tolerating oral feeding yesterday. Twenty-four hours after being transferred to the medical floor, he started to have a spiking fever up to 102?F (39? C), and developed foul-smelling nasal discharge and pain in his right paranasal area. On examination, the nasal mucosa was inflamed, and there was a black discoloration in the antero-inferior aspect. Which of the following is the most effective agent for the treatment of this infection? A. Amphotericin B B. Terbinafine C. Itraconazole D. Voriconazole E. Flucytosine

Explanation: Patients with DKA are predisposed to get mucormycosis, an infection caused by fungi from the class Zygomycetes. Although this class of fungi does not cause infections in a normal host, it may do so in patients with DKA. Because of the acidic environment and high free iron, these pathogens can infect the nasal mucosa, and invade the surrounding structures rapidly. The common clinical features are fever, facial swelling, maxillary pain and tenderness, nasal discharge, ophthalmoplegia and headache. The diagnosis is suggested by the characteristic clinical picture, and confirmed by performing a biopsy of the infected tissues. Treatment consists of debridement of the necrotic tissues and amphotericin B. Despite aggressive treatment, the mortality remains high. (Choices C and D) Voriconazole and itraconazole are not effective against mucormycosis. (Choice B) Terbinafine is approved for the treatment of onychomycosis of the fingernails. It is not used for systemic fungal infections. (Choice E) Flucytosine is used in cryptococcal meningitis in combination with amphotericin B. It is a weak anti-fungal agent on its own. Educational Objective: Rhinocerebral mucormycosis is best treated with surgical debridement and intravenous amphotericin. 78% of people answered this question correctly. A 53-year-old African-American female is seen for her annual physical examination. She denies any symptoms, except for minimal hot flashes. She had menopause one year ago. Since then, she started having some menopausal symptoms, which have been gradually improving during the last two months. She has not been on any hormone replacement therapy. Her past medical history is unremarkable. She is currently on no medication. Her father has premature coronary artery disease. There is no history of cancer or osteoporosis in her family. She exercises regularly, and does not smoke or drink alcohol. Her mammogram and pelvic exam were performed eight months ago, and were reported to be normal. Her physical examination is unremarkable. What is the next best step in the management of this patient? A. Start her on hormone replacement therapy B. Start calcium and vitamin D supplementation C. Start low-dose alendronate D. Tell her to discontinue the exercise program E. Start raloxifene Explanation: Most females rapidly lose considerable bone mass following menopause. Lifestyle modification, including weightbearing exercises and optimum calcium and vitamin D supplementation, prevents postmenopausal bone mass loss and possibly reduces the risk of fragility fractures. The National Academy of Science recommends daily supplementation of elemental calcium (1200 mg) and vitamin D (400 to 800 international units) in women after 50 years of age. This can be easily achieved by taking one multivitamin tablet (which contains 400 international units of vitamin D) with two tablets of calcium/vitamin D (containing 600 mg of elemental calcium and 200 international units of vitamin D) everyday. (Choice D) Exercise with weightbearing leads to improvement in bone mass, and should be continued in this patient; however, exercise should be individualized in patients with significant osteoporosis, since they have a higher risk of fracture. In such patients, stressful exercises should be avoided as much as possible. (Choices C and E) Short-term use of low-dose estrogen for menopausal symptoms does not appear to be harmful; however, hormone replacement therapy fell out of favor following a recently published study of the women?s health initiative because of increased, undesirable cardiovascular and other side effects. This patient has minimal hypoestrogenic symptoms in the form of hot flashes, which are likely to improve with hormone replacement therapy; however, because her symptoms appear to improve with time, hormone replacement therapy could be possibly avoided. This patient does not

have any risk factors for osteoporosis, except for postmenopausal stature. There is no documentation of low bone density (osteopenia or osteoporosis). There is also no history of fragility fractures. Treatment with bone-specific antiresorptive therapy (e.g., alendronate, raloxifene) is not warranted at this time. Educational Objective: Calcium and vitamin D supplementation should be recommended in all postmenopausal females to protect bones. A 56-year-old female presents with lethargy, weight loss, nausea, vomiting, and constipation for the past two months. She is three years postmenopausal, and has no long-standing medical problems. She currently takes no prescription medications. She denies allergies to drugs. She does not use tobacco or alcohol. Her family history is positive for osteoporosis and hypertension. Her heart rate is 64/min and blood pressure is 124/66 mmHg. Her height is 5?3" (160cm) and she weighs 138 lbs (63kgs). She has mild pallor. The thyroid gland is normal. The chest is clear on auscultation. Heart sounds are normal. Neurological examination reveals hung-up ankle jerks. Abdominal examination is unremarkable. Laboratory tests reveal a hemoglobin of 11.2 g/dL, and a hematocrit of 34%. RBCs are normochromic and normocytic. Serum sodium is 129 mEq/dL , while the rest of the basic chemistries are normal. TSH is 0.35 mU/ml (normal 0.35-5.0 mU/ml) and free T4 is 0.5 mg/dl (normal 0.8 to 1.8 mg/dl). What is the next best step in this patient?s care? A. Measurement of antithyroid antibodies B. TRH stimulation test C. Serum T3 levels D. Measurement of cortisol with cosyntropin stimulation E. Thyroglobulin levels Explanation: The patient most likely has central hypothyroidism, as suggested by borderline TSH and low free T4 levels. Central hypothyroidism can also have slightly elevated TSH levels; however, this elevated TSH is not biologically active. Imaging studies (preferably a MRI) are done to look for a mass lesion in the sellar area. When central hypothyroidism is suspected, it is pragmatic to rule out concomitant central adrenal insufficiency before beginning therapy. Patients with primary adrenal insufficiency (Addison?s disease) can have increased TSH levels without hypothyroidism due to the loss of an inhibitory effect of glucocorticoids on TSH secretion. Conversely, a patient on high-dose glucocorticoid treatment will have a transient decrease in TSH levels. Treating a patient with central hypothyroidism and concomitant adrenal insufficiency could precipitate adrenal crisis. A cosyntropin stimulation test is the most preferred initial test to screen for adrenal insufficiency. (Choice A) Antithyroid antibodies are useful in the evaluation of goiter and primary hypothyroidism, but not suspected central hypothyroidism. (Choice B) After the advent of sensitive TSH assays, TRH stimulation test is now rarely performed. It still haa a role in differentiating isolated pituitary resistance to thyroid hormones (normal response) from TSH secreting pituitary tumors (diminished response). (Choice C) T3 level determination has no role in the evaluation of hypothyroid patients. (Choice E) Thyroglobulin level determination is useful in the follow-up management of patients with differentiated thyroid cancers, and in the evaluation of suspected factitious thyrotoxicosis. Educational Objective: Evaluation of the adrenal status is warranted in patients with suspected central hypothyroidism. 31% of people answered this question correctly.

A 65-year-old white female presents to the emergency department with a sudden onset of severe periumbilical pain, nausea and vomiting. She denies any fever, diarrhea, or constipation. She denies any urinary complaints. Her last menstrual period was fourteen years ago. Her past medical history is significant for coronary artery disease, a recent acute anterior myocardial infarction, hypertension, and hyperlipidemia. She had an appendectomy at the age of eighteen. She has a history of 20 pack years of smoking. She is an occasional alcohol drinker. Her temperature is 36.7C(98F), blood pressure is 120/76 mm Hg, pulse is 120/min, irregular, and respirations are 16/min. The abdomen is soft, non-distended and non-tender, with normal bowel sounds. There is no rebound tenderness or rigidity. Pelvic examination is within normal limits. EKG shows atrial fibrillation. CBC shows a WBC count of 14,000/microL. Serum sodium is 140 mEq/L, serum K is 4.0 mEq/L, blood glucose is 110mg/dL, BUN is 20 mg/dL, and serum creatinine is 0.7 mg/dL. The serum lipase is normal, while the serum amylase is slightly elevated. Urinalysis is normal. Liver function panel is also normal. Fecal occult blood test is negative. Plain abdominal x-ray fails to show any abnormalities. Which of the following is the most likely diagnosis of this patient? A. Intestinal obstruction B. Acute mesenteric ischemia C. Acute colonic ischemia D. Acute intermittent porphyria E. Acute diverticulitis Explanation: This patient is most likely suffering from acute mesenteric ischemia secondary to embolic phenomenon caused by atrial fibrillation. This patient also had a recent anterior myocardial infarction. Embolization usually throws clots into the superior mesenteric artery, resulting in mesenteric ischemia. Patients usually present with a sudden onset of periumbilical abdominal pain. The abdominal examination is initially normal, and the pain felt by the patient is usually out of proportion to the initial examination findings. (Choice A) Patients with intestinal obstruction usually have a history of absolute constipation. Plain abdominal radiographs will show air fluid levels. (Choice C) Patients with acute colonic ischemia usually have a more lateralized abdominal pain with tenderness, along with a history of bloody diarrhea or hematochezia. This usually results from hypovolemic states or transient ischemia to the bowel. This is not because of the obstruction of flow to the colon. (Choice D) Acute intermittent porphyria is a rare disease. Patients will usually have recurrent episodes of abdominal pain that is precipitated by several factors. The symptoms are usually manifested much earlier (usually after puberty), and occur in the presence of other manifestations such as neuropathy, hyponatremia, etc. (Choice E) Acute diverticulitis usually causes a more localized pain (LLQ). The physical exam would show tenderness in the LLQ. Patients may also present with diarrhea. Educational Objective: A sudden onset of severe periumbilical pain that is out of proportion to the physical examination findings, and occurs in a setting with risk factors such as atrial fibrillation, congestive heart failure, and peripheral vascular disease, should always raise the suspicion for acute mesenteric ischemia. 90% of people answered this question correctly. A 53-year-old asymptomatic, Caucasian woman comes to the physician for a routine health maintenance examination. She has no complaints; however, she is concerned about her risk of developing osteoporosis. She is an executive secretary with a sedentary lifestyle. Her last menstrual period was two years ago. She drinks coffee for at least five times daily, and soda at least twice daily.

She is a vegetarian. She does not use drugs. She has smoked one pack of cigarettes daily for 35 years. She drinks 1-2 ounces of alcohol daily. Her father has hypertension. Her mother had a stroke. She takes no medication. She has no known drug allergies. Examination shows no abnormalities. Which of the following interventions will be the most beneficial to decrease her risk for osteoporosis? A. Stop drinking alcohol B. Stop drinking coffee and soda C. Stop smoking D. Start hormonal replacement therapy E. Start an exercise program Explanation: Although the most important risk factors for osteoporosis are the non-modifiable ones (e.g., age, race, family history, and history of previous fractures), it is also necessary to know the modifiable factors in order to provide interventions which can lower a patient's risk for the disease. For instance, smoking half a pack of cigarettes a day has a mild negative effect on osteoporosis, but a smoker who consumes one or more packs of cigarettes a day will benefit more from smoking cessation than from any other intervention. (Choice D) Although hormonal replacement therapy (HRT) can effectively decrease a patient?s risk of osteoporosis, HRT can unfortunately increase the risk of breast and endometrial cancer. Some recent studies even show that it may also increase the risk of cardiovascular disease. (Choices A, B and E) Lack of exercise, alcohol, coffee, and soda ingestion mildly increase the risk of osteoporosis compared to menopause or smoking. Educational Objective: There are multiple risk factors for osteoporosis, the majority of which are non-modifiable. The interventions that have proven to offer more benefit are: smoking cessation (if the patient smokes one pack or more of cigarettes a day), raloxifene, calcium and vitamin D supplementation, and discontinuation of medications such as heparin or glucocorticoids. HRT is extremely effective in osteoporosis prevention, but widespread use is not advisable because of the risk of malignancy or cardiovascular disease. 39% of people answered this question correctly. The following vignette applies to the next 3 items A healthy 8-year-old Caucasian boy is brought to the office by his mother because of redness and copious, purulent discharge from his right eye. The school nurse informed the mother that the boy's disease is very contagious. The boy denies fever or blurred vision. His vital signs are within normal limits. Examination shows conjunctival erythema and yellow exudates in his right eye. The conjunctiva of the left eye is also erythematous, but there are no secretions. The mother is especially concerned about the possible complications of the disease. Item 1 of 3 Which of the following complications is most likely to occur with his condition? A. Endophthalmitis B. Nothing C. Recurrence D. Cataracts E. Keratitis Explanation: This is a typical presentation of bacterial conjunctivitis. increased risk of a corneal lesion in the form of keratitis.

When the infection is severe, there is an

(Choice B) Although this is a self-limiting disease in the majority of situations, this is not always true. (Choice A) Endophthalmitis is acquired through an open wound to the eye (post-cataract surgery, post-traumatic) or a systemic route, and not usually through conjunctivitis. (Choice D) Cataracts have not been described as complications of this disease. (Choice C) Recurrence is only possible if the patient has bad hygienic habits or if he acquires viral conjunctivitis. Educational Objective: Bacterial conjunctivitis is usually a self-limiting disease; however, it must be treated because there is a small but real risk of keratitis, which can lead to visual impairment. 34% of people answered this question correctly. Item 2 of 3 Which of the following is the most appropriate pharmacotherapy? A. Ciprofloxacin eye drops B. Ofloxacin eye drops C. Antihistamine eye drops D. Corticosteroid eye drops E. Erythromycin ointment Explanation: Erythromycin ointment or sulfa drops are the first line agents for the treatment of uncomplicated bacterial conjunctivitis, as these cover most organisms. (Choices A and B) Fluoroquinolones are preferred for contact lens wearers and corneal ulcers because of their activity against Pseudomonas. These should not be used as first line agents in uncomplicated bacterial conjunctivitis because of the risk of the emergence of resistance. (Choice C) Antihistamines are used for allergic conjunctivitis, which is characterized by watery discharge and usually involves both eyes. (Choice D) Primary care physicians should not prescribe corticosteroids, as these can cause sightthreatening complications in patients with bacterial conjunctivitis and herpes keratitis. Educational Objective: Erythromycin ointment or sulfa drops are the first line antibiotic agents for the empirical treatment of patients with bacterial conjunctivitis. 53% of people answered this question correctly. Item 3 of 3 The appropriate evaluation was performed. The mother wants to know when her son can go back to school. What is the best response? A. One week from now. B. This is not contagious; he can go to school tomorrow. C. He should stay at home until the discharge has cleared. D. With the treatment, he can go to school tomorrow . E. He has to wait at least 2 weeks from now and will have to be reevaluated by me. Explanation:

Bacterial conjunctivitis is very contagious. In such cases, the best thing to do is to keep the child at home until the discharge has cleared. However, this may not be feasible for many patients, especially for those with work/demanding occupations. In those patients, at least 24-hours of topical antibiotic therapy should be applied before returning to work. Educational Objective: Bacterial conjunctivitis is very contagious. Patients should be advised to stay home until the discharge has cleared. If this is not possible, patients should have had at least 24 hours of topical antibiotic therapy before returning to work/school. A 55-year-old African-American man is brought to the emergency department because of an episode of hematemesis. He has a history of alcoholic cirrhosis with ascites for the last two years. In the emergency room, he is appropriately treated, and has no new episodes of bleeding. He is admitted to the hospital for observation and further work-up. His upper GI endoscopy reveals the presence of esophageal varices with stigmata of recent bleeding. Which of the following is the most likely complication that he is at risk of developing during his hospitalization? A. Spontaneous bacterial peritonitis B. Renal failure C. Congestive heart failure D. Disseminated intravascular coagulopathy E. Hemolytic uremic syndrome

Explanation: Patients who are admitted to the hospital because of recent variceal bleeding are at an increased risk of developing complications during their hospitalization. The principal complications in these patients that lead to increased mortality are: infections, hepatic encephalopathy, and renal failure. The most common complication is the development of an infection, which usually occurs as a urinary tract infection, spontaneous bacterial peritonitis, respiratory infection, aspiration pneumonia or primary bacteremia. A large number of trials have evaluated the efficacy of prophylactic antibiotics in cirrhotic patients that were hospitalized for variceal bleeding. All these trials have suggested a decreased incidence of infectious complications with the use of prophylactic antibiotics. The optimal choice of antibiotics and the duration of therapy remain unclear. The currently preferred regimen is the use of a fluoroquinolone (ofloxacin, norfloxacin, or ciprofloxacin) agent for 7-10 days. (Choice B) The development of renal failure in these patients can have multiple etiologies, such as acute tubular necrosis (ischemic or toxic) or precipitation of hepatorenal syndrome; however, this complication is not as common as the development of an infection. (Choice C) Congestive heart failure is uncommonly seen in these patients. (Choice D) Disseminated intravascular coagulopathy occurs rarely in patients with acute variceal hemorrhage, and is usually seen in the presence of underlying infections. (Choice E) Hepatorenal syndrome is a complication of cirrhosis and a variceal bleed. Hemolytic uremic syndrome (HUS) is associated with Shiga toxin-producing Escherichia coli (E. coli O157:H7), and is not associated with cirrhosis. Educational Objective: Bacterial infections can develop in up to 50% of patients who are hospitalized for acute variceal bleeding; therefore, these patients should be treated prophylactically with antibiotics. The preferred regimen involves the use of a fluoroquinolone (ofloxacin, norfloxacin, or ciprofloxacin) agent for 7-10 days. 70% of people answered this question correctly. A 17-year-old, Caucasian, female, college student presents to the student health center on a Monday morning after having unprotected intercourse during a party the last Saturday evening. She is concerned about getting pregnant and requests contraception. She has been sexually active with two partners over the last several months, but her partners used condoms. Her past medical history is insignificant. She is currently not taking any medications. She does not smoke, but consumes alcohol occasionally. She confessed that she has tried several recreational drugs, but does not remember their names. The physical examination is insignificant. Which of the following is the best contraception modality for this patient? A. High dose estrogen B. Ethinyl estradiol plus levonorgestrel C. Levonorgestrel D. Copper intrauterine device E. Reassurance and no intervention Explanation: Levonorgestrel is the recommended method of emergency contraception if used soon enough after an unprotected sexual intercourse. It has maximal efficacy when used within the first 12 hours after intercourse, good efficacy within 48 hours, and appears to work up to 120 hours after intercourse. (Choices B and A) Levonorgestrel seems to be more effective than ethinyl estradiol /levonorgestrel combination or high dose estrogen. It also has fewer side effects. One controlled trial showed that

levonorgestrel prevents 85% of expected estrogen/progesterone combination).

pregnancies

(compared

to

57%

prevented

by

(Choice D) The copper intrauterine device is an effective emergency contraception tool that can be used if a patient presents more than 120 hours after unprotected intercourse. Educational Objective: Levonorgestrel is the recommended method of emergency contraception if used soon enough after an unprotected sexual intercourse. It has maximal efficacy when used within the first 12 hours after intercourse, good efficacy within 48 hours, and appears to work up to 120 hours after intercourse. The copper intrauterine device is an effective emergency contraception tool that can be used if a patient presents more than 120 hours after unprotected intercourse. 30% of people answered this question correctly. A 9-year-old Caucasian boy is brought to your office with a two-month history of periodic clear rhinorrhea, nasal and orbital itching, and dry cough. His mother notes that the symptoms usually appear after the child plays with a neighbor?s dog. His past medical history is significant for an episode of severe pneumonia experienced two years ago. Physical examination reveals excoriations of the external nares. The lungs are clear on auscultation. Which of the following is the next best step in the management of this patient? A. Avoidance of the dog B. Nasal decongestant sprays C. Nasal corticosteroids D. Second-generation antihistamines E. Cromolyn sodium Explanation: This patient presents with signs and symptoms suggestive of allergic rhinitis. The neighbor?s dog is the most likely allergen trigger for this patient?s condition. Dog or cat dander is one of the most common identifiable allergen in patients with allergic rhinitis. Allergen avoidance is traditionally considered the first step in the management. (Choice C) If the allergen is not identified, or if the symptoms persist after avoidance measures, nasal corticosteroids should be the first-line therapy. (Choices D and E) Second-generation antihistamines and cromolyn are less effective than topical steroids. (Choice B) Nasal decongestive sprays are not recommended because tachyphylaxis usually develops, and rebound phenomena may result. Educational Objective: When possible, avoidance of the antigen triggers is typically the first step in the management of allergic disorders. 75% of people answered this question correctly. A 23-year-old Caucasian female has just delivered a 9 lb healthy baby at 37 weeks gestation thirty minutes ago. Although the labor was prolonged, the baby and placenta were delivered spontaneously, without application of forceps. Even after placental delivery, the patient is bleeding continuously. She has lost approximately 1.5 liters of blood. IV ringers lactate is administered. Her temperature is 36.7 C (98 F), blood pressure is 100/60 mmHg, pulse is 102/min, and respirations are 18/min. Which of the following is the most appropriate next step in the management of this patient?

A. Intravenous oxytocin B. Pelvic examination C. Observation D. Bimanual uterine massage E. Uterine artery ligation

Explanation: The most common cause of post partum hemorrhage (PPH) is uterine atony, and the first step in the management of all patients with PPH is to do a pelvic examination to identify any retained placental fragments. (Choice D) If the retained placental products are not identified, manual uterine massage should be started. Uterine massage stimulates the uterus, and the resulting contractions stop the bleeding. (Choice A) If bimanual uterine massage fails to control bleeding, uterotonic drugs (e.g., IV oxytocin) are used to control the bleeding. (Choice E) If bleeding does not stop with medical measures, surgical measures are taken. Uterine artery ligation is one of the surgical measures for the treatment of PPH. (Choice C) PPH is a serious obstetrical complication. Measures should be taken immediately to stop the bleeding. Observation alone may be fatal. Educational Objective: Pelvic examination to look for any retained placental products is the first step in the management of PPH. 20% of people answered this question correctly. An 83-year-old Caucasian man comes to the emergency department because of a sudden onset, transient visual loss in his right eye. He is currently symptom-free, and denies any other new symptoms. His other medical problems include hypertension and peripheral vascular disease. His medications include aspirin, hydrochlorothiazide, and enalapril. His temperature is 36.7 C (98 F), blood pressure is 160/90 mmHg, pulse is 80/min., and respirations are 12/min. Which of the following signs is most likely to be found during the physical examination? A. Systolic heart murmur B. Diastolic heart murmur C. Papilledema D. Hemorrhagia exudates in the retina E. Carotid bruit Explanation: This patient has developed amaurosis fugax, an acute ischemic event involving the retinal artery. This condition is sudden and usually reversible. It is a marker of carotid artery atherosclerotic disease. A carotid bruit on auscultation of the neck is a very frequent finding; this indicates a significant carotid obstruction. (Choice B) A diastolic heart murmur can be a marker of aortic insufficiency or mitral stenosis. Both conditions are related to ischemic cerebrovascular accidents (CVA). Although these could occur, this elderly patient's transient symptom is more likely to be concurrent with findings suggestive of carotid artery disease. (Choice A) Conditions producing systolic heart murmurs such as mitral regurgitation, aortic stenosis, or tricuspid regurgitation, are more related to heart failure symptoms. Even though aortic stenosis can be related to syncope, it does not cause amaurosis fugax . (Choice D) Acute retinal vein thrombosis can be diagnosed by fundoscopy, when extensive hemorrhage is seen in the retina. It will not present as amaurosis fugax.

(Choice C) Papilledema is seen as a result of increased intracranial pressure. Pseudopapilledema, on the other hand, may be seen in an established nerve damage or in optic neuritis, but not in amaurosis fugax. Ischemia will manifest as disc paleness, not papilledema, especially in a transient event.

Educational Objective: Amaurosis fugax is a sudden and transient monocular blindness. Even though it is reversible, it is a marker of carotid artery atherosclerotic disease that is usually advanced. Physical examination will often reveal a carotid bruit. Carotid Doppler evaluation is necessary to evaluate the extent of the disease and to assess the need for a carotid endarterectomy. 86% of people answered this question correctly. The mother of a two-year-old girl comes to your office. She is extremely anxious and wants you to see her daughter right away. She tells you that her daughter came back two months ago after spending six weeks with her grandmother in a ranch in Texas. The grandmother was recently hospitalized, diagnosed with pulmonary tuberculosis, and prescribed six months of antitubercular therapy. The mother is concerned about her daughter?s risk of contracting the illness, and asks for your advice. The girl does not have any new symptoms suggestive of active tuberculosis. Which of the following should be your next step in management? A. Reassure the mother and do nothing at this point. B. Perform a tuberculin skin test on the daughter. C. Obtain a plain chest x-ray. D. Schedule a serial followup examination of the patient every month. E. Prescribe a weight-based isoniazid therapy for nine months. Explanation: Tuberculosis is spread from person to person through airborne droplets. The risk of spread of latent or active tuberculosis is greatest when there is a history of contact with patients who have active (sputum-positive) pulmonary tuberculosis. Once infected, a person?s body mounts a cell-mediated immune response which is detected by a tuberculin skin test. This is why the first step in evaluating an asymptomatic person with recent significant exposure to tuberculosis patients is to obtain a tuberculin skin test. If the initial test result is negative, a second test should be performed 10 weeks after the last known exposure. Tuberculin skin testing detects the presence of latent tubercular infection. The treatment of latent tubercular infection should be strongly considered in all patients, especially in children less than 15 years of age and in HIV-positive patients. (Choice A) A person exposed to a known tuberculosis patient has the greatest risk of developing tuberculosis within the next two years. This person should receive a tuberculin skin test to document the presence of latent tuberculosis. Reassurance is not appropriate at this point. (Choices C and D) The majority of primary pulmonary tubercular infections are radiographically and clinically silent. In most cases, a positive tuberculin skin test is the only indication that infection with Mycobacterium tuberculosis has occurred. (Choice E) Isoniazid therapy for nine months should be recommended for all patients with a positive PPD or recent tuberculin skin test conversion (at least 10 mm increase in the skin reaction within a two-year period). It is important to document latent tubercular infection with a positive tuberculin skin test before prescribing isoniazid therapy. It is also important to document that the patient does not have active tuberculosis, in which case she should be treated with a full course of antitubercular therapy. Educational Objective: Tuberculin skin testing should be initially performed in all persons exposed to patients with tuberculosis. 85% of people answered this question correctly.

A 76-year-old gravida 4, para 4 woman presents to the clinic complaining of pelvic pressure and heaviness, lower back pain, and constipation. These symptoms were gradual in onset and have been present for a period of months. Her past medical history is significant for mild congestive heart failure, emphysema, hypertension, and peptic ulcer disease. She has a 52 pack-year smoking history and occasionally drinks bourbon with her friends. Her vital signs are normal. The physical examination reveals a moderately overweight woman in no apparent distress. The gynecologic examination demonstrates protrusion of the posterior vaginal wall that is most prominent with bearing down while in the lithotomy position. The patient is diagnosed with rectocele, and surgical repair is recommended. The woman declines to have surgery and remains steadfast on this point. What is the most appropriate response in this situation? A. Recommend oral hormone replacement therapy B. Recommend pelvic exercises C. Recommend pessary use D. Convince her to undergo surgical repair E. Inform the patient she needs to find a new physician Explanation: Rectocele is a relatively common condition in older women and is characterized by the displacement of the rectum through posterior vaginal wall defect(s). The condition is typically caused by damage to the rectovaginal septum incurred during vaginal childbirth and is exacerbated by periodic increases in intraabdominal pressure (e.g., when laughing or coughing) and the effects of gravity. Women with symptomatic rectoceles who are poor surgical candidates may be treated with pessaries, which are structures designed to support the vaginal wall. Pessaries should only be used in conjunction with vaginal estrogen; without it, these can cause chronic discharge and bleeding secondary to injury of the vaginal tissues. (Choice A) Although estrogen creams are useful in conjunction with pessary usage, there is no indication for oral hormone replacement therapy in the treatment of rectoceles. (Choice B) Pelvic exercises are appropriate to recommend in women with asymptomatic rectocele. Additional recommendations for this patient group include avoidance of activities related to increased intraabdominal pressure and regular usage of intravaginal estrogen to prevent tissue atrophy. (Choice D) Surgical repair, most commonly via a posterior colporrhaphy, is an appropriate recommendation for women with symptomatic rectoceles. However, it is important to advise these patients that correction of the condition does not always provide symptomatic relief. Patients who are not good surgical candidates or who prefer not to undergo surgery should not be pressured to proceed. Rather, they should be advised about alternative treatment options (e.g., pessaries). (Choice E) There is no need to deny this patient further care simply because she does not want to undergo surgery. Since there is a reasonable alternative available, and since she is an elderly woman with numerous serious medical conditions, it is preferable to avoid elective surgical procedures. Educational Objective: Women with symptomatic rectoceles who are poor surgical candidates may be treated with pessaries in conjunction with estrogen cream. 70% of people answered this question correctly. The following Vignette applies to the next 5 items A 26-year-old Italian-American man comes to your office for the evaluation of low back pain. He has had this pain for the last eight months, and it is progressively getting worse. The pain is worse in the morning and gets better as the day progresses. He denies any history of trauma in the past. He works as a chef at a local restaurant. His father and uncles also have "back problems." His physical

examination reveals a limited range of motion of his lumbosacral spine and markedly reduced chest expansion. The rest of his physical and musculoskeletal examination is unremarkable.

Item 1 of 5 Which of the following is the most appropriate next step in the management of this patient? A. HLA-B 27 levels B. ANA and rheumatoid factor C. Bone scan D. A MRI of the spine E. X-ray of the sacroiliac joint Explanation: The patient has typical features of ankylosing spondylitis. These include back pain with morning stiffness which improves with exercise, insidious onset, presence of symptoms for more than three months, reduced range of forward flexion of the lumber spine on Schober testing, and reduced chest expansion. A diagnosis of ankylosing spondylitis cannot be made unless there is evidence of sacroiliitis, which is the earliest change that is seen radiographically. Therefore, when a patient has typical features of ankylosing spondylitis, a plain x-ray of the sacroiliac joint is the next step to establish the diagnosis. Other radiographic abnormalities suggesting the diagnosis of ankylosing spondylitis include erosions of the ischial tuberosity and iliac crest. Early in the course of disease, squaring of the vertebral disease on plain x-ray may also suggest the diagnosis. (Choice D) When ankylosing spondylitis is strongly suspected clinically and radiographic findings of plain x-ray are negative or equivocal, a CT scan is indicated to establish the diagnosis. MRI may also be used for this purpose, but a CT scan is usually recommended in such setting. (Choice A) HLA-B 27 is frequently (>90% of the patients) present in patients with ankylosing spondylitis and other spondyloarthropathies. Its presence is not specific for the diagnosis of ankylosing spondylitis; however, its absence (negative for HLA B 27) makes the diagnosis of ankylosing spondylitis unlikely. (Choice B) Testing for ANA and rheumatoid factor is not the next appropriate step when a patient has a history typical of ankylosing spondylitis. ANA and rheumatoid factor are absent in ankylosing spondylitis and other seronegative spondyloarthropathies. (Choice C) A bone scan has no role in the diagnosis of ankylosing spondylitis. Educational Objective: The three important clinical criteria for the diagnosis of ankylosing spondylitis are: Presence of low back pain and stiffness for more than a three month duration that improves with exercise or activity. Limitation of the range of motion of the lumbar spine. Limitation of chest expansion relative to the normal values. Plain x-ray of the sacroiliac joint is the next best step in the management of a patient who is suspected with ankylosing spondylitis. 63% of people answered this question correctly. Item 2 of 5 If the initial evaluation shows mild disease, what would you do to monitor the disease activity? A. Repeat x-rays after 3 months B. Repeat MRI in 3 months C. Repeat bone scan in 3 months D. Repeat HLA-B 27 levels in 3 months E. Repeat ANA and rheumatoid factor levels in 3 months Explanation:

The following radiographs are used in monitoring the disease progression of patients with ankylosing spondylitis: 1. Anteroposterior and lateral views of the lumbar spine 2. Lateral view of the cervical spine 3. Pelvic radiograph, including the sacroiliac joints and hip Other acute phase reactants such as ESR can also be used. (Choice B) CT scans are almost never used to monitor the disease activity. MRI is also not used to monitor the disease activity, unless the patient develops complications such as cauda equina syndrome. Educational Objective: Radiographs and acute phase reactants (i.e., ESR) are used to monitor the disease progression of patients with ankylosing spondylitis. 59% of people answered this question correctly. Item 3 of 5 Which of the following conditions is associated with the patient?s diagnosis? A. Restrictive lung disease B. Acute narrow angle glaucoma C. Coarctation of aorta D. Early development of testicular cancer E. Sclerosing cholangitis Explanation: Once a diagnosis of ankylosing spondylitis is being considered, it is important to search for concurrent extraarticular manifestations. Although they are usually asymptomatic in the initial stages, many patients have restrictive lung disease due to the limited costovertebral joint motion and development of apical pulmonary fibrosis. It is important to counsel such patients regarding smoking cessation to prevent early deterioration of lung function. (Choice B) Ocular manifestations with AS include acute anterior uveitis, cataracts, and cystoid macular edema. Uveitis presents with acute onset unilateral eye pain, photophobia, and blurring of vision. It is sometimes the initial presentation of ankylosing spondylitis. (Choice C) The most common valvular abnormalities associated with ankylosing spondylitis include aortic regurgitation and mitral valve prolapse. The incidence of coarctation of the aorta and septal defects is not increased in these patients. (Choice D) There is an increased prevalence of varicocele, not testicular cancer, in these patients. (Choice E) The gastrointestinal manifestations are nonspecific ileal and colonic mucosal ulcerations. Some other extraarticular features include atlanto-axial subluxation causing spinal cord compression, cauda equina syndrome, IgA nephropathy, and secondary amyloidosis presenting as nephrotic syndrome. Educational Objective: The most common and important extraarticular manifestations of ankylosing spondylitis are acute anterior uveitis, aortic regurgitation, apical pulmonary fibrosis, IgA nephropathy, and restrictive lung disease. 58% of people answered this question correctly. Item 4 of 5

The patient is given a prescription for non-steroidal anti-inflammatory agents. He returns for a followup visit after two weeks, and says that he feels a lot better. His pain and stiffness have almost resolved with daily intake of medications. He used to swim and exercise regularly before his symptoms developed a few months ago, and he wants to resume these activities, but he is concerned that his disease may get worse if he does so. Which of the following is the most appropriate response? A. Aerobic exercises will accelerate the joint destruction B. Extension exercises of the spine can be harmful C. He should not resume exercise for at least 1 year D. Swimming should be avoided as it increases the strain on back muscles E. Aerobic exercises improve overall functional status without increasing the disease activity

Explanation: Regular aerobic exercises such as swimming, walking, and bicycling improve joint stability, muscle strength, and overall functional status without an increase in the disease activity. These also help to prevent bone loss due to immobility and use of medications. The exercise regimen should be tailored to the individual, depending on the exercise tolerance, extent and severity of joint disease, and other coexisting medical conditions. An initial evaluation by a physical therapist should be a part of therapy for ankylosing spondylitis. (Choice A) Supervised physical therapy and exercises can lead to significant improvement of the range of motion of the spine. It does not lead to acceleration of the joint destruction. (Choice B) In ankylosing spondylitis, postural training and extension exercises help to prevent spine fusion in a flexed position. (Choice C) There is no reason to avoid exercise for a year. The patient should be evaluated by a physical therapist and initiate an exercise program as early as possible. (Choice D) Swimming (hydrotherapy) is a type of aerobic exercise with special benefits for patients with rheumatologic disorders. It is associated with less joint stress and weight bearing and is useful for patients with advanced disease. Educational Objective: Aerobic exercises improve overall functional status of patients with rheumatologic disorders and should be encouraged. 81% of people answered this question correctly. Item 5 of 5 Which of the following is the most appropriate statement regarding the prognosis of this condition? A. Overall mortality is increased in this patient population B. Most patients will have functional and employment disability C. Smoking is surprisingly protective in this population D. Life expectancy is not reduced E. Prolonged standing at the work place is protective Explanation: There are no studies that have demonstrated an increased overall mortality or reduced life expectancy in patients with ankylosing spondylitis. Most patients do well and have no functional or employment disabilities. (Choice C) Smoking cessation should be strongly advised, as this is associated with a very bad prognosis. (Choice E) Prolonged standing at work or exposure to cold conditions are risk factors for disability. Educational Objective: Most patients with ankylosing spondylitis do well and have no functional or employment disabilities. There is no increased overall mortality or reduced life expectancy. 46% of people answered this question correctly.

The following vignette applies to the next 3 items A 45-year-old Caucasian woman comes to the emergency department (ED) with complaints of severe abdominal pain. She started having pain in her upper abdomen approximately twelve hours ago. The pain is sharp and radiates to her back. She had two episodes of vomiting before coming to the ED. She has been hospitalized four times in the past with similar complaints. On review of her previous hospitalizations, you notice that there was never a specific cause found for her symptoms. She has a history of diabetes and hypertension, and both are well controlled by diet and exercise. Her blood pressure is 130/70 mmHg, pulse is 92/min, temperature is 36.7C (98F), and respirations are 16/min. She is moderately obese, and is in distress. Her lungs and cardiovascular examination are within normal limits. There is diffuse tenderness to palpation over the whole abdomen, with maximum tenderness present at the epigastric area. Item 1 of 3 Which of the following is the next step in the diagnosis of the patient?s condition? A. Serial abdominal examination B. CT scan of the abdomen C. LFTs with lipase levels D. No further tests are needed at this point as prior workup was normal E. Ultrasound of the right upper quadrant Explanation: The clinical presentation of the patient is consistent with acute pancreatitis, which is characterized by an acute onset of steady, upper abdominal pain radiating to the back, with associated nausea and vomiting. However, since these symptoms are nonspecific and can be seen in a number of acute abdominal illnesses, confirmation of the diagnosis is necessary with clinical and biochemical markers, as well as radiographic imaging. A variety of biochemical tests have been devised to diagnose acute pancreatitis. Of these, measurement of the serum amylase and lipase levels are the most frequently used. Serum lipase is more sensitive and specific than serum amylase; therefore, measurement of the former is the diagnostic test of choice. (An elevated serum amylase level is nonspecific as it is also elevated in a number of other conditions such as acute parotitis, intestinal disorders, renal failure, cholecystitis, and fallopian tube diseases.) In addition, a liver function test (LFT) is also obtained since an elevated alkaline phosphatase level may point towards gallstone/common bile duct stones as the etiology of the acute pancreatitis. (Choice A) Serial abdominal examination can delay the diagnosis of acute pancreatitis, and may even prove to be potentially fatal in these patients. It is only useful to follow the clinical progress of the patient once the diagnosis has been made. (Choice B) CT scan may be eventually needed to document the severity of pancreatitis and to detect the presence of other intra-abdominal complications. These should only be done if all other biochemical markers fail to provide a diagnosis, or if the patient fails to improve with initial conservative treatment. (Choice D) Her current symptoms should not be assumed to be benign based on her past history of hospitalizations. (Choice E) An ultrasound of the right upper quadrant of the abdomen may provide a clue to the cause of her symptoms; however, up to one-third of patients have bowel or intestinal gas that may obscure the pancreas, and thus, the diagnosis. Educational Objective: The diagnosis of acute pancreatitis is confirmed initially with an elevated level of serum biochemical markers (amylase or lipase).

*Extremely important question for USMLE step-3 56% of people answered this question correctly.

Item 2 of 3 The initial lab results of the patient reveal the following: Total bilirubin: 1.2 mg/dL Direct bilirubin: 1.0 mg/dL Alkaline phosphatase: 382 units/L AST: 40 units/L ALT: 38 units/L Albumin: 4 g/dL Serum calcium: 8.9 mg/dL Amylase: 1026 mg/dL Lipase: 662 mg/dL A right upper quadrant ultrasound reveals an enlarged hypoechoic area in the head of the pancreas with mild dilatation of the common bile duct. A contrast-enhanced CT scan of the abdomen reveals enlargement and inflammation of the pancreatic head without any areas of necrosis. Which of the following is the best next step in the management of this patient? A. Intravenous antibiotics B. CT-guided needle aspiration of the pancreatic tissue C. Daily abdominal CT scans D. IV fluids and nasogastric tube aspiration E. Surgical consult Explanation: Most episodes of acute pancreatitis are mild, and patients usually recover in five to seven days. Treatment is usually conservative (i.e., supportive therapy) and aimed at correcting the underlying predisposing condition and preventing any further damage to the pancreatic tissue. Patients can have a significant amount of third space loss of fluids, thereby causing hypotension, acute renal failure and even worsening of the pancreatitis; therefore, all patients should have early and aggressive fluid resuscitation with close monitoring of the urine output to ensure adequate tissue perfusion. Pain control is also important, and is generally achieved with the use of intravenous narcotics. Lastly, patients are kept off an oral diet (NPO), and their gastric contents are aspirated via a nasogastric tube (NGT) to prevent further pancreatic stimulation. Based on the radiographic imaging, the patient is suffering from a mild, edematous, acute pancreatitis. The next best step in management is nasogastric tube (NGT) aspiration and administration of IV fluids. (Choice A) Intravenous antibiotics are indicated only in patients with severe necrotizing pancreatitis or in patients with clinical or tissue evidence of infection of the necrotic pancreatic tissue. These are not given to patients with mild attacks of acute pancreatitis. (Choice B, C) There is no indication for CT-guided aspiration of the pancreatic tissue or daily CT scans. (Choice E) Surgical d?idement is indicated only in patients with extensive tissue necrosis or localized abscess formation. Educational Objective: Most of the patients with mild acute pancreatitis can be managed conservatively by adequate pain control and intravenous fluid resuscitation. *Extremely important question for USMLE step-3 77% of people answered this question correctly.

Item 3 of 3 While you are making your rounds the next morning, the nurse informs you that the patient has developed a temperature of 39.4C (103?F). The rest of her vital signs are within normal limits. Which of the following is the next best step in the management in this patient? A. Obtain blood cultures B. Obtain blood cultures and start the patient on imipenem C. Obtain urine cultures D. Repeat a stat CT scan E. Start the patient on intravenous ampicillin Explanation: The occurrence of fever in a previously afebrile patient with acute pancreatitis is one of the earliest signs of pancreatic infection. Infection of the pancreatic tissue is the major cause of morbidity and mortality in patients with acute pancreatitis. Most infections are seen late in the clinical course of the disease. There is considerable evidence from several studies that the early use of antibiotics in patients with severe necrotizing pancreatitis or in patients with evidence of pancreatic infection improves the outcome and reduces mortality. It is important to use a broad-spectrum antibiotic that can achieve good penetration in the pancreatic tissue. Examples of such antibiotics are: imipenem, third generation cephalosporins, piperacillin, fluoroquinolones, and metronidazole. If the patient?s condition fails to improve after one week of antibiotic therapy, a CT-guided aspiration of the tissue should be performed to obtain tissue samples for culture and sensitivity. (Choices A and C) It is important to start the patient on antibiotics as soon as the cultures are drawn. Obtaining blood or urine cultures alone is not sufficient. (Choice D) Another CT scan is unlikely to change the treatment at this point. The patient should be started on broad-spectrum antibiotics (imipenem) and followed closely for signs of clinical improvement or deterioration. (Choice E) Ampicillin has a poor tissue penetration into the pancreas, and there is considerable drug resistance against its action by most of the gut bacteria which are associated with pancreatic infection. Educational Objective: All patients with severe necrotizing pancreatitis or suspected pancreatic infection should be started on imipenem to decrease the morbidity and mortality associated with the disease. People rarely use a combination of ampicillin, gentamycin, and metronidazole. *Extremely important question for USMLE step-3 68% of people answered this question correctly. A 26-year-old Caucasian male presents to the emergency department with a three day history of dull back pain, progressive difficulty in walking, and urinary retention. He had a mild upper respiratory infection one week ago, but he is otherwise healthy. He is not taking any medications and denies drug abuse. His temperature is 36.7 C (98 F), blood pressure is 120/76 mmHg, pulse is 80/min, and respirations are 16/min. Neurologic examination reveals a decreased muscle strength (2/5) bilaterally in the lower extremities, hyporeactive knee and ankle reflexes, and decreased pain sensation up to the umbilical level. Which of the following is the most likely diagnosis of this patient? A. Herniated intervertebral disk B. Epidural abscess C. Acute subdural hemorrhage

D. Malignancy E. Transverse myelitis

Explanation: Rapidly progressive weakness of the lower extremities following an upper respiratory infection, accompanied by sensory loss and urinary retention, is characteristic for transverse myelitis. Dull back pain may be present. Neurologic examination initially reveals muscle flaccidity and hyporeflexia, but spasticity and hyperreflexia develop subsequently. In patients presenting with acute transverse myelopathy, other causes (especially compressive lesions) should be ruled out by obtaining a careful history and performing the appropriate imaging procedures. (Choice A) A herniated intervertebral disk is characterized by an abrupt onset of neurologic deficit and may be related to strenuous activity. A straight-leg raising test will be positive. (Choices B and C) Although an epidural abscess or an acute subdural hemorrhage may develop in otherwise healthy individuals, an underlying cause is usually present (e.g., IV drug abuse or treatment with anticoagulants). (Choice D) Spinal cord compression from a metastatic tumor may present acutely, but the history of systemic malignancy is typical. Educational Objective: Rapidly progressive weakness of the lower extremities following an upper respiratory infection, accompanied by sensory loss and urinary retention, is characteristic for transverse myelitis. 78% of people answered this question correctly. A healthy 29-year-old Caucasian woman comes to the physician for contraception counseling. She has had two unplanned pregnancies, and doesn?t want to have more. She has no medical problems. She does not use tobacco, alcohol, or drugs. She takes no medications. Her friend recommended a contraceptive method named "Norplant." The patient is concerned about the risks that this method could pose to her health. Which of the following complications is most likely to occur in a patient who uses this method? A. Vaginal spotting B. Thromboembolism C. Rash D. Menorrhagia E. Breast cancer Explanation: "Norplant" consists of six capsules of levonorgestrel which are placed subdermally, generally in the upper arm. It offers contraceptive protection for about five years. The most common complication is menorrhagia (prolonged vaginal bleeding during each period), which occurs in about 28% of the cases. (Choice A) Vaginal spotting is the second most frequent complication, affecting around 17% of the patients. (Choices B, C, and E) "Norplant" increases the risks of thromboembolism and breast cancer. It can also produce a rash. However, all these three complications are not frequent, and occur in less than 10% of the cases. Educational Objective: The use of "Norplant" is associated with menorrhagia and vaginal spotting. Other possible but less common complications are: venous thromboembolism, myocardial infarction, pulmonary embolism, thrombotic thrombocytopenic purpura (TTP), stroke, and breast cancer. 18% of people answered this question correctly.

A 24-year-old male presents to your office with a one-week history of night sweats and progressive fatigue. He also complains of palpitations and shortness of breath on moderate exertion. His past medical history is insignificant. His blood pressure is 120/80 mmHg, pulse is 98/min, temperature is 38.9C (102F) and respirations are 24/min. Lung auscultation reveals scattered bilateral rales. A grade II/VI blowing systolic murmur that increases with respiration is heard over the lower part of the left sternal border. The chest x-ray demonstrates several circumscribed round pulmonary infiltrates on both sides. Transesophageal echocardiography reveals small vegetations confined to the tricuspid valve. What is the most likely cause of this patient?s problem? A. Rheumatic fever B. Rheumatoid arthritis C. Illicit drug use D. SLE E. Congenital valve defect Explanation: The clinical scenario is highly suggestive of infective endocarditis involving the right cardiac chambers. Two very important clues to the diagnosis are: evidence of right-sided cardiac involvement (systolic murmur with inspiratory accentuation and tricuspid vegetation on echocardiography) and evidence of septic pulmonary emboli (present in 75% of patients with right-sided endocarditis). Peripheral manifestations and heart failure are uncommon in these patients, and a vast majority are IV drug users. The endocarditis seen in drug users is different from non-drug users in that the tricuspid valve is involved in 30 to 70% of endocarditis cases in injection drug users. Overall, rightsided endocarditis (predominantly involving the tricuspid valve) is far less common than left-sided disease. (Choices A and D) Rheumatic fever and SLE predominantly involve left cardiac structures. Isolated lesions of right cardiac valves are quite uncommon with these diseases. Educational Objective: A vast majority of patients with right-sided endocarditis are IV drug users. 94% of people answered this question correctly. After drawing blood from an HIV-positive patient, a 28-year-old Caucasian female nurse accidentally pricks her finger with the needle. Recent laboratory testing of the patient demonstrated a CD4 count of 410/?L and HIV load 12,000 copies/mL. The nurse has no significant medical history and, for baseline purposes, is found to be HIV-negative. She takes no medications and is not pregnant or breastfeeding. Her immunizations are current. What is the best approach to handling this situation? A. Therapy with one nucleoside reverse transcriptase inhibitor and one protease inhibitor to be initiated immediately and continue for four weeks B. Therapy with one nucleoside reverse transcriptase inhibitor and one protease inhibitor to be initiated immediately and continue for six months C. Therapy with two nucleoside reverse transcriptase inhibitors to be initiated immediately and continue for four weeks D. Therapy with two nucleoside reverse transcriptase inhibitors to be initiated immediately and continue for six months E. Unless seroconversion is documented, no antiviral therapy is necessary Explanation: A prophylactic antiretroviral regimen is always indicated when a healthcare worker is exposed to the bodily fluids of an HIV-positive patient. Immediate consultation with the hospital?s infectious disease expert is recommended. The CDC recommends that two nucleoside reverse transcriptase inhibitors (e.g., zidovudine and lamivudine) be started immediately after exposure and continued for the next

four weeks (Choice C). Some experts have suggested modifying the regimen to also include a protease inhibitor (e.g., indinavir) if viral resistance to the postexposure prophylaxis is known or suspected. Therapy with one nucleoside reverse transcriptase inhibitor and one protease inhibitor (Choices A and B) is not recommended. Therapy with two nucleoside reverse transcriptase inhibitors (Choice D) is indeed the recommended regimen, but the therapy should be continued for four weeks, not six months. Foregoing antiviral therapy (Choice E) is not advisable. Systemic infection does not occur immediately after inoculation with HIV; early on, there is a brief period in which antiretroviral therapy is thought to limit viral replication. One study showed that healthcare workers using zidovudine alone as postexposure prophylaxis were 81% less likely to seroconvert as compared to those who used no treatment regimen. Therefore, any healthcare worker exposed to the bodily fluids of an HIV-positive individual should be strongly urged to use a prophylactic regimen to minimize the likelihood of becoming HIV-positive. Educational Objective: Exposure to the body fluid of an HIV-positive individual requires antiretroviral prophylaxis. Accepted regimens include zidovudine and lamivudine or zidovudine, lamivudine, and indinavir. Treatment should begin within hours of the exposure and be continued for four weeks. 18% of people answered this question correctly. A 44-year-old Caucasian male presents to the emergency department with a 12 hour history of severe epigastric pain and vomiting. He had a similar episode one year ago that required hospitalization. He admits to excessive drinking for the past several weeks. He is not taking any medications, and denies any drug abuse. His blood pressure is 100/70 mmHg, and heart rate is 90/min. His serum lipase level is elevated. Which of the following admission criteria is indicative of a severe attack in this patient? A. Age B. Hypercalcemia C. High blood glucose level D. Hypokalemia E. Elevated ESR Explanation: Ranson?s prognostic criteria gained widespread use in predicting the severity of acute pancreatitis. Since the criteria were initially designed for pancreatitis not induced by gallstones, the scoring system was modified so it could also be used in the evaluation of patients with gallstone-induced pancreatitis. The prognostic factors include admission criteria, and criteria observed during the initial 48 hours. Admission criteria are age > 55 years, high WBC count, high serum AST, high serum LDH, and high blood glucose level. Criteria observed during the initial 48 hours include a hematocrit fall (>10%), BUN elevation, hypocalcemia, hypoxemia (PO2 < 60 mmHg), base deficit > 4 mEq/L, and an estimated fluid sequestration > 6L. (Choice A) The admission criteria includes an age > 55 years; however, this patient is only 44 years old. (Choice B) Hypocalcemia, not hypercalcemia, is a part of the Ranson's criteria during the initial 48 hours. (Choices D and E) Hypokalemia and elevated ESR are not considered significant prognostic factors. Educational Objective:

Ranson?s criteria gained widespread use in predicting the severity of acute pancreatitis. 59% of people answered this question correctly.

A 44-year-old Caucasian male presents to his primary care physician for an annual physical exam. He has no significant past medical history and takes no medications. He admits to drinking socially. He has a 25 pack-year history of tobacco use, and denies recreational drug use. He states that his mother and brother are both recovering alcoholics. What is the most effective method of detecting alcohol abuse or dependence in this patient? A. Inquire about psychiatric history B. Inquire about the type, frequency, and quantity of alcohol use C. Administer a screening questionnaire such as CAGE D. Administer a breath test to rule out alcohol intoxication E. Order laboratory testing to detect hepatic dysfunction Explanation: The approach to a patient with alcoholism is a stepwise process. Step 1: Inquire about current and past alcohol use, and about a family history of alcohol problems. Step 2: Obtain information regarding the type, quantity and frequency of alcohol use. This is useful to distinguish between moderate and heavy drinkers. Step 3: Use a standard screening questionnaire such as CAGE. Step 4: Ask more specific questions based upon steps 1 to 3 in patients with suspected alcohol problems. Studies have shown screening questionnaires (Choice C) to be superior to both laboratory testing (Choice E) and inquiries about quantity or frequency of drinking (Choice B) in evaluating individuals for alcohol abuse or dependence. Appropriate screening questionnaires include CAGE, AUDIT, and TWEAK. (Choice A) Although there is a high rate of comorbid psychiatric disorders in individuals with alcohol abuse and dependence, the likelihood of alcohol abuse should be more fully explored before discussing the psychiatric history. (Choice D) Breath tests are administered by law enforcement personnel when evaluating an individual for acute alcohol intoxication. They are not typically used in the setting of a physician?s office, and are more likely to inspire annoyance than compliance. Educational Objective: Studies have shown screening questionnaires to be superior to both laboratory testing and inquiries about quantity or frequency of drinking in evaluating individuals for alcohol abuse or dependence. * If this question had asked about ?next step? the answer would be choice B. 84% of people answered this question correctly. A 6-year-old Caucasian girl is brought to the office by her mother because of loss of appetite, malaise, and pain in the right arm. She has had these symptoms for the past five days. She has been playing with a small cat for the past month. She also has a small pet dog, which she feeds personally everyday. Examination shows tender cervical and right axillary lymphadenopathy. Over the next week, which of the following complications is most likely? A. Myocarditis B. Seizures C. Renal failure D. Hepatosplenomegaly E. Suppuration of the lymph nodes Explanation:

Cat-scratch disease is an infection that usually affects the young immunocompetent population. It is produced by Bartonella henselae. The most common complication is suppuration of the lymph nodes. (Choice D) Hepatomegaly and splenomegaly can sometimes be seen. (Choice C) Renal compromise has not been reported. (Choices A and B) Myocarditis and seizures are not part of the expected complications. Educational Objective: Approximately 10% of patients with cat-scratch disease can develop suppuration of the lymph nodes. Other complications are: visual loss due to neuroretinitis, encephalopathy, fever of unknown origin, and hepatosplenomegaly. 76% of people answered this question correctly. The following vignette applies to the next 2 items A previously healthy, 30-year-old Caucasian female is admitted to the hospital. She has a rash over her lower legs and ankles. She first noticed the rash over her ankles 2 weeks ago, and it slowly spread up to her knees. She has no other past medical history. She denies any regular medication intake. Her vital signs are within normal limits. Physical examination reveals the presence of fine petechiae and purpura around her ankles and lower legs. The rest of her physical examination is normal. The labs reveal the following: Hb: 14.4 g/dL MCV: 90 fl Platelet count: 16,000/cmm Leukocyte count: 8,000/cmm Segmented neutrophils: 60% Bands 3% Eosinophils 6% Lymphocytes 24% Monocytes 6% Prothrombin time: 14 sec Partial thromboplastin time: 30 sec Plasma fibrinogen: 300 mg/dL D-dimer: 0.22 ?g/mL Peripheral blood smear is normal. The admitting resident gets concerned with the lab results and orders 6 units of random donor platelet transfusion. The following day, the platelet count drops even further to 10,000/microliter. Item 1 of 2 Which of the following is the most likely cause of the drop in the platelet count? A. Disseminated intravascular coagulation B. Thrombotic thrombocytopenic purpura C. Antiplatelet antibodies D. Septicemia E. Drug induced thrombocytopenia Explanation: A presumptive diagnosis of idiopathic thrombocytopenic purpura (ITP) can be made when the history, physical examination, complete blood count, differential count, and peripheral blood smear do not suggest any causes for isolated thrombocytopenia. ITP occurs as a result of platelet destruction by specific autoantibodies. Platelet transfusions are rarely necessary to maintain the platelet count. The circulating platelets are rapidly removed by the autoantibodies anyway, making

platelet transfusions futile. Patients with mild and asymptomatic thrombocytopenia (platelet count between 30,000 to 50,000/microliter) should not be treated. Transfusions are only necessary in severe or symptomatic thrombocytopenia (i.e. life threatening emergencies such as intracerebral or massive gastrointestinal hemorrhage). (Choice A, B, and C) There is no clinical or laboratory evidence of DIC, TTP or septicemia in this patient. (Choice E) The patient has not been exposed to any drugs which could cause or precipitate thrombocytopenia. Educational Objective: ITP is due to autoimmune phenomena involving the formation of antiplatelet antibodies. Treatment is only necessary in severe or symptomatic thrombocytopenia. Item 2 of 2 Which of the following is the most appropriate next step in the management of this patient? A. Corticosteroids B. Plasmapheresis C. Low-dose heparin D. Intravenous immunoglobulins E. Give more platelets Explanation: ITP is acute and self-limiting in children, but it usually becomes a chronic disorder in adults. In such cases, corticosteroid therapy is the treatment of choice. Most adults respond to steroids within two weeks, with most patients responding within the first week. IV IG can be used if steroids fail. A splenectomy is rarely performed in refractory cases. (Choice B) Plasmapheresis is the treatment of choice for hemolytic uremic syndrome (HUS) and thrombotic thrombocytopenic purpura (TTP). (Choice C) Heparin is used very cautiously in patients with early disseminated intravascular coagulation (DIC). Once the patient starts to bleed, heparin should not be used. Educational Objective: Corticosteroid therapy is the treatment of choice for ITP in adult patients. 43% of people answered this question correctly. A 56-year-old Caucasian man comes to the emergency department with complaints of fever, pain, and swelling around the right lower leg for the last five days. His symptoms are getting worse, and he has been having difficulty weight bearing and ambulating for the last two days. He has a past medical history of coronary artery disease, ischemic cardiomyopathy with an ejection fraction of 25%, pacemaker implantation for sick sinus syndrome, and insulin-dependent diabetes mellitus with diabetic neuropathy. He quit smoking 12 years ago. He is a manager at a local restaurant. His temperature is 38.9 C (101 F), heart rate is 86/minute, blood pressure is 132/76 mmHg, and respiratory rate is 16/minute. Physical examination reveals significant swelling and erythema around the right lower leg. The lower extremity ultrasound is negative for deep vein thrombosis. A plain radiograph of the lower extremity is also negative for any acute pathology. The patient is started on the appropriate antibiotics; the blood culture results are still pending. What is the most appropriate next step in the management of this patient? A. Obtain a MRI scan of the lower extremity. B. Order a three phase technetium 99m bone scan. C. Repeat the x-ray in 2 weeks to confirm the diagnosis. D. Treat the patient for 4-6 weeks for tibial osteomyelitis. E. Obtain open bone biopsy to confirm the diagnosis.

Explanation: Osteomyelitis is an infection of the bone which causes bone destruction and necrosis. An acute infection usually presents with fever, chills, erythema, and swelling over the involved bones, although it is difficult to diagnose early in its course because the presenting symptoms and signs can be nonspecific. Radiologic imaging plays an important role in the evaluation of patients with suspected osteomyelitis. A three-phase technetium bone scan is the diagnostic test of choice for suspected uncomplicated osteomyelitis if the plain films are negative. It uses technetium 99-m bound to phosphorus as a tracer, which accumulates in the area of increased osteoblast activity. It is usually positive 2-3 days after the onset of infection. Some other nuclear scans that can be used include indium-labeled white cell scan and gallium citrate scans. (Choice A) CT scan and MRI are very accurate in detecting changes of osteomyelitis in the early stages. MRI is especially useful in patients with suspected vertebral osteomyelitis and in infected diabetic foot lesions. It is also useful in detecting abscesses around the site of infection. However, MRI cannot be used in patients with certain metal hardware and implants. This patient has a pacemaker implant, which precludes the use of MRI to evaluate suspected osteomyelitis. Furthermore, a technetium bone scan can easily make the diagnosis. For all these reasons, MRI is not needed in this patient. (Choice C) Plain films are insensitive for the diagnosis of acute osteomyelitis. This is due to the 2-3 week lag time for the changes of acute osteomyelitis to appear on the plain films. Repeating the films is also an insensitive procedure, and will only serve to delay the diagnosis. (Choice D) Empiric treatment for 4?6 weeks is inappropriate in the absence of a specific diagnosis. Furthermore, imaging is necessary to rule out the presence of potential complications (e.g., abscess or gas formation in the soft tissue). (Choice E) Although a bone biopsy is the gold standard for the diagnosis of osteomyelitis, it should only be performed if the non-invasive tests are inconclusive and the index of suspicion for osteomyelitis is high. Educational Objective: The three-phase technetium 99-m bone scan is the diagnostic test of choice for evaluation of acute uncomplicated osteomyelitis. MRI is better for evaluating spine and complicated foot osteomyelitis, including diabetic foot ulcers. A 33-year-old African-American female comes to your clinic. She has no significant past medical history. She works as an executive secretary. She drinks alcohol occasionally (twice a month), and denies any smoking or use of illegal drugs. She is married, and has a 5-year-old daughter. She was pregnant only once, and denies a prior history of miscarriages. She had preeclampsia during her first pregnancy at the 34th week of gestation. She is using oral contraception. She wants to get pregnant again, but she is afraid of having preeclampsia again. What is the most proper statement to tell her? A. If you had preeclampsia once, it does not mean that it will happen again. B. You will have preeclampsia for sure again, so you have to be very careful. C. Nobody can predict if it will happen again, so you must be careful. D. You don?t have to worry about that, preeclampsia usually occurs during the first pregnancy, it is rare during the second. E. You have a higher risk of developing preeclampsia than other women, but that does not mean it will happen again. Explanation:

A history of preeclampsia in the first pregnancy increases the possibility of a second episode in the following pregnancy. The risk is at least seven times higher, but it can be as high as 15 if the previous preeclampsia presented before 33 weeks of pregnancy. The patient had preeclampsia between 33 and 36 weeks of pregnancy, and is therefore ten times more likely to develop preeclampsia than a woman without that history. (Choices A and D) It is inappropriate to give the patient false reassurance. She has to be informed that the risk for preeclampsia exists. (Choices B and C) Although the patient is at risk, her chances for getting preeclampsia again are uncertain; therefore, one cannot make false assumptions, nor ignore that there are studies which address the topic and supply the patient with an ambiguous answer. Educational Objective: A history of preeclampsia in the past increases the risk of developing this complication during a subsequent pregnancy. The risk is higher if the preeclampsia presented earlier (age of delivery was less than 32-33 weeks), or if the patient has renal disease or chronic hypertension.

The following Vignette applies to the next 3 items A 34-year-old Caucasian male presents to your office with abdominal cramps, watery stools and mild fever. His symptoms started two days ago, and he has tried several over-the-counter anti-diarrheal agents without any success. He denies any gross blood or black discoloration of the stool, nausea and vomiting, although he admits that his appetite has decreased a little. His past medical history is significant for a recent episode of acute sinusitis treated with amoxicillin. He does not smoke or consume alcohol. He denies any recent travel or contact with a patient with similar symptoms. He is sexually active with his wife and uses condoms for contraception. He works as a programmer at a private firm and does not consider his job stressful. Physical examination reveals mild tenderness in the left lower quadrant of his abdomen. His stool is positive for occult blood, but is negative for C. difficile toxin by rapid immunoassay. Item 1 of 3 Which of the following is the best next step in the management of this patient? A. Repeat immunoassay for C. difficile toxin B. Do colonoscopy C. Order abdominal CT D. Order stool culture for C. difficile E. Obtain blood cultures and liver function tests Explanation: The given symptoms and findings in this patient, as well as the history of recent antibiotic treatment, are highly suggestive of C. difficile colitis. Rapid immunoassays to detect C. difficile toxins are gaining popularity among clinicians because these are less time-consuming and less expensive than the older stool cytotoxin test. These rapid tests have very high specificity (close to 100%); however, their sensitivity is about 70-87%, which is lower than that of the stool cytotoxin test (94-100%). Because of this lower sensitivity, repeating the test may be necessary in patients in whom the pretest probability of C. difficile infection is high and an initial test result is negative. (Choices B and C) Repeating the rapid immunoassay test is more reasonable and cost-effective in this case than proceeding with colonoscopy or abdominal CT. Furthermore, endoscopy is not generally recommended in patients with a classical scenario of C. difficile colitis, although this may provide valuable information in specific situations when a quick diagnosis is imperative, or when the diagnosis is in doubt.

(Choice D) Stool cultures of C. difficile are very labor-intensive and are not helpful because nontoxigenic strains of C. difficile exist. Educational Objective: Rapid stool tests to detect C. difficile toxins have very high specificity (close to 100%); however, their sensitivity is about 70-87%. Due to this lower sensitivity, repeating the test may be necessary in patients in whom the pretest probability of C difficile infection is high and an initial test result is negative. Item 2 of 3 After running the appropriate tests, you proceed with treatment. Which of the following agents is the best initial choice for this patient? A. Vancomycin B. Cholestyramine C. Bacitracin D. Metronidazole E. Clindamycin Explanation: There are several agents available for the treatment of C. difficile colitis. Vancomycin and metronidazole are the most popular agents because their efficacy and safety have been tested in many clinical trials. Metronidazole can be given in either oral or intravenous form because of its biliary secretion, whereas vancomycin should be given only in oral form as it will not be absorbed from the intestine. Most authors currently recommend metronidazole as the first-line agent over vancomycin because of the following reasons: 1) both agents have been shown to be equally effective; 2) metronidazole treatment is much more cheaper than vancomycin treatment; 3) vancomycin may lead to the selection of vancomycin-resistant enterococci, and this may create a future problem in public health. (Choice C) Oral bacitracin has been tried with success for C. difficile-induced diarrhea, but this is not commonly used, mainly due to its limited availability. (Choice D) Cholestyramine has also been proven to be effective for this condition, but its efficacy is much lower than that of vancomycin and metronidazole. (Choice E) Clindamycin is an agent that is often implicated as a cause of C. difficile-induced diarrhea. It is not the agent used for its treatment. Educational Objective: Most authors currently recommend oral or intravenous metronidazole as the first-line agent over oral vancomycin for the treatment of C. difficile-induced colitis. Item 3 of 3 Soon after the initiation of therapy, the patient seems to recover completely. One week after the completion of therapy, he returns to your office and complains of abdominal cramps and watery diarrhea again. His stool is positive for occult blood and the rapid immunoassay test is positive for C. difficile toxin in the stool. Which of the following is the best next step in the treatment of this patient? A. Vancomycin B. Cholestyramine C. Bacitracin D. Metronidazole E. Clindamycin

Explanation: This patient seems to be suffering from a relapse of C. difficile colitis. The most likely reason for the recurrence is the regeneration of the replicating form, with toxin production from the spore form. In patients with recurring symptoms after successful treatment, the cause of diarrhea should be ascertained first (the scenario says that rapid immunoassay is positive for C. difficile toxin in the stool). The management of a first relapse following therapy for C. difficile diarrhea and colitis does not differ substantially from treatment of the initial episode: metronidazole is preferred to vancomycin due to cost and bacterial resistance considerations. (Choice A) There is no reason to suspect C. difficile resistance to metronidazole because it is believed to be very rare, if it even exists; however, most authors recommend switching to vancomycin after more than one relapse because it seems to be more effective in such cases than metronidazole treatment. (Choices C and B) Bacitracin and cholestyramine have very limited roles in the treatment of patients with relapsing C. difficile diarrhea. (Choice E) Clindamycin is an agent that is often implicated as a cause of C. difficile-induced diarrhea; it is not the agent for its treatment. Educational Objective: The most common cause of recurrence of C. difficile is not due to resistance to the treatment. Rather, it is due to the development of the replicating form with toxin production from the spore form. The management of a first relapse following therapy for C. difficile diarrhea and colitis does not differ substantially from treatment of the initial episode.

A male infant is born at term to a 32-year-old African-American multigravida. The mother was diagnosed with gestational diabetes mellitus. The newborn?s length is 55 cm and weight is 4,500 kg. No respiratory symptoms or breathing difficulty is present. Physical examination reveals an adducted and internally rotated right arm with forearm pronation and flexed wrist. Moro reflex is asymmetric. Palmar grasp reflex is present on both sides. Which of the following is the best statement concerning the prognosis in this patient? A. By the age of one year the infant would most probably have no or mild paresis B. The condition most probably would not improve or may improve mildly C. The condition is progressive and would eventually involve the hand muscles D. Symmetric palmar grasp reflex has no prognostic significance E. Horner?s syndrome is typically associated with this type of injury Explanation: The classical scenario of Erb?s palsy is described. It is the most common form of obstetrical brachial plexus injury and involves the upper roots (C5, C6, and sometimes C7) of the plexus, resulting in an adducted and internally rotated right arm with forearm pronation and flexed wrist. A serious complication is diaphragmatic paralysis due to phrenic nerve involvement. The prognosis of Erb?s palsy is typically good, with an 80% chance of full or near-full recovery. (Choice D) Symmetric palmar grasp reflex is a good prognostic sign because it indicates that the lower roots of the brachial plexus are intact. (Choice E) Horner?s syndrome may be associated with injury of the lower roots of the brachial plexus.

Educational Objective: The prognosis of obstetrical Erb?s palsy is typically good, with an 80% chance of full or near-full recovery. A 22-year-old Asian-American woman returns to your clinic for a follow-up visit. You recently evaluated her for weight loss, amenorrhea, chronic diarrhea, and anxiety. Her laboratory tests show a suppressed TSH and elevated levels of FT3 and FT4. You then make a diagnosis of hyperthyroidism. The patient is currently complaining of palpitations. The physical examination shows that her heart rate is 118/min, with a blood pressure of 100/60 mmHg, and a respiratory rate of 18/min. Her heartbeat and pulse are rhythmic, but fast. No murmurs are detected. What is the most immediate step in the management of this patient?s condition? A. Propylthiouracil (PTU) B. Propranolol C. Methimazole D. Thyroidectomy E. Radioactive Iodine ablation therapy (RAI) Explanation: Symptomatic tachycardia in a hyperthyroid patient can be rapidly managed with beta-blockers. (Choices A and C) Methimazole or PTU is needed to control the thyrotoxic state, but is not an urgent indication, unless the patient is going through a thyroid storm (fever, altered mental status, CHF, liver disease). These drugs improve the cardiovascular complications of hyperthyroidism, but will not relieve palpitations or control tachycardia. (Choices D and E) Surgery and RAI are definitive therapies to be attempted after oral medication has decreased or controlled the thyrotoxic state, or if drug therapy fails. Educational Objective: Symptomatic tachycardia in patients with hyperthyroidism can be quickly controlled with betablockers (usually propranolol or atenolol). Treatment with PTU or methimazole will certainly improve the cardiac condition of the thyrotoxic patient, but only after several weeks. A 57-year-old female is started on subcutaneous erythropoietin injections for the treatment of anemia associated with endstage renal disease. Her hemoglobin and hematocrit levels at the start of erythropoietin therapy are 7 g/dL and 20%, respectively. She returns for a follow-up visit after six weeks of erythropoietin treatment, and her current hemoglobin and hematocrit levels are 7.1 g/dL and 22%, respectively. Her MCV is 82 cubic microns. She has been taking erythropoietin regularly. What is the next best step in the management of this patient? A. Increase erythropoietin dose. B. Start to administer erythropoietin intravenously in a higher dose. C. No change in treatment and follow up in 4 to 6 weeks. D. Measure percent transferrin saturation and ferritin levels. E. Start weekly blood transfusions. Explanation: Anemia of endstage renal disease is generally due to the decreased production of erythropoietin by the kidneys; however, other causes may contribute to the patient's anemia. These causes include iron deficiency, severe hyperparathyroidism (causes erythropoietin resistance), folate deficiency, systemic inflammation, and aluminum toxicity.

The goal of erythropoietin therapy is to increase the hematocrit level by 4 to 6% in 4-6 weeks time, and the final hematocrit level should lie between 33% and 36%. If the hematocrit level fails to increase adequately within 4 to 6 weeks following erythropoietin treatment, iron saturation and ferritin levels should be measured. Iron deficiency is treated by weekly intravenous administration of iron. (Choice A) There is no indication to increase the erythropoietin dose at this point. (Choice B) Intravenous administration is generally less successful than subcutaneous administration; the target hemoglobin level can be achieved at a lower dose with subcutaneous administration. (Choice C) The patient needs investigations for the cause of her suboptimal increase in hematocrit and hemoglobin levels. A follow-up visit in 4 to 6 weeks is not justified. (Choice E) Blood transfusion is usually avoided in patients with ESRD because they can sensitize a patient for allograft rejection following kidney transplantation. Educational Objectives: The anemia of patients with ESRD is generally due to erythropoietin deficiency. Other causes of anemia should be ruled out before starting erythropoietin therapy. Iron-deficiency anemia is a common cause of suboptimal increase in hematocrit and hemoglobin levels following erythropoietin therapy. A 55-year-old Caucasian male presents to the ER with fever, cough and chest pain. He also complains of right-sided headaches and recurrent nasal bleeding over the last two days. He underwent allogeneic bone marrow transplantation because of aplastic anemia. He had a mild episode of acute graft-versus-host disease soon after the engraftment, and this was successfully treated by high-dose corticosteroids. He received standard antimicrobial prophylaxis with ganciclovir, fluconazole and ciprofloxacin. Chest x-ray demonstrates bilateral patchy infiltrates. Which of the following is the most likely cause of this patient?s condition? A. Pneumocystis carinii B. Cytomegalovirus (CMV) C. Aspergillus D. Pseudomonas aeruginosa E. Legionella Explanation: Bone marrow transplant recipients are susceptible to a variety of infections, including bacterial, fungal, viral and protozoal infections. Carefully planned antimicrobial prophylaxis, as well as early recognition and treatment of established infections are important in decreasing morbidity and mortality in these patients. The clinical scenario describes three main categories of complaints: systemic infection (fever), pulmonary symptoms (cough and chest pain), and probable sinus symptoms (localized headache, nasal bleedings). Such constellation of symptoms is suggestive of an invasive fungal infection, which is most probably invasive aspergillosis. Risk factors for invasive aspergillosis include allogeneic transplantation (vs autologous transplantation), older age, acute graft-versus-host disease and corticosteroid therapy. Disseminated Candida infection is less likely because fluconazole prophylaxis was used. (Choices D, E and A) Gram-negative bacterial infection (P. aeruginosa and Legionella) and Pneumocystis carinii are important pathogens in patients who received bone marrow transplantation. These can cause pulmonary or disseminated infections; however, the combination of lung involvement and nasal/sinus infection is less typical in these infections.

(Choice B) CMV most commonly presents as fever of unknown origin, interstitial pneumonitis, or enteritis. Less often, these patients can develop retinitis, encephalitis, hepatitis, or bone marrow suppression. Ganciclovir prophylaxis significantly reduces the risk of CMV infection. Educational Objective: Invasive aspergillosis is common in bone marrow transplant recipients. respiratory tract, including the lungs and the sinuses.

It typically involves the

The following vignette applies to the next 3 items A 32-year-old woman comes to the emergency department (ED) for the evaluation of a low-grade fever and cough for the past five days. The cough is nonproductive and is associated with progressive difficulty in breathing. The patient easily gets short of breath with minimal walking and going upstairs. She was diagnosed with HIV infection approximately one year ago. She was started on antiretroviral treatment at that time, but it was stopped after two months due to intolerable side effects. Her last CD4 count done two months ago was 215 cells/cmm. On physical examination in the ED, her vital signs reveal a temperature of 37.9?C (100F), blood pressure of 110/64 mmHg, heart rate of 106/min and respiratory rate of 26/min. Her oxygen saturation is 92% on room air, and decreases to 86% with minimal walking. Her lung exam reveals scattered bilateral crackles and rhonchi. You suspect that she is suffering from Pneumocystis carinii pneumonia. Item 1 of 3 What should be the most appropriate next step for establishing a definite diagnosis of Pneumocystis carinii pneumonia? A. Start her on anti-retroviral therapy if the CD4 count is less than 200 cells per cubic millimeter. B. Obtain a high-resolution CT scan for confirmation of the diagnosis. C. Measure the diffusion capacity for carbon monoxide (DLCO). D. Perform a fiberoptic bronchoscopy with bronchoalveolar lavage for diagnosis. E. Legionella Explanation: Pneumocystis carinii pneumonia is seen in patients with defects in cell-mediated immune response, especially in those with HIV infections. It is usually seen in HIV-positive patients when their CD4 cell count decreases below 200 cells/cubic mm. It has an insidious onset and presents with fever, nonproductive cough, and progressive dyspnea and tachypnea. Chest x-ray of patients with Pneumocystis carinii pneumonia reveals diffuse and bilateral ground glass or alveolar infiltrates. It is generally recommended to establish a definite diagnosis of Pneumocystis carinii pneumonia before instituting specific therapy (some advocate initiating an empiric therapy in HIV-positive patients presenting with typical signs and symptoms). Fiberoptic bronchoscopy with bronchoalveolar lavage is the recommended procedure for the direct identification of organisms in the bronchoalveolar tree or respiratory secretions. It is a highly sensitive procedure and can be combined with a transbronchial biopsy to further improve its diagnostic yield. (Choice A) It is recommended to obtain further diagnostic workup for a definite diagnosis of Pneumocystis carinii pneumonia. (Choice B) Although a high-resolution CT scan has a high sensitivity for the diagnosis of Pneumocystis carinii pneumonia in a HIV-positive patient, it does not provide a specific diagnosis or documentation of the presence of Pneumocystis carinii in the respiratory secretions. (Choice C) A normal diffusion capacity for carbon monoxide essentially rules out the presence of Pneumocystis pneumonia in HIV-positive patients; however, an abnormal DLC does not provide a specific diagnosis of Pneumocystis carinii pneumonia. Educational Objective:

Fiberoptic bronchoscopy with bronchoalveolar lavage should be performed for direct identification of organisms in HIV-positive patients suspected of having Pneumocystis carinii pneumonia. Item 2 of 3 Which of the following is the most appropriate next step in the management of this patient? A. Patient aged less than 60 years B. Obtain a chest x-ray and treat with oral trimethoprim-sulfamethoxazole. C. Supply her with a two-month supply of aerosolized pentamidine. D. Admit the patient to the hospital and treat her with intravenous trimethoprim-sulfamethoxazole. E. Legionella Explanation: The patient in the above vignette has typical signs and symptoms of bilateral pneumonia, most likely Pneumocystis carinii pneumonia (PCP), in the setting of an HIV infection. She also has significant respiratory distress and marked desaturation with minimum activity. She has a very high risk of developing hypoxic respiratory failure and should be admitted to the hospital for close observation, oxygen therapy, and intravenous antibiotics. Trimethoprim-sulfamethoxazole (TMP-SMX) is the initial treatment of choice for all patients with suspected or documented PCP. Intravenous therapy should be given in the early stages of hospitalization and should be converted to oral therapy when the patient has signs of clinical recovery. (Choice A) Anti-retroviral treatment is an important part of the overall management of this patient, regardless of the CD4 cell count; however, specific treatment of Pneumocystis carinii pneumonia using TMP-SMX should first be started at this point. Anti-retroviral treatment alone may cause worsening of her respiratory failure due to an immune reconstitution phenomenon. (Choice B) Oral trimethoprim-sulfamethoxazole should be used only in patients with a mild pneumonia without any signs of impending respiratory failure. (Choice C) Intravenous pentamidine is the drug of choice for moderate or severe Pneumocystis carinii pneumonia in patients who are intolerant of trimethoprim-sulfamethoxazole. Aerosolized pentamidine is associated with a higher relapse rate, and is not recommended for treatment. Educational Objective: Intravenous trimethoprim-sulfamethoxazole should be used initially for all patients with moderate to severe Pneumocystis carinii pneumonia. Item 3 of 3 Which of the following is the most appropriate indication for the use of corticosteroids in this patient? A. Patient aged less than 60 years B. CD4 cell count of less than 50 per cubic millimeter C. Alveolar-to-arterial oxygen gradient of more than 35 mmHg on room air D. Suspected co-infection with Mycobacterium avium complex E. An arterial oxygen tension (PaO2) of less than 85 mmHg on room air

Explanation: The early use of corticosteroids has been shown to decrease the mortality rate and the rate of respiratory failure in patients with moderate to severe Pneumocystis carinii pneumonia (PCP). These observations are thought to be secondary to the anti-inflammatory effects, thus preventing further alveolar damage. Corticosteroid use is recommended in moderate to severe PCP infections with an

alveolar-arterial oxygen gradient of 35 mmHg or more, and/or an arterial oxygen tension (PaO2) of 70 mmHg or less on room air. (Choices A and B) The patient?s age and CD4 cell count do not influence the decision to use corticosteroids in patients with moderate to severe cases of PCP. (Choice D) Patients with suspected co-infections with Mycobacterium tuberculosis or Mycobacterium avium complex should not be treated with adjunctive corticosteroids. (Choice E) An arterial oxygen tension of less than 70 mmHg or less (not 85 mmHg) on room air is an indication for treatment with corticosteroids. Educational Objective: Adjunctive treatment with corticosteroids is recommended in patients with an alveolar-to-arterial gradient of 35 mmHg or more and/or an arterial oxygen tension of 70 mm or less in patients on room air. A new screening test is being evaluated for the early detection of stomach cancer. It is based on the measurement of a new serologic marker of gastric adenocarcinoma. The test is compared to traditional strategy of endoscopic evaluation of high-risk patients; this showed an increase in the survival of patients by several weeks. This increase in survival is statistically significant, although no difference is detected in the rate of radical gastrectomy between the two groups. Which of the following is the best explanation for the study results presented above? A. Low sensitivity B. Selection bias C. Length-time bias D. Lead-time bias E. Recall bias Explanation: A useful screening test is one that can detect the disease early enough to allow for an intervention which will improve the patient?s prognosis. The two components of a screening test which should then be evaluated are (1) early detection, and (2) associated increased survival; however, it is also important to detect any errors/bias which can possibly lead to an incorrect conclusion about the test. For instance, the phenomenon involving the screening test in this case can be explained by lead-time bias. Lead-time bias occurs when a screening test detects the disease at an earlier point in time (making it look like the survival rate increased), but the associated prognosis of the disease does not actually change. Since this test only diagnoses patients earlier than the standard test (i.e., endoscopy) but its associated prognosis is the same as the standard, its use offers no advantage to the standard screening test. (If you use this new test, you?re only going to let the people know that they have the stomach cancer at an earlier point in time, but you won?t be able to change their prognosis). (Choices A and B) Selection bias and low sensitivity cannot adequately explain the results of this study. (Choice E) Recall bias can distort the results of a case-control study because the investigators have to rely on the accounts of exposure as recalled by the participants of the study. Educational Objective: Lead-time bias occurs when a screening test detects the disease at an earlier point in time (making it look like the survival rate increased), but the associated prognosis of the disease does not change. (If you use this new test, you?re only going to let the people know that they have the disease at an earlier point in time, but you won?t be able to change their prognosis).

A 66-year-old African-American man is admitted to the hospital with complaints of constant low back pain for the last six weeks. The pain is insidious in onset, and is progressively getting worse. It does not radiate down his legs, and is not associated with any neurological symptoms. He has a history of hypertension, diabetes mellitus, and prostrate cancer treated with radical prostatectomy six months ago. His medications include aspirin, amlodipine, and metformin. You suspect vertebral metastasis from the prostrate cancer, and a radionuclide scan confirms your suspicion of the osteoblastic bony metastatic disease. Which of the following is the most appropriate initial treatment for this patient? A. Leuprolide alone B. Flutamide alone C. Diethylstilbestrol alone D. Chemotherapy with mitoxantrone E. Leuprolide and flutamide Explanation: This patient has presented with a metastatic prostrate cancer with spread to the vertebral column. Metastatic prostrate cancer remains a common clinical problem, despite early detection and treatment of the cancer. The most common site of spread is the bones, especially the axial skeleton. Skeletal metastases are usually osteoblastic, and should be suspected in a patient with prostrate cancer presenting with a new onset or worsening of chronic back pain. A radionuclide bone scan is the single most useful imaging modality for assessing any new bony metastasis. Androgen depletion is the primary treatment for men with a metastatic prostrate cancer. It is usually palliative rather than curative, and most patients lose the responsiveness to hormonal manipulations within two years. In most cases, luteinizing hormone releasing hormone (LHRH) agonists (leuprolide) are used as the first line therapy for advanced metastatic prostrate cancer. These bind to the LHRH receptors in the anterior pituitary gland and cause an initial release of luteinizing hormone (LH) and follicle stimulating hormone (FSH), which causes a transient rise of testosterone levels. After approximately one week of continuous therapy, there is down regulation of LHRH receptors, causing a decrease in the levels of LH. This leads to a significant decrease in the levels of testosterone. In the first week of therapy, the patient's symptoms tend to get worse during the initial testosterone surge. LHRH agonist monotherapy is contraindicated in patients with painful vertebral metastases and severe ureteral obstruction. These symptoms get worse during the initial testosterone surge seen in the first week of therapy. In such cases, an antiandrogen (e.g., flutamide) is given for one week before starting LHRH agonist monotherapy to block the effects of the initial testosterone surge. (Choice A) As mentioned above, LHRH agonist monotherapy is contraindicated in this patient. (Choice B) Antiandrogens used in patients with metastatic prostate cancer are flutamide, bicalutamide, and nilutamide. These block androgen receptors and prevent the growth-promoting effects of endogenous testosterone and dihydrotestosterone. These are associated with lower survival rates when used as a single agent. (Choice C) Diethylstilbestrol is a nonsteroidal estrogen, which exerts its effect by negative feedback action on the hypothalamic pituitary axis. It reduces the release of LHRH from the hypothalamus, thus decreasing the release of LH from the anterior pituitary. It is not available in the United States for commercial use, and its use is discouraged, due to the frequent side effects (e.g., myocardial infarction, stroke, and pulmonary embolism) associated with estrogen use. (Choice D) Patients with advanced, symptomatic, hormone-resistant prostrate cancer can be treated palliatively with chemotherapy. The optimal regimen for the chemotherapy is uncertain. Educational Objective:

LHRH agonists are the initial treatment of choice in patients with metastatic prostrate cancer. The concurrent administration of an antiandrogen for a week prior to LHRH therapy reduces the initial symptom flare.

An asymptomatic 42-year-old Caucasian woman comes to the physician because of "hair loss in groups" for the past six weeks. Her other medical problems include diabetes mellitus-type 1, gastroesophageal reflux disease, and bipolar disorder. She has smoked one pack of cigarettes daily for 15 years. She drinks 1-2 ounces of alcohol daily, and is a vegetarian. Her mother has bipolar disorder and breast cancer. Her medications include lansoprazole, glyburide, and lithium. Her vital signs are within normal limits. On close examination, the hairs have split-ends, and oil is visible on the strands. Which of the following is the most likely cause of her condition? A. Lithium B. Chemical reaction C. Oil toxicity D. Diabetes mellitus E. Allergic reaction Explanation: Because the hairs have split-ends, there should be a toxic or chemical reaction causing the hair loss. This can be the application of certain substances over the scalp or hair. (Choice A) Lithium is a recognized cause of alopecia. It produces thinning of the hair and widespread hair loss without damage to the hair itself. (Choice E) Allergic reactions will affect the scalp, and not the hair shaft. (Choice C) Oil is commonly used by women for hair grooming. It is only harmful if applied through hot compresses, in which case, it will also damage the scalp. (Choice D) Diabetes Mellitus has not been linked to alopecia. Educational Objective: Numerous drugs can produce hair loss. Lithium, thallium, and chemotherapeutic agents are between the most common ones; however, they characteristically do not affect the hair shaft, produce splitends, or other lesions. Traumatic alopecia, caused by traction, hair eating (trichotillomania), or chemical reactions, usually produces lesions on the hair strand or shaft itself, and is manifested as split-ends, trichoclasis, or fractured hair. The following vignette applies to the next 3 items A 45-year-old Caucasian female presents to the office because of progressive swelling, pain, and stiffness of the joints of her hand and wrist for the past several months. She says it takes hours to get some relief because her hands are so stiff in the early morning. Her past medical history is not insignificant. Her mother has a history of systemic lupus erythematosus. Examination of her joints shows warmth, swelling, and tenderness of the proximal interphalangeal joints, metacarpophalangeal joints, and wrists. Item 1 of 3 Which of the following is the most appropriate next step in the management of this patient? A. Order rheumatoid factor and prescribe ibuprofen B. Order antinuclear antibodies and start her on prednisone C. Check parvovirus titers and prescribe ibuprofen

D. Order anti dsDNA level and start naproxen E. Check serum uric acid level and start on colchicine Explanation: The above patient is most likely suffering from rheumatoid arthritis (RA). His clinical features of symmetric polyarthritis (involving MCP and PIP joints) associated with morning stiffness lasting for more than 30 minutes for several (more than six) weeks are compatible with the diagnosis of RA. This diagnosis is made clinically. The best next step in this case is to order rheumatoid factor and antinuclear antibodies because SLE patients can have a similar presentation, even though this is not classic. Hand x-rays should be ordered, and steroids should be avoided, if possible. For symptomatic relief, the patient should be started on NSAIDs, which are the first line drugs in the management of RA. (Choice C) Joint involvement in parvovirus infections is also symmetrical. The hands, wrists, knees, and feet are the most frequently involved joints. The patient may have arthralgias or arthritis, and a rash may or may not be present. Joint involvement most frequently occurs in adult females; however, the parvovirus infection will not last that long. (Choice E) Although the patient could be having a very atypical presentation of gout, NSAIDs should still be given to relieve her pain. Serum uric acid has no value in diagnosing the gouty arthritis. Educational Objective: Clinical features of symmetric polyarthritis (involving MCP and PIP joints) associated with morning stiffness lasting for more than 30 minutes for several (more than six) weeks are compatible with the diagnosis of rheumatoid arthritis. Item 2 of 3 X-ray of the hands shows periarticular osteopenia and erosions of the proximal interphalangeal and metacarpophalangeal joints. Her lab results are as follows: ANA negative anti dsDNA negative rheumatoid factor negative Parvovirus titers positive IgG One week later, she comes back and says that her symptoms are relieved by her new medications. Which of the following is the most appropriate next step in the management of this patient? A. Start her on low dose maintenance glucocorticoids B. Start her on methotrexate C. Start her on allopurinol D. Obtain PPD and start her on infliximab E. Tell her that she does not need additional treatment Explanation: The above patient has erosive joint disease. She should be started on disease-modifying antirheumatic drugs (DMARDs), because these agents slow down the progression of bony erosions and cartilage loss. According to current recommendations, DMARDs should be be used earlier in the course of disease. Other indications for the use of these agents are: disease refractory to conservative treatment, and dependence on steroids. Some examples of DMARDs are methotrexate, hydroxychloroquine, sulfasalazine, leflunomide, etanercept, infliximab, and azathioprine. Methotrexate is the initial drug of choice, but if the response is not adequate, other DMARDs may be used.

(Choice E) Hydroxychloroquine is inferior to methotrexate, even though it is very safer than methotrexate. Leflunomide has a role in patients who have contraindications to methotrexate, such as significant pulmonary fibrosis. (Choice D) Etanercept and infliximab are the new generation TNF inhibitors, and are highly effective in patients who have refractory disease with methotrexate. They are very expensive and are not indicated as the first line drugs of treatment. PPD is indicated before the treatment with TNF inhibitors is started. (Choice A) Glucocorticoids do not alter the natural course of the disease, and are indicated when the disease is refractory to NSAIDs and DMARDs, or when there are severe constitutional symptoms or extra-articular manifestations. (Choice E) Positive IgM antibodies make the diagnosis of parvovirus infection. IgG can be present in the general population. Educational Objective: Erosive joint disease in rheumatoid arthritis is a clear-cut indication for the use of disease-modifying anti-rheumatic drugs (DMARDs), and methotrexate is the initial drug of choice for this purpose. Item 3 of 3 What is the most likely diagnosis of this patient? A. Rheumatoid arthritis B. Systemic lupus erythematous C. Polymyalgia rheumatica D. Parvovirus infection E. Polyarticular gout Explanation: Rheumatoid arthritis (RA) is a clinical diagnosis. In patients with negative rheumatoid factor (RF) levels but with the classic clinical and radiologic presentation, the most likely diagnosis is still RA. Elevated RF levels are detected in approximately 75 to 80% of patients at some time during the course of their disease. A new test, called anti-CCP antibodies (anti-citrulline containing peptide antibodies) has some value in suspected patients with negative RF levels. Educational Objective: RA is a clinical diagnosis. RF levels may be negative in approximately 20% of the patients. An 82-year-old Caucasian female is brought to the emergency department (ED) with high-grade fever. She resides at a local nursing home. The nursing home staff reports that she has been acting strangely for the past two days. She has a past medical history of COPD, diabetes mellitus, hypothyroidism, atrial fibrillation, and severe, deforming rheumatoid arthritis. Her medications include aspirin, insulin, albuterol inhaler, levothyroxine, warfarin, and low-dose prednisone. In the ED, her temperature is 38.9?C (102F), blood pressure is 126/84 mmHg, respiratory rate is 20/min, and heart rate is 92/min. Her oxygen saturation is 96% on 3 liters oxygen. She is awake and alert, but does not appear oriented to her surroundings. Her mucous membranes are extremely dry. The rest of her physical examination is unremarkable. Initial diagnostic work-up reveals the following: Serum chemistry Serum Na: 158 mEq/L Serum K: 4.2 mEq/L Chloride: 108 mEq/L Bicarbonates: 22 mEq/L BUN: 68 mg/dL Serum Creatinine: 2.4 mg/dL Calcium: 10.2 mg/dL Blood Glucose: 140 mg/dL

Urine Specific gravity: 1.020 Blood: trace Glucose: negative Ketones: negative Leukocyte esterase: positive Nitrites: positive WBC: 50+/hpf RBC: 10-20/hpf Casts: none The patient is admitted to the hospital and started on intravenous fluids and antibiotic therapy. On the first night of admission, she becomes very confused and agitated. She pulls her IV line out, and this causes significant bleeding. She then hits the head nurse on the face. Which of the following is the best initial treatment option for this patient? A. Lorazepam B. Haloperidol C. Venlafaxine D. Buspirone E. Zolpidem Explanation: The patient in the above vignette is suffering from an acute confusional state, also known as delirium. This is a common occurrence in elderly, hospitalized patients, and is manifested as a change in the level of awareness, cognitive and perceptual disturbances, disorientation and agitation. Delirium is a multifactorial disorder. Some common conditions that can precipitate acute delirium include the use of multiple medications (polypharmacy), fluid and electrolyte disturbances (dehydration, hypo or hypernatremia), infections (urinary tract infections and pneumonia), malnutrition, immobility (hospitalization), and the use of bladder catheters. Supportive measures such as reassurance, frequent orientation, and constant supervision should be used initially to manage confusion and disorientation in such patients. If the patient demonstrates combative behavior, low-dose antipsychotics (haloperidol or risperidone) should be used to acutely control the symptoms, in order to prevent any harm and to allow for the safe continuation of treatment. (Choice A) Benzodiazepines have a rapid onset of action and can rapidly control agitation and combative behavior; however, these drugs can cause disinhibition and potentially worsen the acute confusional state. These can also suppress the respiratory drive, and should therefore be used with caution in patients with chronic respiratory disorders. (Choices C and D) Venlafaxine is a serotonin/norepinephrine re-uptake inhibitor which is used in the treatment of depression and generalized anxiety disorder. Buspirone is an anti-anxiety drug used for the treatment of generalized anxiety disorder. Both drugs have a slow onset of action and should not be used for acute management of agitated or aggressive behavior. (Choice E) Zolpidem is used in the treatment of insomnia. Educational Objective: High potency neuroleptics (haloperidol) should be used for the acute management of agitated and combative hospitalized patients. The following vignette applies to the next 2 items

A 60-year-old woman is brought to the emergency department following a motor vehicle accident. She hit a car from her side and claims that she did not see another car coming towards her. This accident took place at a small intersection with little traffic. Her past medical history is significant for Cushing?s disease and an intra-abdominal operation 15 years ago for the disease. Physical examination showed a tanned female with normal vital signs. She has a few abrasions on her face and chest. Ophthalmologic examination reveals bitemporal hemianopsia. Her injuries are managed appropriately. Item 1 of 2 What is the next best step in the management of this patient? A. X-ray of the skull B. MRI of the brain C. CT angiogram D. Waters? view E. Slit lamp examination Explanation: The clinical features in this patient are suggestive of a mass lesion in the sellar and suprasellar region. Furthermore, the history of bitemporal hemianopsia and hyperpigmentation following an abdominal operation (bilateral adrenalectomy) is very suggestive of Nelson?s syndrome. The increased pigmentation is due to melanocytes, stimulating the activity of ACTH. MRI and plasma ACTH levels are required for making the diagnosis. A pituitary microadenoma with suprasellar extension on MRI and extremely high plasma ACTH levels are diagnostic. (Choice A) X-ray of the skull may show an enlarged cella, but a plain x-ray is much less sensitive than MRI in the diagnosis and evaluation of pituitary masses. (Choice C) Bitemporal hemianopsia cannot be localized to any specific cranial arterial territory; therefore, CT angiography is not warranted. (Choice D) Waters? view is done to evaluate sinusitis. It is not used in the evaluation of pituitary adenomas. (Choice E) The clinical features are not suggestive of intraocular defect; thus, a slit lamp exam is unlikely to be helpful. Educational Objective: Bitemporal hemianopsia and hyperpigmentation following bilateral adrenalectomy for Cushing?s disease is suggestive of Nelson?s syndrome. A pituitary microadenoma with suprasellar extension on MRI and extremely high plasma ACTH levels are diagnostic. Item 2 of 2 What is the most likely diagnosis? A. Prolactinoma B. Nelson?s syndrome C. Craniopharyngioma D. Brain tumor E. Empty cella syndrome Explanation: Pituitary enlargement and hyperpigmentation following bilateral adrenalectomy for Cushing?s disease is termed as Nelson?s syndrome. The cause of pituitary enlargement is the loss of feedback by the adrenal glucocorticoids following bilateral adrenalectomy. The tumor in Nelson?s syndrome is aggressive and is treated by surgery and/or pituitary radiation. Following bilateral adrenalectomy, prophylactic pituitary radiation sometimes prevents the development of Nelson?s syndrome; however,

this leads to an increased risk for hypopituitarism. Previously, bilateral adrenalectomy was the preferred treatment for Cushing?s disease; however, with the advent of better localization and improved techniques of transsphenoidal surgery, primary pituitary surgery is now the preferred treatment for Cushing?s disease. (Choice A) Prolactinoma can cause bitemporal hemianopsia but will not lead to increased pigmentation. Furthermore, the past history of bilateral adrenalectomy favors the diagnosis of Nelson?s syndrome. (Choice C) Although craniopharyngioma can occur in adults, it is uncommon at this age. The clinical features of craniopharyngioma are visual field defects and hypopituitarism. Diabetes insipidus is more common compared to pituitary macroadenoma. (Choice D) Except for parasellar or intrasellar meningiomas, brain tumors usually do not present with bitemporal hemianopsia. Hyperpigmentation does not occur in brain tumors. (Choice E) Visual field defects are uncommon in patients with empty sella syndrome. Educational Objective: Nelson?s syndrome is characterized by pituitary enlargement, hyperpigmentation, and visual field defect following bilateral adrenalectomy. Usually, these tumors are rapidly growing and can be treated with surgery and/or local radiation.

A group of researchers wants to investigate an outbreak of acute diarrhea that occurred in a small coastal town. Approxiately 50 people had a severe hemorrhagic diarrhea, and one fatal case was reported. The researchers believe that the outbreak is related to the seafood prepared at one of the coastal restaurants. Which of the following study designs is most appropriate to test the hypothesis? A. Cohort study B. Cross-sectional study C. Case-control study D. Correlational study E. Clinical trial Explanation: A case-control study is the most appropriate study design to investigate an outbreak of an acute infectious disease. The subjects who developed the disease should be questioned about their recent exposures (eating a particular dish at a particular restaurant in this case), and unaffected subjects should be chosen as controls to reflect the exposure experience of the general population. If cases are more likely to be exposed than the controls (i.e., the odds ratio is > 1 and statistically significant), then an association between the exposure and the disease can be established. (Choice A) A cohort study is not appropriate because the exposure status of already diseased subjects is of primary interest. (Choice E) A clinical trial is clearly not ethical. (Choices B and D) Cross-sectional and correlational studies are not informative in such cases.

Educational Objective: A case-control study is the most appropriate study design to investigate an outbreak of an acute infectious disease. It allows for the quick localization of the source of the outbreak. The following vignette applies to the next 2 items A 19-year-old female was found to have a 2.1 cm nodule in her right thyroid lobe. She is otherwise healthy and takes no medications. She has no family history of thyroid diseases or a history of exposure to radiation in the past. She denies any pain, difficulty in swallowing, or voice change. Physical examination is otherwise normal. Labs reveal normal T3, T4, and TSH levels. Ultrasound of the thyroid gland shows a solid solitary nodule measuring 2.6 x 1.7 x 1.3 cm in the right lobe. The left lobe appears to be normal in size and echotexture. Item 1 of 2 What is the next best step in the management of this patient? A. Perform a fine needle aspiration biopsy of the right lobe nodule B. Start the patient on suppressive doses of levothyroxine to decrease the size of the nodule C. Refer to a surgeon for thyroidectomy D. Repeat thyroid ultrasound next year to monitor the size of the nodule E. Perform a radioactive iodine uptake and scan Explanation: Nodules in the thyroid gland are very common, particularly in older subjects. Thyroid nodules measuring 1 cm or more should be subjected to a fine needle aspiration biopsy. Nodules measuring less than 1 cm in size are generally followed with thyroid ultrasound on a yearly basis. (Choice D) This nodule is more than 1 cm, therefore repeating the ultrasound after one year is incorrect . (Choice E) A radioactive scan is seldom useful in the management of thyroid nodules. Although the incidence of thyroid cancer is higher in cold nodules (a focal decrease in radioactive iodine uptake), the majority of cold nodules are benign. Radioiodine uptake and scan are useful for diagnosing toxic adenoma in thyrotoxic patients. (Choice C) Most of these nodules are benign. Surgery is not warranted in benign thyroid nodules unless the nodules are very large or the patients have compressive symptoms. (Choice B) Benign nodules grow very slowly over time. A suppressive dose of levothyroxine has not been found to be consistently useful in reducing their size, therefore its use is not recommended. Furthermore, suppressive doses of levothyroxine can cause bone loss and atrial fibrillation. Educational Objective: Thyroid nodules measuring 1 cm or more should be subjected to a fine needle aspiration biopsy. Nodules measuring less than 1 cm in size are generally followed with thyroid ultrasound on a yearly basis. A radioactive scan is seldom useful in the management of thyroid nodules. Radioiodine uptake and scan are useful for diagnosing toxic adenoma in thyrotoxic patients. Item 2 of 2 Further workup was very suspicious for medullary thyroid cancer. Serum calcitonin was checked and found to be elevated at 300 pg/mL (0-10 pg/mL is normal). Genetic studies reveal a mutation in the RET proto-oncogene. Which of the following is the next best step in the management of this patient? A. Refer for thyroidectomy B. Measure plasma free metanephrine C. Refer to an oncologist for cancer chemotherapy D. Treat with radioactive iodine

E. Treat with external beam radiation Explanation: This patient is likely to have MEN 2 syndrome caused by RET proto-oncogene mutation. Medullary cancer arises from parafollicular cells rather than thyroid follicular cells. Medullary cancer could be familial (familial medullary cancer) or associated with other endocrinopathies, like pheochromocytoma and hyperparathyroidism in MEN 2 syndromes. Pheochromocytoma may be present in 40% of MEN 2 patients, although some of them are not clinically apparent. (Choice A) Screening for pheochromocytoma is mandatory in these cases before subjecting them to thyroid surgery. Undiagnosed pheochromocytoma can cause life-threatening hemodynamic complications during and after surgery. (Choice D) Medullary thyroid cancer cells do not take up iodine; therefore, radioactive iodine is not used in these patients. (Choices C and E) Anti-cancer drugs and external beam radiation play no role in the initial management of medullary thyroid cancer . Educational Objective: Recognize the importance of screening for pheochromocytoma in patients with medullary thyroid cancer. A healthy 15-year-old girl comes to the clinic with her boyfriend because she has just discovered she is pregnant. Her last menstrual period was approximately seven weeks ago. She states that her parents would be quite upset if they discovered she was sexually active. She is certain that she wants to have an elective abortion and requests a referral to a local provider. Given the circumstances, who should provide legal consent for the abortion? A. Patient B. Parent C. Sibling D. Boyfriend E. Court Explanation: As a physician, it can be difficult to balance the adolescent?s right to confidentiality against the parental right to oversee the child?s health. To address this conflict, most states have implemented laws that allow for a physician to provide care to adolescents without parental consent. These laws are typically limited to the management of certain issues, including pregnancy, contraception, sexually transmitted diseases, substance use, and emotional illness. Therefore, it is the teenaged patient who will ultimately provide consent for the abortion. (Choices B, C, and D) Abortion providers do not normally consult a patient?s parents, siblings, or boyfriend for approval before carrying out the procedure. Some states, however, do have parental notification laws regarding abortion, which the patient should be instructed about. (Choice E) Typically, teenaged girls are able to obtain abortions without the approval of the court. However, the court is sometimes petitioned to allow for an exception to parental notification laws, especially in cases of abuse or incest. Educational Objective: Most states have implemented laws that allow for a physician to provide certain types of medical care to adolescents without parental consent. Elective abortions are typically protected under these laws. Therefore, a teenaged girl would have the authority to provide legal consent for an abortion.

A 17-year-old female comes to your office for an annual physical examination. Her best friend's mother recently died from metastatic breast cancer. She seems very concerned, and requests a clinical breast examination and a referral for a screening mammography for herself. Which of the following is the most appropriate response to her request? A. Clinical Breast Examination (CBE) alone has never been shown to reduce breast cancer mortality in the clinical studies B. Monthly Breast Self Examination (BSE) is the most effective screening tool to detect early breast cancer and reduce mortality C. She should start an annual screening mammography to detect early breast cancer from age 30 years D. Annual or biennial screening mammography has shown to reduce breast cancer mortality across all age groups E. We should start annual clinical breast examination and screening mammography at 20 years of age Explanation: Breast cancer is the second leading cause of cancer deaths in women after lung cancer. The concerns and apprehension in women associated with breast cancer are understandable, given the physical and emotional impact of the disease. According to the USPSTF guidelines, there is insufficient evidence to recommend for or against routine clinical breast examination (CBE) alone as a screening tool for breast cancer. This is based on the fact that there are no clinical studies that have examined the benefits of CBE alone. (Choice B) The USPSTF found poor evidence to support the hypothesis that breast self examination (BSE) reduces breast cancer mortality. On the other hand, it found fair evidence that BSE increases the chances of false positive results and biopsies. (Choices C, D, and E) The USPSTF recommends screening mammography, with or without clinical breast examination, every one to two years for women aged 40 or older. This is based on the evidence from clinical studies that screening mammography every 12 to 33 months reduces mortality from breast cancer. This evidence is strongest for women 50-69 years of age, and weaker for women 40-49 years of age. Older women (> 70 years) have a higher probability of developing breast cancer, but also have a higher chance of dying from an unrelated cause. Therefore, screening in patients who are 40-49 years or older than 70 years of age should be done after appropriate clinical considerations and patient counseling. Educational Objective: The current USPSTF guidelines recommend screening mammography, with or without clinical breast examination, every one to two years for women aged 40 or older.

A 54-year-old Caucasian woman comes to the office for abdominal discomfort. She has been feeling weak for the past seven days. She is not eating well because of nausea, and has noticed that her urine has become darker. She has hypertension, generalized anxiety disorder, and obesity. Her medications include lisinopril, hydrochlorothiazide and aspirin. She has been taking some herbal medications for the past four months. She was using gingko biloba as an energy booster and kava to control her anxiety. Two months ago, she started to use ginseng in an attempt to improve her memory problems. She states that she had been feeling better, less anxious, and more alert, until last week. She does not use tobacco, alcohol or drugs. Her family history is not significant. She has no known allergies. Examination shows mild jaundice. Heart sounds are normal and lung sounds are clear. The liver is palpable 2 cm below the costal margin and tender. Neurologic examination reveals no abnormalities. Her laboratory tests reveal the following: CBC

Ht: 44% Platelet count: 220,000/cmm Leukocyte count: 9,000/cmm Segmented neutrophils: 55% Lymphocytes: 42% Monocytes: 3% Serum Chemistry Serum Na: 143 mEq/L Serum K: 4.2 mEq/L Chloride: 106 mEq/L Bicarbonate: 26 mEq/L BUN: 21 mg/dL Serum Creatinine: 1.0 mg/dL Calcium: 9.7 mg/dL Blood Glucose: 98mg/dL LFT Total bilirubin: 3.2 mg/dL Direct bilirubin: 2.8 mg/dL Alkaline phosphatase: 200 U/L Aspartate aminotransferase: 350 U/L Alanine aminotransferase: 420 U/L Which of the following is the most probable cause of the patient?s disease? A. Viral hepatitis B. Enalapril C. Kava D. Gingko biloba E. Ginseng Explanation: The patient?s laboratory test results show that there is moderate hepatic injury. Kava (Piper methysticum) has been identified as a cause of hepatitis, cirrhosis and liver failure. Several patients have died in Europe, Canada and the United States after its ingestion. The mechanism is still unclear, but the United States Food and Drug Administration (FDA) has already issued a warning to the public concerning its use (year 2002). (Choice A) Viral hepatitis is usually associated with AST and ALT levels over 500 units. The classic presentation is a period of fever, malaise, asthenia, nausea and vomiting, followed by the development of jaundice. This patient?s clinical features are different. (Choice B) Enalapril is metabolized in the kidney. hepatotoxic drug.

There are no cases reporting its role as a

(Choice D) Gingko has been related to bleeding (due to its antiplatelet activity) and seizures. These adverse effects are generally reversible, temporary and mild. There are no reports of liver toxicity with this herb. (Choice E) Ginseng has been associated with some serious adverse effects (i.e., schizophrenia, severe headache, and Stevens-Johnson syndrome), as well as other less dangerous effects (i.e., vaginal bleeding, insomnia, diarrhea or mastalgia). Its use has not been related to liver damage. Educational Objective: Some popular herbal remedies have significant side effects and can be toxic to a specific organ. Aconite is cardiotoxic, and may cause arrhythmias and hypotension. Kava can cause liver injury.

Gingko biloba has been related to bleeding, while ginseng has been associated with Stevens-Johnson syndrome and psychosis. An 82-year-old Caucasian man presents to the emergency department for the evaluation of increased urinary frequency and burning, and mental confusion. He is admitted to the hospital, and is started on intravenous fluids and antibiotics. The next day, during the rounds, he is found to be in mild respiratory distress. His physical examination reveals the presence of jugular venous distention (9 cm) and bilateral basilar crackles. His chest x-ray is consistent with pulmonary edema. A 2D echocardiogram done two months ago revealed an ejection fraction of approximately 35%. You give the patient 40 mg of IV furosemide (Lasix), after which his respiratory status improves. His mental status remains unchanged, and he occasionally has episodes of agitation and visual hallucinations. The attending physician tells you to start him on medications for heart failure that would not affect his mental status. Which of the following is the most appropriate drug to start at this point? A. Perform lumbar puncture and start antibiotics B. Digoxin C. Spironolactone D. Verapamil E. Ginseng Explanation: The patient in the above vignette is admitted to the hospital with urinary tract infection and possible urosepsis. Acute infections are a common cause of delirium, especially in elderly patients being admitted to the hospital. An important goal of treatment in such patients is to avoid the use of medications that can potentially worsen the acute delirious state. Diuretics, ACE inhibitors, spironolactone, beta-blockers, and digoxin are all a part of standard therapy for patients with heart failure or cardiomyopathy due to any cause. ACE inhibitors (lisinopril) have consistently been shown in multiple, large, randomized trials to have a significant beneficial effect on patients with heart failure. They have a very low incidence of central nervous system (CNS) side effects, and do not cause worsening of confusional states or delirium. Lisinopril can be safely started in this patient without the risk of exacerbating his confusion and agitation. (Choice B) Digoxin is a cardiac glycoside frequently used in patients with congestive heart failure due to systolic dysfunction and atrial dysrhythmias; however, its use has been associated with significant CNS side effects. These include blurred vision, dizziness, confusion, mental disturbances, anxiety, delirium, and hallucinations. Its use should be avoided in patients with ongoing acute mental confusion or delirium. (Choice C) Spironolactone is a useful adjunct in the management of patients with congestive heart failure, especially in patients with a low ejection fraction; however, its use can also cause CNS disturbances such as drowsiness, lethargy, and mental confusion. (Choice D) Verapamil is a calcium channel blocker used in the treatment of hypertension, angina, and supraventricular tachyarrhythmias. It has a lower incidence of CNS side effects, but can precipitate or exacerbate heart failure symptoms. It should be avoided in patients with congestive heart failure. Educational Objective: Digoxin use can exacerbate confusion and mental disturbances and should be avoided in patients with delirium.

A 66-year-old Asian-American woman presents to the emergency department and complains of lower back pain, difficulty walking, urinary incontinence, fever of 39.3C (102.8F), and decreased appetite.

She was recently diagnosed with a herniation of disc L4-5 and was given an epidural injection for radicular pain two days ago. Her past medical history is significant for diabetes mellitus, chronic renal failure, and hypertension. She is alert and oriented. Her temperature is 38.3C (100.9F), blood pressure is 136/88 mm Hg, pulse is 86/min, and respirations are 14/min. Physical findings include poor dentition, diffusely tender abdomen, decreased rectal sphincter tone, mild tenderness upon palpation of the lumbar region, and absent deep tendon reflexes in both lower extremities. Initial laboratory examination includes a WBC of 22,000/mm3 with 69% neutrophils and 15% bands, and an erythrocyte sedimentation rate of 104 mm/hr. The result of her urinalysis is normal. What is the most appropriate course of action in this woman?s care? A. Perform lumbar puncture and start antibiotics B. Obtain MRI of spine C. Immediate neurosurgery consult D. Administer antibiotics E. Obtain bone scan Explanation: An epidural abscess should be suspected in a susceptible patient who presents with fever and back pain. Patients who are susceptible are the elderly, immunocompromised individuals (i.e., patients with AIDS, cancer, diabetes mellitus, chronic renal failure, or alcoholism), and patients who were recently involved in trauma, spinal surgery, or administration of epidural anesthesia. The diagnosis must be made immediately because delays in treatment increase the risk of permanent neurologic damage or death. A gadolinium-enhanced MRI is the preferred imaging modality to confirm the diagnosis because it adequately demonstrates inflammatory changes in the soft tissue. When MRI is not available, CT with myelography is an alternative. Once the diagnosis is made, antibiotic therapy should be guided by obtaining a culture specimen via CT-guided aspiration or open biopsy. The most common etiologic agent is Staphylococcus aureus. Spinal epidural abscesses is a surgical emergency; early surgical decompression and drainage, preferably within the first 24 hours, is the most important part of management to improve the ultimate prognosis. (Choice A) Lumbar puncture is not required to make the diagnosis and is not recommended. However, blood cultures should be drawn and antibiotics should be started. (Choice C) A neurosurgery consult should be obtained once more information is collected that suggests a diagnosis. (Choice D) Even though antibiotics are important, making the diagnosis and urgent surgical drainage is more important. Educational Objective: Epidural abscesses are best diagnosed with gadolinium-enhanced MRI of the spine. Spinal epidural abscess is a surgical emergency. Early surgical decompression and drainage, preferably within the first 24 hours is the most important part of management to improve the ultimate prognosis. A healthy 48-year-old African-American man comes to the physician for a routine health maintenance examination. He has no complaints. He has no medical problems. He does not drink alcohol. He has smoked one pack of cigarettes daily for 22 years. He takes no medication. His father was diagnosed with type-2 diabetes at the age of 48. His vital signs are within normal limits. Examination shows no abnormalities. Which of the following is the most appropriate next step in the management of this patient? A. Check random blood sugar B. Obtain fasting blood sugar C. Obtain glycosylated hemoglobin level D. No screening required at this time E. Perform glucose tolerance test

Explanation: Screening for diabetes mellitus (DM) is indicated in everyone aged 45 years or older, and should be repeated every three years in the absence of other risk factors. Screening should be performed earlier and more frequently if risk factors are present. The recommended screening test is measurement of the fasting blood glucose level. Risk factors for DM include: 1. age 45 years or greater 2. family history of diabetes in a first-degree relative 3. overweight 4. dyslipidemia 5. hypertension 6. impaired glucose tolerance or impaired fasting glucose 7. history of vascular disease 8. habitual physical inactivity 9. history of gestational DM or history of delivering a baby weighing greater than 9 lbs 10. polycystic ovarian disease 11. high-risk race (like African-American or Hispanic) The above patient has two risk factors for DM (e.g., history of diabetes mellitus in his father, and above 45 years of age); therefore, he needs to be screened by obtaining his fasting blood glucose levels. (Choice C) Glycosylated hemoglobin levels are not currently recommended for diabetes mellitus screening. It is used for monitoring the disease activity. (Choice A) Measurement of random blood sugar levels may be used for DM screening when a person comes without overnight fasting; however, obtaining fasting blood sugar levels is the more preferred screening tool. (Choice E) A glucose tolerance test is usually not used for DM screening in non-pregnant patients. Educational Objective: Measurement of fasting blood sugar levels to screen for DM is recommended in (1) all persons aged 45 or above, and (2) all persons who are at high risk for DM. The following vignette applies to the next 2 items Item 1 of 2 A 26-year-old African American female comes to the emergency department with fever and a dry cough. She is also complaining of weakness, loss of appetite, a 10-lb weight loss, nausea, and mild constipation. She denies any medical problems in the past and was fine until four weeks ago, when all these symptoms started. Her symptoms are gradually worsening. She denies any headaches, visual problems, abdominal pain, skin rashes, dysuria or polyuria. She is currently taking over-the-counter acetaminophen. Her family history is unremarkable. She is a single mother, and lives with her three children. She has a 5-pack year history of smoking, and still smokes one pack daily. She occasionally drinks alcohol. The physical examination reveals a sick-appearing woman in no acute distress. Her blood pressure is 100/70 mm Hg, heart rate is 96/min regular, and temperature is 99? F (37.2C). Lymph nodes are palpated in the cervical, axillary and inguinal areas bilaterally. Her skin is normal; and there is no pallor or icterus. Her extremities have no clubbing, cyanosis or edema. The other systems are normal. Laboratory investigations: Serum Na: 142 mEq/L Serum K: 3.9 mEq/L Chloride: 100 mEq/L Bicarbonate: 32 mEq/L BUN: 32 mg/dL Serum Creatinine: 1.3 mg/dL

Calcium: 11.4 mg/dL Blood Glucose: 65 mg/dL Aspartate aminotransferase: 45 U/L Alanine aminotransferase: 56U/L Albumin: 3.2 g/l Serum TSH: 0.4 mU/ml Chest-x ray showed multiple parenchymal nodules. What is the most likely cause of her hypercalcemia? A. Hyperparathyroidism B. Hyperthyroidism C. Overproduction of 1, 25-dihydroxyvitamin D D. Renal failure E. Humoral hypercalcemia of malignancy Explanation: The patient has several clinical features suggestive of sarcoidosis. Sarcoid granulomas produce the enzyme 1-alpha-hydroxylase, which converts 25-hydroxyvitamin D to 1,25-dihydroxyvitamin D, leading to an increase in the gastrointestinal absorption of calcium. In hypercalcemia due to sarcoidosis, PTH secretion is suppressed, and urinary calcium excretion is increased. Hypercalcemia caused by overproduction of 1,25-dihydroxyvitamin D can also occur in other granulomatous diseases, such as fungal infections, berylliosis and tuberculosis. (Choices A and E) Hypercalcemia in this patient is unlikely to be induced by an increase in PTH or PTHrP. (Choice B) Hypercalcemia in thyrotoxicosis is very mild. Although some of the patient's clinical manifestations suggest thyrotoxicosis, her TSH level is borderline-low and not suppressed. Furthermore, the lymphadenopathy cannot be explained by thyrotoxicosis. (Choice D) The patient has an increased BUN level due to dehydration. This may be corrected with hydration, and is an unlikely cause of her moderately severe hypercalcemia. Educational Objective: Sarcoid granulomas produce the enzyme 1-alpha-hydroxylase, which converts 25-hydroxyvitamin D to 1,25-dihydroxyvitamin D, leading to an increase in the gastrointestinal absorption of calcium. The resulting hypercalcemia leads to a suppressed PTH secretion and an increased urinary calcium excretion. Item 2 of 2 Which of the following is the most appropriate next step in the management of this patient? A. Alendronate B. Glucocorticoids C. Hydration and glucocorticoids D. Hydration and Pamidronate E. Calcitonin Explanation: Since the hypercalcemia in sarcoidosis is caused by an increased calcium uptake, glucocorticoids and hydration are sufficient to bring the calcium levels to normal. Hypercalcemia secondary to sarcoidosis responds quickly to glucocorticoids. Serum calcium levels return to normal within days after starting 40-60 mg/d of prednisone. If within 10 days of glucocorticoid use, the serum calcium levels remain elevated, alternative diagnoses for hypercalcemia should be sought.

(Choice B) Since this patient is also dehydrated, giving intravenous fluids with a glucocorticoid is a better choice for her treatment than treatment with a glucocorticoid alone. (Choices A and D) Alendronate is a bisphosphonate given orally. It is approved for the treatment of osteoporosis and Paget?s disease. Its oral absorption is very poor (350 and viral loads
View more...

Comments

Copyright ©2017 KUPDF Inc.
SUPPORT KUPDF